Sei sulla pagina 1di 115

ACCSAP8 Exam1&2

Friday, September 07, 2012


7:11 AM

Question
1
of 60
Which of the following CVD risk prediction tools could be used in low-resource settings, where lab testing is
not readily available, yet provides effective risk discrimination?
A.The Framingham Heart Study risk equation with additional novel risk factors.
B.The Reynolds Risk Score.
C.The Harvard-derived NHANES risk score using age, sex, systolic blood pressure, smoking, and diabetes status
and that replaces BMI for cholesterol can predict high-risk patients equally as well when compared to the
Framingham risk score.
D.Just using blood pressure is sufficient because identifying patients at high absolute risk for CVD is not a costeffective prevention strategy.
The correct answer is C. The Harvard NHANES score can predict high-risk patients equally as well when
compared to the Framingham risk score. Novel risk factors, when added to the traditional risk factors (gender,
age, diabetes status, smoking status, blood pressure, total or LDL and HDL cholesterol) used in the
Framingham CVD or CHD risk equations, have not been shown to significantly increase the discriminatory
power (C-statistic) of the risk equations. The Reynolds Risk Score, which adds hs-CRP, and glycated
hemoglobin, and family history, did not significantly increase the C-statistic, but did lead to reclassification of
patients to higher- and lower-risk classifications. Identifying patients at high risk using risk scores has been
proven to be cost-effective in preventing CVD even in low-resource settings.
References
1. Wang TJ, Gona P, Larson MG, et al. Multiple Biomarkers for the Prediction of First Major Cardiovascular
Events and Death. N Engl J Med 2006;355:2631-9.
2. Ware JH. The limitations of risk factors as prognostic tools. N Engl J Med 2006;355:2615-7.
3. Ridker PM, Buring JE, Rifai N, Cook NR. Development and validation of improved algorithms for the
assessment of global cardiovascular risk in women: the Reynolds Risk Score. JAMA 2007;297:611-9.
4. Mendis S, Lindholm LH, Mancia G, et al. World Health Organization (WHO) and International Society of
Hypertension (ISH) risk prediction charts: assessment of cardiovascular risk for prevention and control of
cardiovascular disease in low and middle-income countries. J Hypertens 2007;25:1578-82.
5. Gaziano TA, Young CR, Fitzmaurice G, Atwood S, Gaziano JM. Laboratory-based versus non-laboratorybased method for assessment of cardiovascular disease risk: the NHANES I Follow-up Study cohort.
Lancet 2008;371:923-31.
6. Gaziano TA, Steyn K, Cohen DJ, Weinstein MC, Opie LH. Cost-effectiveness analysis of hypertension
guidelines in South Africa: absolute risk versus blood pressure level. Circulation 2005;112:3569-76.

BOARD Page 1

Question
2
of 60
A novel biomarker is being evaluated for its ability to diagnose PE in patients presenting to the emergency
department with suggestive symptoms. Previous studies have suggested that a biomarker level over 500 U is
diagnostic for PE. One hundred patients are enrolled; their levels of this biomarker are measured, but not
revealed to the treating physicians in the emergency department, who are asked to determine whether the
patient has a PE based on standard testing alone.
The study results are as follows:

PE Diagnosed

No PE Diagnosed
Biomarker level >500 U
40

10
Biomarker level <500 U
20

30
Which of the following percentages is the PPV of this biomarker for diagnosis of PE?
A.40%.
B.60%.
C.67%.
D.75%.
E.80%.

The correct answer is E. PPV is calculated as (true positives)/(true positives + false positives). For this
BOARD Page 2

The correct answer is E. PPV is calculated as (true positives)/(true positives + false positives). For this
example, the PPV is: (40)/(40 + 10) = 40/50 = 0.80 or 80%.

Question
3
of 60
A 79-year-old woman with a long history of hypertension who ran out of her medications about 2 weeks ago,
presents to the clinic with dyspnea on exertion, lower extremity edema, and mild orthopnea. Her blood
pressure is 170/110 mm Hg. Her ECG shows normal sinus rhythm and suggests LV hypertrophy. A
transthoracic echocardiogram confirms mild, concentric LV hypertrophy and a normal LV ejection fraction.
Which of the following is the most important long-term intervention for this patient?
A.Evaluation and treatment of diabetes mellitus.
B.Lifestyle modification (diet/exercise).
C.Identification and treatment of CAD.
D.Intensive lipid management.
E.Intensive blood pressure control.

The correct answer is E. The case presents a patient with diastolic heart failure, or heart failure with
preserved systolic function. Approximately 90% of patients with diastolic heart failure have a history of
hypertension. The treatment of hypertension reduces incident heart failure (especially in the elderly), and
exacerbations of heart failure (e.g., flash pulmonary edema) are often associated with hypertension. In such
patients, there have been no studies showing improved mortality with any of the medications studied in
randomized trials (e.g., ACE inhibitors, angiotensin II receptor blockers). While all of the interventions are
reasonable, hypertension control is the most important intervention. For her current congestive symptoms, a
diuretic will be indicated, and may also play a long-term role in her management.
References
1. Lindenfeld J, Albert NM, Boehmer JP, et al., on behalf of the Heart Failure Society of America. HFSA 2010
Comprehensive Heart Failure Practice Guideline. Section 11: Evaluation and Management of Patients
with Heart Failure and a Preserved Left Ventricular Ejection Fraction. J Card Fail 2010;16:e126-33.
2. Hunt SA, Abraham WT, Chin MH, et al. 2009 focused update incorporated into the ACC/AHA 2005
Guidelines for the Diagnosis and Management of Heart Failure in Adults: a report of the American
College of Cardiology Foundation/American Heart Association Task Force on Practice Guidelines:
developed in collaboration with the International Society for Heart and Lung Transplantation. J Am Coll
Cardiol 2009;53:e1-e90.

Question
BOARD Page 3

Question
4
of 60
A 55-year-old man with previous coronary bypass surgery notes worsening dyspnea on exertion, fatigue, and
pedal edema. On exam, he is noted to have elevated JVP that increases with inspiration and a loud early
diastolic sound. He is most likely to have which of the following?
A.Mitral stenosis.
B.Ischemic dilated cardiomyopathy.
C.Constrictive pericarditis.
D.ASD.

The correct answer is C. This patient has signs and symptoms of elevated RA pressure, a Kussmaul sign, and a
pericardial knock, which are all consistent with constrictive pericarditis. With mitral stenosis, a low frequency,
rumbling diastolic murmur should be present, as well as signs and symptoms of an elevated left atrial
pressure. The absence of an S3 and/or S4, AV valve regurgitation makes ischemic cardiomyopathy less likely.
Although right heart failure occurs in patients with longstanding uncorrected large ASD, a fixed split S2 and
systolic flow murmur (due to increased flow across the pulmonary valve) would be expected.

Question
5
of 60
A 45-year-old woman with a Framingham risk score of 2% and a family history of premature CHD presents for
risk assessment.
Referral for a calcium score in this patient would be considered which of the following?
A.Appropriate.
B.Uncertain appropriateness.
C.Inappropriate.
D.Unknown appropriateness.

The correct answer is A. The American College of Cardiology Foundation Cardiac CT Appropriate Use Criteria
BOARD Page 4

The correct answer is A. The American College of Cardiology Foundation Cardiac CT Appropriate Use Criteria
and other Expert Consensus documents primarily identify patients with an intermediate risk of CHD as
appropriate candidates for calcium scoring. The basis for this recommendation lies in the potential to alter
patient management through reclassification of risk to a higher or lower level according to the scan findings.
One exception when low-risk patients are appropriate for scanning is in the presence of a family history of
CHD.
References
1. Taylor AJ, Cerqueira M, Hodgson JM, et al. ACCF/SCCT/ACR/AHA/ASE/ASNC/NASCI/SCAI/SCMR 2010
appropriate use criteria for cardiac computed tomography. A report of the American College of
Cardiology Foundation Appropriate Use Criteria Task Force, the Society of Cardiovascular Computed
Tomography, the American College of Radiology, the American Heart Association, the American Society
of Echocardiography, the American Society of Nuclear Cardiology, the North American Society for
Cardiovascular Imaging, the Society for Cardiovascular Angiography and Interventions, and the Society
for Cardiovascular Magnetic Resonance. J Am Coll Cardiol 2010;56:1864-94.
2. Greenland P, Bonow RO, Brundage BH, et al. ACCF/AHA 2007 clinical expert consensus document on
coronary artery calcium scoring by computed tomography in global cardiovascular risk assessment and
in evaluation of patients with chest pain: a report of the American College of Cardiology Foundation
Clinical Expert Consensus Task Force (ACCF/AHA Writing Committee to Update the 2000 Expert
Consensus Document on Electron Beam Computed Tomography) developed in collaboration with the
Society of Atherosclerosis Imaging and Prevention and the Society of Cardiovascular Computed
Tomography. J Am Coll Cardiol 2007;49:378-402.

Question
6
of 60
A 52-year-old woman undergoes cardiac catheterization to evaluate dyspnea, which has become progressive
over the last year. She smoked one pack of cigarettes per day for 12 years but quit 20 years ago. She had
noted some lower extremity edema and some weight gain. Four months ago, her physician prescribed
furosemide 20 mg/day, which improved her lower extremity edema, but her dyspnea continued to progress.
The mean RA and LA pressures are 20 mm Hg, the PA pressure is 42/24 mm Hg, and the Fick cardiac index is
2.1 l/m/m2. The RV and LV pressure tracings are shown in Figure 1.

BOARD Page 5

Based on the hemodynamic findings, which of the following is the most likely cardiac diagnosis?
A.Hypertrophic obstructive cardiomyopathy.
B.Cor pulmonale.
C.Restrictive cardiomyopathy.
D.Constrictive pericarditis.
E.Primary pulmonary hypertension.

The correct answer is D. The hemodynamic finding of elevated and equal filling pressures and ventricular
discordance with respiration is consistent with the diagnosis of constrictive pericarditis. Restrictive
cardiomyopathy causes elevated filling pressures but ventricular concordance with respiration. Cor pulmonale
and primary pulmonary hypertension do not cause elevated left-sided filling pressures. Hypertrophic
cardiomyopathy can cause elevated filling pressures but does not cause ventricular discordance.

Question
7
of 60
A 52-year-old woman with hypertension presents to the emergency department with the acute onset of chest
BOARD Page 6

A 52-year-old woman with hypertension presents to the emergency department with the acute onset of chest
pain and markedly elevated blood pressure. You discover that she is not having an acute ST elevation
myocardial infarction, but are concerned she may be having an aortic dissection.
Based on the results of the imaging study (Figure 1), which of the following do you recommend as the next
best step in management of this patient?

A.Medical therapy to reduce her blood pressure and repeat imaging in 1 week.
B.Medical therapy to reduce her blood pressure and immediate surgical repair.
C.Medical therapy to reduce her blood pressure and surgical repair once her hypertension is under control.
D.Medical therapy to reduce her blood pressure and endovascular stent placement.
E.Medical therapy alone.

The correct answer is B. The image shows a Stanford type A or DeBakey type I aortic dissection. Therapy for
this type of dissection is immediate surgical repair, because mortality increases with time; therefore, options
A, C, D, and E are incorrect.

Question
8
of 60

BOARD Page 7

An 85-year-old male patient is undergoing precatheterization evaluation and is found to have a Cr of 2.8
mg/dl, heart failure, anemia, and diabetes. You plan to use approximately 100 cc of iodinated contrast. He is
hemodynamically stable. You have time to provide a prophylaxis.
Which of the following would be the best management approach?
A.Intravenous half normal saline 75 cc/h, 1 hour before and during the procedure.
B.Intravenous sodium bicarbonate solution, 300 cc 1-3 hours before the procedure.
C.Intravenous normal saline 300 cc 1-3 hours before the procedure at 150 ml/h for up to 6 hours after the
procedure as feasible to ensure a post-procedure urine output of 150 cc/h.
D.Increase oral hydration the day before the procedure, and then intravenous normal saline 75 cc/h during
the procedure.

The correct answer is C. There are no proven preventive agents for CI-AKI. Volume depletion is recognized to
be a risk factor for all forms of AKI, and thus, the use of intravenous fluids reduces this contributing factor.
Indirect evidence suggests that an elevated urine flow rate may allow more urinary losses of contrast and less
uptake in proximal tubular cells, thus, less CI-AKI.
References
1. Goldfarb S, McCullough PA, McDermott J, Gay SB. Contrast-induced acute kidney injury: specialtyspecific protocols for interventional radiology, diagnostic computed tomography radiology, and
interventional cardiology. Mayo Clin Proc 2009;84:170-9.
2. Stevens MA, McCullough PA, Tobin KJ, et al. A prospective randomized trial of prevention measures in
patients at high risk for contrast nephropathy: results of the P.R.I.N.C.E. Study. Prevention of
Radiocontrast Induced Nephropathy Clinical Evaluation. J Am Coll Cardiol 1999;33:403-11.

Question
9
of 60
A 64-year-old tobacco farmer is seen in clinic on referral from his primary care physician for vascular
evaluation. The patient has a 40-year history of smoking and family history of vascular disease. He reports that
he has worked hard for most of his life, and over the last year, has noted an increase in pain in his buttocks
and calves with walking. He states that when walking up a hill to a barn on the property (200 yards or so), he
has significant discomfort in both legs and sometimes now has to stop to rest. His left leg takes longer to
resolve than his right leg.
His physical examination is notable for blood pressure of 148/85 mm Hg and heart rate of 44 bpm with regular
rhythm. He has bilateral carotid bruits, evidence of tobacco staining on his fingernails, no evidence of enlarged
abdominal aorta on examination, +2 femoral pulses bilaterally, +1 popliteal pulses, and palpable DP and PT
pulses in both legs. He has a rest and exercise ABI performed; the results are shown (Figures 1a, b).

BOARD Page 8

Which of the following is your interpretation and plan?


A.Normal rest and exercise ABIno evidence of PAD.
B.Normal rest ABIevaluate for sciatica.
C.Normal rest ABI with abnormal exercise ABI consistent with PADrecommend pentoxifylline.

BOARD Page 9

D.Normal rest ABI with abnormal exercise ABIrecommend angiography.


E.Normal rest ABI with abnormal exercise ABI consistent with PADrecommend structured exercise and
cilostizol.

The correct answer is E. The question identifies a prototypical patient with intermittent claudication. In the
face of normal resting ABI, an exercise ABI helps to confirm the diagnosis. These findings are not consistent
with sciatica, and the exercise ABI demonstrates ABI <0.70 after 5 minutes of exercise 1 minute into recovery.
The ACC/AHA PAD guideline recommendations for lifestyle limiting claudication include initial management
with a structured exercise program and cilostazol, which has been shown to improve walking distance.
Pentoxifylline is not recommended, as it did not improve walking distance beyond that of placebo in
randomized trials. The use of angiography and revascularization is intended for patients that fail structured
exercise and medical therapy.
References
1. Hirsch AT, Haskal ZJ, Hertzer NR, et al. ACC/AHA 2005 guidelines for the management of patients with
peripheral arterial disease (lower extremity, renal, mesenteric, and abdominal aortic): executive
summary a collaborative report from the American Association for Vascular Surgery/Society for Vascular
Surgery, Society for Cardiovascular Angiography and Interventions, Society for Vascular Medicine and
Biology, Society of Interventional Radiology, and the ACC/AHA Task Force on Practice Guidelines
(Writing Committee to Develop Guidelines for the Management of Patients With Peripheral Arterial
Disease) endorsed by the American Association of Cardiovascular and Pulmonary Rehabilitation;
National Heart, Lung, and Blood Institute; Society for Vascular Nursing; TransAtlantic Inter-Society
Consensus; and Vascular Disease Foundation. J Am Coll Cardiol 2006;47:1239-312.

Question
10
of 60
TJ is a 70-year-old retired school teacher with a strong family history of premature heart disease. She is
worried about her own risk for CHD and makes an appointment for preventive evaluation. She has no prior
history of CHD, does not smoke, and exercises regularly. She has no symptoms suggestive of cardiovascular
disease.
Her blood pressure is 120/80 mm Hg and heart rate is 54 bpm. Her BMI is 23 kg/m2.
Her lipid profile: Total cholesterol 250 mg/dl, LDL-C 151 mg/dl, HDL-C 42 mg/dl, and triglycerides 120 mg/dl.
Her calculated Framingham 10-year risk is 5%.
Initial preventive therapy should include which of the following?
A.Lifestyle moderation.

BOARD Page 10

B.Statin for an LDL <130 mg/dl.


C.Statin and niacin for an LDL <130 mg/dl and HDL >50 mg/dl.
D.Lifestyle modification and statin for LDL <130 mg/dl.

The correct answer is D. This patient has two risk factors for CHD: age of 55 years and family history of CHD.
These risk factors and a 10-year risk of CHD <10% place her at moderate risk, and would indicate an LDL goal
of <130 mg/dl. Lifestyle modification, together with a statin, would most likely result in obtaining this LDL
target.
References
1. Grundy SM, Cleeman JI, Merz CN, et al., on behalf of the National Heart, Lung, and Blood Institute;
American College of Cardiology Foundation; American Heart Association. Implications of recent clinical
trials for the National Cholesterol Education Program Adult Treatment Panel III guidelines. Circulation
2004;110:227-39.
2. Expert Panel on Detection, Evaluation, and Treatment of High Blood Cholesterol in Adults. Executive
Summary of the Third Report of the National Cholesterol Education Program (NCEP) Expert Panel on
Detection, Evaluation, and Treatment of High Blood Cholesterol in Adults (Adult Treatment Panel III).
JAMA 2001;285:2486-97.

Question
11
of 60
A 38-year-old female with a history of lupus presents for a discussion of cardiac risk. Her mother has a history
of hypertension at age 70. Her father has diabetes. Her brothers have obesity. Her child is in good health. The
patient works as an IT consultant and does not exercise regularly.
On physical exam, her heart rate is 85 bpm and blood pressure is 125/80 mm Hg. In general, she is wellappearing. Her cardiovascular exam demonstrates regular rate and rhythm, normal S1 and S2 and no
murmurs, rubs, or gallops. She has no edema.
Which of the following is TRUE regarding CVD in women?
A.Breast cancer is the leading cause of death in women.
B.Presence of lupus does not affect a womans risk of CVD.
C.The presence of diabetes is a relatively greater risk factor for men than women.
D.Goal level of HDL-C in women is 40 mg/dl.
E.Among patients under age 50, acute MI mortality is higher in women than men.

BOARD Page 11

The correct answer is E. Among patients under age 50, acute MI mortality is higher in women than men.1 This
may be surprising to many, because CHD events lag 10 years in women compared to men.2
CVD, not breast cancer, is the leading cause of death in women.3
Systemic autoimmune collagen-vascular disease is listed as a criterion for the at risk status.4
The presence of diabetes is a relatively greater risk factor for CHD in women compared with men, increasing a
womans risk of CHD by three- to sevenfold with only a two- to threefold increase in diabetic men.5
Goal HDL-C in women is 50 mg/dl and in men is 40 mg/dl.4
References
1. Vaccarino V, Parsons L, Every NR, Barron HV, Krumholz HM. Sex-based differences in early mortality
after myocardial infarction. National Registry of Myocardial Infarction 2 Participants. N Engl J Med
1999;341:217-25.
2. Lerner DJ, Kannel WB. Patterns of coronary heart disease morbidity and mortality in the sexes: A 26-year
follow-up of the Framingham population. Am Heart J 1986;111:383-90.
3. Roger VL, Go AS, Lloyd-Jones DM, et al., on behalf of the American Heart Association Statistics
Committee and Stroke Statistics Subcommittee. Heart disease and stroke statistics--2011 update: a
report from the American Heart Association. Circulation 2011;123:e18-e209.
4. Mosca L, Benjamin EJ, Berra K, et al. Effectiveness-based guidelines for the prevention of cardiovascular
disease in women--2011 update: a guideline from the American Heart Association. Circulation
2011;123:1243-62.
5. Huxley R, Barzi F, Woodward M. Excess risk of fatal coronary heart disease associated with diabetes in
men and women: meta-analysis of 37 prospective cohort studies. BMJ 2006;332:73-8.

Question
12
of 60
Which of the following agents is/are reversible P2Y12 receptor inhibitor/s?
A.Ticlopidine.
B.Clopidogrel.
C.Prasugrel.
D.Ticagrelor.
E.All of the above.

The correct answer is D. Ticlopidine, clopidogrel, and prasugrel are thienopyridines. These prodrugs undergo
hepatic conversion to active metabolites that bind irreversibly to the P2Y12 receptor. Ticagrelor is a direct
BOARD Page 12

hepatic conversion to active metabolites that bind irreversibly to the P2Y12 receptor. Ticagrelor is a direct
P2Y12 receptor inhibitor that binds irreversibly to the P2Y12 receptor.

Question
13
of 60
Which one of the following conditions or diagnoses is an absolute contraindication to fibrinolytic
administration?
A.ICH 4 months prior to presentation.
B.Acute ischemic stroke with onset of symptoms 2 hours before presentation.
C.Prolonged CPR >10 minutes.
D.Pregnancy.
E.Active treatment with warfarin with therapeutic international normalized ratio between 2 and 3.

The correct answer is A. Prior ICH is an absolute contraindication to fibrinolysis, regardless of the interval
before presentation. An acute ischemic stroke is not an absolute contraindication and in fact may benefit from
fibrinolytic therapy (although the dose/agent may differ from treatment of acute MI). Prolonged CPR,
pregnancy, and active anticoagulation may require caution but are all relative contraindications to fibrinolytic
therapy and are not absolute.
References
1. Antman EM, Anbe DT, Armstrong PW, et al. ACC/AHA guidelines for the management of patients with
ST-elevation myocardial infarction--executive summary: a report of the American College of
Cardiology/American Heart Association Task Force on Practice Guidelines (Writing Committee to revise
the 1999 guidelines for the management of patients with acute myocardial infarction). J Am Coll Cardiol
2004;44:671-719.

Question
14
of 60
A-75-year old diabetic man is admitted to the coronary care unit after a drug-eluting stent (DES) placement in
his mid right coronary artery following an acute inferior wall MI. He had earlier received 600 mg of clopidogrel
in the emergency department and had been administered a bolus of bivalirudin followed by an IV infusion. He
smokes one pack of cigarettes per day and has a history of a prior ischemic stroke with no residual in the
remote past. He has a history of mild claudication in the calf when walking uphill. On arrival to the unit, his
ECG shows sinus rhythm at 72, inferior Q waves with almost complete resolution of his inferior ST elevation.
BOARD Page 13

ECG shows sinus rhythm at 72, inferior Q waves with almost complete resolution of his inferior ST elevation.
He is pain free, blood pressure is 148/70 mm Hg, and lungs are clear to auscultation with adequate air entry.
Cardiac examination reveals an LV fourth heart sound and a loud aortic component of the second heart sound.
Initial labs are pertinent for a troponin T of 0.03 ng/ml, a creatinine of 1.3 mg/dl, and blood glucose of 180
mg/dl.
Which of the following medical interventions is unwarranted?
A.Initiate treatment with Lopressor 25 mg orally every 8 hours.
B.Initiate treatment with ramipril 5 mg/day.
C.Initiate IV insulin to target blood sugar control of 80-100 mg/dl.
D.Continue treatment with clopidogrel 75 mg/day.
E.Initiate treatment with pantoprazole 40 mg/day.

The correct answer is C. In the absence of absolute contraindications, beta-blockade benefits most
hemodynamically stable patients with an STEMI. An ACEI is a preferred agent in a diabetic individual with
hypertension. Extrapolating data from a stable CAD population, the initiation of ramipril in this subject may
also reduce the long-term risk of recurrent MI, stroke, and death. Dual antiplatelet therapy is indicated in the
setting of stent placement, and the use of a proton pump inhibitor (PPI) will decrease the risk of
gastrointestinal bleeding. Although a PPI should be used with caution, concerns for a PPI/clopidogrel
interaction have not translated into confirmed clinical harm. While achieving blood sugar control is reasonable
in this setting, current targets support a more conservative target in the 180 mg/dl range with IV insulin and
not a target in the 80-100 mg/dl range.

Question
15
of 60
A 71-year-old woman presents with dyspnea and fatigue. She reports that the dyspnea comes on while going
up one flight of stairs and when walking her dog, though if she slows down and rests, she can often continue
walking. She does not have any history of CAD, but has hypertension and type 2 diabetes. She is taking
metformin. Her ECG reveals LV hypertrophy with strain. She underwent an exercise stress test with
radionuclide imaging and exercised for 430 on a standard Bruce protocol. Her blood pressure response was
normal. She developed dyspnea and epigastric tightness at 4 minutes, eventually stopping due to the dyspnea.
Her ECG did not change. Her radionuclide images demonstrated areas of reversible ischemia in the apical
anterior, inferoseptal, and apical segments. Her calculated ejection fraction was 42%.
Which of the following is the next most appropriate test?
A.Stress echocardiography.

BOARD Page 14

B.CTA.
C.Coronary angiography.
D.No testing is needed at this time; begin optimal medical therapy.

The correct answer is C. This patient has several features that indicate that she is at an intermediate-to-high
risk for future CV disease. Her intermediate features are a mildly reduced LV function and an intermediate-risk
treadmill score (even without calculable ST-segment depression, her DTS would be -5). The next appropriate
test would be coronary angiography to define the degree of atherosclerosis. Treatment decisions could then
be best determined based on the extent of disease. In this case, another stress test with echocardiography
would not give additional value, although a standard echocardiography would offer further data regarding LV
function, wall-motion abnormalities, and valvular disease. CTA is not indicated because this patient should
proceed to catheterization, and the CTA would only expose her to additional contrast and radiation.
References
1. Fox K, Garcia MA, Ardissino D, et al. Guidelines on the management of stable angina pectoris: executive
summary: the Task Force on the Management of Stable Angina Pectoris of the European Society of
Cardiology. Eur Heart J 2006;27:1341-81.
2. Gibbons RJ, Abrams J, Chatterjee K, et al. ACC/AHA 2002 guideline update for the management of
patients with chronic stable angina-summary article: a report of the American College of
Cardiology/American Heart Association Task Force on practice guidelines (Committee on the
Management of Patients With Chronic Stable Angina). J Am Coll Cardiol 2003;41:159-68.
3. Gibbons RJ, Balady GJ, Bricker JT, et al. ACC/AHA 2002 guideline update for exercise testing: summary
article. A report of the American College of Cardiology/American Heart Association Task Force on
Practice Guidelines (Committee to Update the 1997 Exercise Testing Guidelines). J Am Coll Cardiol
2002;40:1531-40.

Question
16
of 60
A 63-year-old man is evaluated for symptoms of chest discomfort. He reports a 3-month history of substernal
chest tightness provoked by moderate exertion, such as walking uphill to his mailbox. The symptoms occur
almost daily. His medications include aspirin 81 mg daily, simvastatin 20 mg daily, and ibuprofen as needed.
Stress testing demonstrated a mild reversible apical perfusion defect, and coronary angiography revealed a
focal 80% stenosis in the mid-LAD artery and no other obstructive lesions.
Which of the following is the most appropriate next step in his treatment?
A.Refer for CABG with a LIMA graft.
B.Prescribe metoprolol and sublingual nitroglycerin.
C.Refer for PCI of the LAD artery.
BOARD Page 15

C.Refer for PCI of the LAD artery.


D.Increase simvastatin to 40 mg daily.
E.Refer for PCI of the LAD artery and start metoprolol.

The correct answer is B. This patient has chronic stable CCS class II angina. Low-risk findings were
demonstrated on noninvasive stress testing, and angiography revealed single-vessel CAD involving the midLAD artery. In this case, maximal medical therapy is recommended with an antianginal agent such as a betablocker or a long-acting nitrate. Revascularization is not recommended in patients with single-vessel CAD, mild
symptoms, and low-risk noninvasive findings.1
References
1. Patel MR, Dehmer, GJ, Hirshfeld JW, et al. ACCF/SCAI/STS/AHA/ASNC 2009 appropriateness criteria for
coronary revascularization. J Am Coll Cardiol 2009;53:530-53.

Question
17
of 60
A 73-year-old man with diabetes and hyperlipidemia is admitted to the hospital with heart failure. As part of
your evaluation, you determine that he has an LVEF of 20% and significant three-vessel CAD.
Which of the following results would be most predictive of identifying hibernating, viable myocardium that is
likely to recover function after revascularization in this patient?
A.A biphasic response with augmentation in wall motion and endocardial thickening at low doses of
dobutamine and deterioration at higher doses of dobutamine.
B.Continued deterioration of wall motion and endocardial thickening with increasing doses of dobutamine.
C.A biphasic response with deterioration of wall motion and endocardial thickening at low doses of
dobutamine and augmentation in wall-motion abnormalities at higher doses of dobutamine.
D. No change in wall-motion abnormalities with either low or high doses of dobutamine.
E.Sustained improvement in wall motion and endocardial thickening with increasing doses of dobutamine.

The correct answer is A. Options B, C, D, and E are incorrect because they have not been shown to best
correlate with improvement in wall motion and endocardial thickening after revascularization.

Question

BOARD Page 16

18
of 60
A 62-year-old woman presents to your office complaining of chest discomfort, which has been increasing in
frequency over the past 2 years. At first she noticed the discomfort with extreme exertion, while running with
her grandchildren. The discomfort began in her substernal region and radiated to her back and left shoulder,
and resolved with rest. She attributed these symptoms to getting old and being out of shape. About 9 months
ago, she noticed that these episodes were occurring about once a week and now were precipitated by either
physical activity (walking up or down the stairs, doing household chores with her arms) or emotional stress
related to her job as a teacher of autistic children.
Although these episodes continue to resolve with rest, she is concerned about whether she should retire. She
has a 50 pack-year smoking history and quit 2 years ago when these episodes first started. She admits to poor
dietary habits, history of hypertension, and infrequent migraine both diagnosed in her late 40s and currently
treated with enalapril 10 mg daily and hydrochlorothiazide 12.5 mg daily. She has dyslipidemia treated with
lovastatin 20 mg daily for 2 years with total cholesterol of 215 mg/dl, low-density lipoprotein (LDL) of 125
mg/dl, high-density lipoprotein of 32 mg/dl, and triglycerides of 178 mg/dl. She does not have diabetes. She
has never seen a cardiologist and has never had an ECG or any other cardiovascular testing.
Her father died of a fatal myocardial infarction at the age of 62, and her mother died of lung cancer at the age
of 58. Her older brother has had coronary artery bypass grafting, and her younger sister has dyslipidemia and
diabetes.
Other than being overweight (5 feet 3 inches, 174 lbs) with a blood pressure today of 162/89 mm Hg, her
exam is unremarkable.
Her cardiologist determines that she has stable angina, and that her blood pressure and LDL need better
control. Before addressing her retirement question and per ACC/American Heart Association (AHA) guidelines,
the cardiologist discusses the need for additional noninvasive cardiac risk stratification. Her resting ECG was
normal, and enalapril was increased to 20 mg, hydrochlorothiazide to 25 mg daily, and lovastin to 40 mg daily.
After a lengthy discussion, the patient was not interested in taking a long-acting nitrate, fearing that it would
provoke her migraine. Likewise, she did not want to take a beta-blocker or calcium antagonist because she
had experienced fatigue and edema when she had been prescribed these agents over the previous years for
hypertension.
The patient returns to the office nurse for blood pressure checks over the next 3 weeks. Then an exercise
stress test was done using the Bruce protocol with Duke treadmill scoring. Her stress test results are as
follows:
Resting heart rate: 80 bpm; resting blood pressure 128/78 mm Hg.
Time on treadmill: 9 minutes and 30 seconds without chest discomfort.
Peak heart rate: 142 bpm; peak blood pressure 185/92 mm Hg.
At peak exercise, she exhibits a 1.5 mm horizontal to downsloping ST depression in inferior leads, consistent
with ischemia with fairly high workload. She did not have any symptoms. An immediate postexercise
echocardiogram reveals an ejection fraction of 65% and no wall motion abnormalities. Her ST-segment
depression resolved completely in 1 minute.
BOARD Page 17

depression resolved completely in 1 minute.


The patient returns to your office for post-test discussion and counseling. Her blood pressure has been 125/80
to 135/84 mm Hg systolic at home. She states that she has had no further episodes of chest discomfort, but
notes that it has only been 4 weeks.
You explain that she is most likely to benefit from which of the following?
A.A nuclear perfusion study with adenosine.
B.An elective coronary angiogram.
C.Yearly exercise stress tests.
D.Retiring from teaching position early.
E.An exercise program.

The correct answer is E. Her clinical findings are clearly abnormal (angina, hypertension,
hypercholesterolemia, ST-segment depression with exercise), and are associated with mild impairment of her
prognosis compared with women of similar age without these findings. Yet other than better blood pressure
and LDL-C control and an exercise program, she is not likely to derive additional benefit from any of the above
additional tests. Because her LV wall motion was not impaired and the ST-segment shifts resolve promptly and
she is currently asymptomatic, it is very unlikely that she has multiple vessel severe obstructive CAD. She also
could have nonobstructive CAD with microvascular dysfunction. Finally, if she remains asymptomatic at her
work place, there is no benefit to recommending early retirement or a change in her job.

Question
19
of 60
An otherwise healthy 54-year-old man was referred to your office for evaluation of a heart murmur. He has no
significant medical history and takes no medications. On physical examination, his blood pressure was 110/78
mm Hg, with a heart rate of 56 bpm, and he had a grade I/IV decrescendo diastolic murmur. An
echocardiogram revealed a BAV with a horizontal commissure, mild aortic insufficiency, an aortic root
diameter of 3.6 cm, and an ascending thoracic aortic diameter of 5.1 cm.
Based on these findings, the most appropriate management strategy would be to do which of the following?
A.Prescribe antibiotic prophylaxis for dental visits.
B.Prescribe a beta-blocker.
C.Prescribe an angiotensin-converting enzyme inhibitor.

BOARD Page 18

D.Refer the patient for surgery to repair the ascending thoracic aorta.
E.Obtain annual surveillance echocardiograms to monitor the diameter of the ascending thoracic aorta and
the severity of the aortic insufficiency.

The correct answer is D. Whereas the threshold for surgery for idiopathic ascending thoracic aortic
aneurysms is a diameter of 5.5 cm, in patients with a BAV, surgery is indicated once the ascending aorta
reaches a diameter of 5.0 cm.
The remaining answers are incorrect for the following reasons. Based on the most recent ACC/AHA guidelines,
native valvular heart disease is no longer an indication for antibiotic prophylaxis. Beta-blockers have been
shown to slow the rate of growth of aortic aneurysms in patients with Marfan syndrome, and by extension,
they have also been used routinely in the treatment of TAAs of other etiologies. However, in this case, the
patient's blood pressure and heart rate are too low to reasonably add a beta-blocker.
Although afterload reduction may be considered in hypertensive patients with aortic insufficiency, this
patient's blood pressure is too low to justify adding an angiotensin-converting enzyme inhibitor. Also, if the
aortic diameter was too small to merit surgical repair, then annual surveillance imaging to monitor aortic size
would be appropriate; however, given this patient's aortic diameter, surgery is indicated.
References
1. Hiratzka LF, Bakris GL, Beckman JA, et al. 2010 ACCF/AHA/AATS/ACR/ASA/SCA/SCAI/SIR/STS/SVM
Guidelines for the diagnosis and management of patients with thoracic aortic disease. A Report of the
American College of Cardiology Foundation/American Heart Association Task Force on Practice
Guidelines, American Association for Thoracic Surgery, American College of Radiology, American Stroke
Association, Society of Cardiovascular Anesthesiologists, Society for Cardiovascular Angiography and
Interventions, Society of Interventional Radiology, Society of Thoracic Surgeons, and Society for Vascular
Medicine. J Am Coll Cardiol 2010;55:e27-e129.

Question
20
of 60
A 78-year-old man with a known history of coronary artery disease, including three-vessel coronary artery
bypass graft (CABG) 7 years ago (left internal mammary artery [LIMA] to left anterior descending [LAD],
saphenous vein graft [SVG] to obtuse marginal [OM], SVG to posterior descending artery [PDA]), presents with
exertional chest discomfort. On physical examination, his blood pressure (BP) is 128/70 mm Hg in the right
arm. Carotid, left supraclavicular, and bilateral femoral bruits are identified. Pulse examination discloses 2+
pulses throughout the body except for the left brachial and radial pulses, which are trace to 1+.
Which of the following pairs of physical examination and noninvasive imaging studies most likely would
disclose the lesion responsible for the patients angina?
A.Bilateral arm BPs and computed tomography angiography (CTA) of the thorax.
B.An ABI and CTA of the pelvis with lower extremity runoff.

BOARD Page 19

C.Auscultation of the abdomen and abdominal duplex.


D.Doppler assessment of the pedal pulses and exercise ABI.

The correct answer is A. The physical examination finding of diminished pulses in the left arm raises suspicion
for left subclavian artery (LSA) stenosis. The fact that this patient has angina after prior CABG surgery, which
included a LIMA graft to the LAD, suggests the possibility of LIMA graft insufficiency (or potentially "steal") in
the context of LSA stenosis. Measuring bilateral arm BPs should disclose a lower BP in the left arm than the
right (>15 mm Hg discrepancy is strongly suggestive of LSA stenosis) and a CTA of the thorax would show the
left subclavian lesion.
The other suggestions would be appropriate to evaluate lower extremity PAD (choice B or D) or to rule out
abdominal aortic aneurysm or renal artery stenosis (choice C), but none of these would shed light on a source
of angina.

Question
21
of 60
A 63-year-old woman is seen in her primary care physicians office for routine follow-up. She has a history of
diabetes and hypertension. Last year, she underwent successful right coronary artery stenting for an acute
coronary syndrome. At that time, she quit tobacco use, having had a previous 75 pack/year history. She has no
complaints.
On exam, her blood pressure is 132/82 mm Hg, heart rate is 68 bpm, height is 167 cm, and weight is 48 kg. Her
lungs are clear; her heart has regular rate and rhythm; and normal S1, S2 without murmurs, rubs, or gallops.
There are normal bowel sounds; scaphoid abdomen is without masses or tenderness. There is no edema of
the lower extremities. Carotid, femoral, and distal pulses are unremarkable. There is a bruit present over the
left femoral artery.
On her visit last year, she weighed 61 kg. She has not made an effort to lose weight or changed her exercise
routine. On review of systems, she reports that she has noticed diffuse abdominal pain after eating. She has
been unable to correlate this with any particular foods or to position. The pain is difficult to localize. She has
learned to avoid meals as a solution to this abdominal pain
Which of the following is the next best step in the management of this patient?
A.Recommend over-the-counter proton pump inhibitor.
B.Right upper quadrant ultrasound to exclude cholecystitis.
C.Right upper quadrant ultrasound to exclude cholelithiasis.
D.Abdominal CT with contrast to exclude tumor.

BOARD Page 20

E.Abdominal duplex ultrasound of visceral vessels.

The correct answer is E. This woman has a history of known atherosclerosis with coronary stenting last year.
She has a history of tobacco abuse, placing her at threefold increased risk for peripheral arterial disease. On
examination, there is a bruit present in her left femoral artery, suggesting that she in fact has peripheral
arterial disease. The absence of a bruit on abdominal exam does not exclude the presence of stenosis of the
visceral vessels. She has lost 13 kg in 1 year, an unintended weight loss. Her abdominal pain is difficult to
localize or characterize. She has learned to avoid food to minimize her pain.
An abdominal duplex ultrasound is a good screening test for CMI. It can be accomplished in 85% of elective
cases, with a 90% accuracy for the detection of >70% stenosis. This is in contrast to patients with acute
mesenteric ischemia, where the abdominal pain and tenderness, combined with bowel ischemia makes
imaging challenging. Duplex ultrasound can be completed without radiation or contrast exposure and is a
good initial screening test.
There is no evidence of fever or infection, making cholecystitis unlikely. The presence of stones in the gall
bladder alone does not make the diagnosis of biliary colic. Further, the absence of a stone in the gall bladder
does not preclude biliary colic, as the stone may be in the bile duct and not seen on the ultrasound. Biliary
colic is typically localized to the right upper quadrant, and more often correlated with fatty foods in particular.
Although cancer may cause weight loss, she has no other constitutional complaints. There is no abdominal
mass on exam. Her history of coronary artery disease, and the presence of peripheral arterial disease on exam
make the diagnosis of CMI more likely. Thus, the test without the risk of contrast and radiation should be the
initial test performed. Although CTA can visualize visceral artery stenosis, it must be performed with a
dedicated CTA protocol. Typical scan protocols for abdominal studies will not provide sufficient resolution.
References
1. Hirsch AT, Haskal ZJ, Hertzer NR, et al. ACC/AHA 2005 guidelines for the management of patients with
peripheral arterial disease (lower extremity, renal, mesenteric, and abdominal aortic): executive
summary a collaborative report from the American Association for Vascular Surgery/Society for Vascular
Surgery, Society for Cardiovascular Angiography and Interventions, Society for Vascular Medicine and
Biology, Society of Interventional Radiology, and the ACC/AHA Task Force on Practice Guidelines
(Writing Committee to Develop Guidelines for the Management of Patients With Peripheral Arterial
Disease) endorsed by the American Association of Cardiovascular and Pulmonary Rehabilitation;
National Heart, Lung, and Blood Institute; Society for Vascular Nursing; TransAtlantic Inter-Society
Consensus; and Vascular Disease Foundation. J Am Coll Cardiol 2006;47:1239-312.

Question
22
of 60
Atherosclerosis is the etiology of the vast majority of carotid artery stenotic disease. Nonatherosclerotic
carotid artery disease must always be considered, however, as this potentially alters treatment strategies.
Which of the following statements is TRUE regarding characteristics suggestive of nonatherosclerotic carotid
disease?

BOARD Page 21

A.Radiation-induced carotid stenosis is most commonly a focal stenotic lesion localized to the carotid bulb and
proximal ICA.
B.Carotid artery dissection typically involves the proximal ICA and is not accompanied by cranial nerve palsy.
C.Fibromuscular dysplasia rarely involves more than one cervical vessel and is not associated with intracranial
aneurysm.
D.Giant cell arteritis can involve the carotid arteries, but does not affect the aorta or its other branch vessels.
E.Takayasu arteritis involving the carotid artery should be suspected in a young adult presenting with neck
pain and noted to have diminished brachial pulses.

The correct answer is E. Option A is incorrect. Radiation-induced carotid artery disease often affects long
segments of the CCA and/or ICA, with preferential involvement of the CCA.1
Option B is incorrect. Carotid artery dissection typically begins more than 2 cm above the carotid bifurcation
within the mobile segment of the ICA.2
Option C is incorrect. Fibromuscular dysplasia often involves more than one cervical vessel and is associated
with a high incidence of intracranial aneurysm.3
Option D is incorrect. Though most commonly appreciated for its involvement of the temporal and other facial
arteries, it can also involve the cervical arteries, aortic branch vessels, and the aorta, the latter potentially
causing aneurysm and rupture.4
Option E is correct. Takayasu arteritis, or "pulseless disease", most commonly causes stenosis of the arteries
to the upper extremities, but can also involve the carotid arteries and the aortic branch vessels.5
References
1. Lam WW, Liu KH, Leung SF, et al. Sonographic characterisation of radiation-induced carotid artery
stenosis. Cerebrovasc Dis 2002;13:168-73.
2. Debette S, Leys D. Cervical-artery dissections: predisposing factors, diagnosis, and outcome. Lancet
Neurol 2009;8:668-78.
3. Olin JW, Sealove BA. Diagnosis, management, and future developments of fibromuscular dysplasia. J
Vasc Surg 2011;53:826-36.e1.
4. Martinez-Valle F, Solans-Laque R, Bosch-Gil J, Vilardell-Tarres M. Aortic involvement in giant cell
arteritis. Autoimmun Rev 2010;9:521-4.
5. Arnaud L, Haroche J, Toledano D, et al. Cluster analysis of arterial involvement in Takayasu arteritis
reveals symmetric extension of the lesions in paired arterial beds. Arthritis Rheum 2011;63:1136-40.

Question
23
of 60
A 72-year-old man presents to the emergency room with progressive dyspnea, now present at rest. He has
had two prior myocardial infractions (MIs) and underwent coronary artery bypass grafting 12 years ago. He is
on simvastatin, a nitroglycerin patch, and atenolol. On examination, his blood pressure is 145/90 mm Hg, his
BOARD Page 22

on simvastatin, a nitroglycerin patch, and atenolol. On examination, his blood pressure is 145/90 mm Hg, his
pulse is 98 bpm, and his saturation is 93% on room air. His neck veins are distended. His lungs are clear.
Cardiac examination shows a regular S1 and S2, with a II/VI holosystolic murmur at the apex and a S3 gallop.
His chest X-ray shows cardiomegaly and vascular redistribution without focal infiltrates. His electrocardiogram
demonstrates sinus rhythm and a left bundle branch block pattern. A stat echocardiogram demonstrates a
dilated LV, with end-diastolic dimension of 7.2 cm, and an ejection fraction of 25% with moderate mitral
regurgitation.
With regard to the 72-year-old man described earlier, which of the following combinations of neurohormonal
abnormalities should be targeted in guidelines-based therapy of his heart failure?
A.Elevated vasopressin, angiotensin II, and aldosterone.
B.Elevated renin, BNP, and norepinephrine.
C.Elevated norepinephrine, angiotensin II, and aldosterone.
D.Elevated endothelin, vasopressin, and reduced nitric oxide.

The correct answer is C. Several classes of neurohormonal-blocking drugs reduce morbidity and mortality in
chronic systolic HF, while others have not been shown to be useful in clinical trials. Blockade of the adrenergic
and the RAAS improves symptoms and prolongs survival. Blockade of endothelin and vasopressin does not
improve outcomes in clinical trials. The effect of direct renin inhibitors in HF has not been tested in large-scale
clinical trials.

Question
24
of 60
A 43-year-old Caucaisan woman is referred to a heart failure clinic for evaluation and consideration of
advanced therapeutics (ventricular assist device, transplantation). She has a 6-year history of nonischemic
dilated cardiomyopathy, atrial fibrillation, and an implantable defibrillator was placed 4 years ago. She has had
progressive symptoms despite optimal medical therapy. In reviewing her medical records, several findings are
suggestive of a poor prognosis.
Which of the following conditions is associated with a worse prognosis in patients with heart failure?
A.Patient does not require loop diuretics.
B.Low cholesterol.
C.Obesity.
D.Hypouricemia.

BOARD Page 23

E.Resting sinus bradycardia.

The correct answer is B. Lower cholesterol has been linked to worse prognosis in heart failure patients and is
likely related to poor nutritional status (cardiac cachexia) and elevated circulating inflammatory cytokines. In
general, patients requiring higher doses of loop diuretics have worse outcomes, so the fact that the case
patient does not require loop diuretics portends a better prognosis. The obesity paradox suggests that
obese patients have better outcomes compared with their thin counterparts. Hyperuricemia has been
associated with worse outcomes, as has resting sinus tachycardia.

Question
25
of 60
A 50-year-old man is referred to your clinic for evaluation of exertional dyspnea. Over the past 3 months he
has noted increasing shortness of breath during daily activities, such as walking short distances or carrying
light bundles. He has no prior history of MI and denies chest pain. His past medical history is notable only for
hypertension and hypercholesterolemia. He takes only atenolol 25 mg once daily and simvastatin 20 mg once
daily. He drinks one glass of wine with dinner each night, and denies use of tobacco or illicit drugs. His family
history is notable for a paternal uncle who died suddenly in his 50s.
His exam is notable for blood pressure 170/80 mm Hg, pulse 80 bpm, mild jugular venous distention at 12 cm
H20, clear lungs to auscultation, and mild peripheral edema. S1 is normal, S2 is paradoxically split, and there is
a soft S3 gallop with a grade 2/6 holosystolic murmur at the apex. His ECG reveals sinus rhythm, with left
bundle branch block and a QRS duration of 150 ms. Echocardiogram reveals a dilated LV with eccentric
hypertrophy, anterior hypokinesis, and EF 25%.
He receives appropriate medical therapy with diuretics, an angiotensin-converting enzyme inhibitor, and a
beta-blocker.
Which of the following is the most reasonable next step in the diagnostic evaluation of this patient?
A.Endomyocardial biopsy.
B.Genetic testing.
C.Measurement of BNP.
D.Coronary angiography.
E.Cardiac MRI.

The correct answer is D. In this gentleman with cardiac risk factors and regional wall motion abnormality on
echocardiography, exclusion of coronary heart disease is the most important next step. Endomyocardial
biopsy is a consideration, but is likely low yield in the setting of an already dilated ventricle and a subacute HF
BOARD Page 24

biopsy is a consideration, but is likely low yield in the setting of an already dilated ventricle and a subacute HF
presentation. Although the patient may have a familial cardiomyopathy, genetic testing is premature and the
history is not characteristic of any stereotypical genetic syndromes (such as cardiolaminopathy).
Cardiac MRI might help to exclude infiltrative or inflammatory processes affecting the myocardium or to
quantitate the burden of scar, but is likely most useful after definitive evaluation of the coronary arteries is
completed. Invasive evaluation of ischemia is preferred when the pretest probability for coronary artery
disease is high. Measurement of BNP can be helpful in the initial diagnosis of HF and for prognostication, but
has little diagnostic role in this patient for whom HF is established based on the physical examination.
References
1. Lindenfeld J, Albert NM, Boehmer JP, et al. HFSA 2010 Comprehensive Heart Failure Practice Guideline. J
Cardiac Fail 2010;16:e1-194.

Question
26
of 60
A 58-year-old women with HCM and a maximal wall thickness of 22 mm is found to have an exercise-induced
LV outflow tract gradient of 90 mm Hg (normal blood pressure response to exercise) and moderate mitral
regurgitation with exertional dyspnea, with symptoms consistent with early NYHA class II symptoms. She has a
normal 24-hour Holter monitor, and no family history of sudden death.
Which of the following would you recommend next?
A.Surgical myectomy.
B.ASA.
C.Beta-blockers.
D.ICD.

The correct answer is C. ICD therapy is reserved only for HCM patients at increased risk for sudden death. LV
outflow tract obstruction is not a primary risk factor, and this patient also has none of the five traditional
primary risk factors for sudden death. Therefore, she would likely not benefit from this treatment. Invasive
septal reduction therapy with either surgical myectomy or ASA is indicated only in patients with advanced
heart failure symptoms consistent with NYHA class III or IV, who have failed medical therapy. This patient
should be initiated on beta-blockers for treatment of heart failure symptoms in the setting of a high
provocable outflow tract gradient.

Question
27

BOARD Page 25

of 60
A 56-year-old man status post anterior myocardial infarction with an LVEF 25% and mild right ventricular
dysfunction presents urgently to the clinic with complaints of orthopnea, paroxysmal nocturnal dyspnea, and
abdominal bloating for 2 days. He recently traveled to visit family for the holidays, and admitted to having
difficulty maintaining sodium and fluid restriction, although he was compliant with his medications. Upon his
return home, he noted that his weight had increased 10 lbs and his legs were swollen. His outpatient medical
regimen includes enalapril 10 mg twice daily, carvedilol 12.5 mg twice daily, spironolactone 25 mg once daily,
furosemide 80 mg twice daily, atorvastatin 40 mg once daily, and aspirin 81 mg once daily. He does not smoke
cigarettes or drink alcohol.
His physical examination reveals a blood pressure of 130/80 mm Hg, heart rate 88 bpm, jugular venous
pressure of 14 cm of water, decreased breath sounds at the left base, and a soft holosystolic murmur at the
left lower sternal border. The liver edge is palpable 3 fingerbreadths below the right costal margin, and the
lower extremities are warm with 1+ pitting edema to the mid-calves bilaterally.
You decide to directly admit this patient to the inpatient cardiology service for management of ADHF.
According to the ACCF/AHA and HFSA guidelines, in addition to a 2-liter fluid restriction, which of the following
would be the safest and most effective initial order for fluid management?
A.Furosemide 40 mg IV bolus every 12 hours.
B.Furosemide 80 mg IV bolus every 12 hours.
C.Bumetanide 2 mg orally every 12 hours.
D.Bedside venovenous ultrafiltration at 100 ml/hr.
E.Bedside venovenous ultrafiltration at 200 ml/hr.

The correct answer is B. This patient presents urgently with symptoms and signs of congestion and normal
systemic perfusion. The likely precipitant to this admission is nonadherence with sodium and fluid restriction,
which can often occur during travel away from home. When patients are admitted to the hospital with ADHF,
initiating an effective diuresis with IV loop diuretics is critical to lowering cardiac filling pressures and reliving
congestive symptoms. According to the ACCF/AHA 2009 Focused Update of the 2005 Guidelines for the
Diagnosis and Management of Heart Failure in Adults, if patients are already receiving loop diuretics, the initial
IV dose should equal or exceed the chronic oral dose (option B, not A).1
Bumetanide is a more potent loop diuretic than furosemide, but 2 mg twice a day is equivalent to furosemide
80 mg twice a day (option C) and should be given intravenously, per guidelines. The recent DOSE-AHF study
showed no benefit of a continuous infusion (option C),2 and while pilot studies suggest that early ultrafiltration
(options D and E) is associated with greater fluid removal than medical therapy alone,3 the HFSA guidelines
raise concerns about safety, cost, and need for specialized medical and nursing support.4
References
1. Hunt SA, Abraham WT, Chin MH, et al. 2009 focused update incorporated into the ACC/AHA 2005
Guidelines for the Diagnosis and Management of Heart Failure in Adults: a report of the American
College of Cardiology Foundation/American Heart Association Task Force on Practice Guidelines:
developed in collaboration with the International Society for Heart and Lung Transplantation. J Am Coll
Cardiol 2009;53:e1-e90.
2. Felker GM, Lee KL, Bull DA, et al. Diuretic strategies in patients with acute decompensated heart failure.
BOARD Page 26

2. Felker GM, Lee KL, Bull DA, et al. Diuretic strategies in patients with acute decompensated heart failure.
N Engl J Med 2011;364:797-805.
3. Costanzo MR, Guglin ME, Saltzberg MT, et al. Ultrafiltration versus intravenous diuretics for patients
hospitalized for acute decompensated heart failure. J Am Coll Cardiol 2007;49:675-83.
4. Lindenfeld J, Albert NM, Boehmer JP, et al. HFSA 2010 comprehensive heart failure practice guideline. J
Card Fail 2010;16:e1-194.

Question
28
of 60
A 60-year-old woman presents to the emergency room with NYHA class IV dyspnea. She has been admitted
with HF three times in the past year. She had a large anterior myocardial infarction 2 years ago and despite
coronary stenting of her left anterior descending artery, she was left with an EF of 10%, end-diastolic
dimension of 8 cm, and moderately severe mitral regurgitation. Her medications include carvedilol, lisinopril,
spironolactone, furosemide, aspirin, and simvastatin.
On exam, she is mildly tachypneic with a heart rate of 90 bpm and a blood pressure of 100/70 mm Hg. Her
venous pressure is 16 cm H20. The lungs are clear. The heart has audible and palpable gallop; murmur of
mitral regurgitation is present. The liver edge is palpable. The extremities are warm; there is pedal edema. An
ECG demonstrates sinus rhythm with an anterior myocardial infarction pattern and a narrow QRS.
A cardiopulmonary exercise test demonstrates an oxygen consumption of 9.0 cc/kg/min with a respiratory
quotient of 1.15, a ventilatory efficiency of 25, and a peak blood pressure of 130 mm Hg.
Which of the following is the primary indication for transplantation?
A.End-diastolic volume of 8 cm.
B.EF of 10%.
C.Ventilatory efficiency of 25.
D.Oxygen consumption of 9.0 cc/kg/min.
E.NYHA IV symptoms.

The correct answer is D. Although the increased heart size, low EF, and advanced symptoms predict increased
mortality, cardiopulmonary exercise capacity is the most potent predictor of prognosis. The ventilatory
efficiency of CO2 removal is characterized by the Ve/VC02 ratio and values over 35 predict worse prognosis.
NYHA class is dynamic and waxes and wanes over time, and should be assessed in the chronic ambulatory
state.

Question
BOARD Page 27

Question
29
of 60
A 76-year-old African-American female with nonischemic cardiomyopathy, history of breast carcinoma
status/post mastectomy, and radiation therapy 30 years prior, as well as type 2 diabetes complicated by
chronic kidney disease (creatinine clearance of 22), is hospitalized with worsening creatinine, decreased
appetite/failure to thrive, cool lower extremities, and confusion by family report. Her ECG showed a left
bundle branch block. A Swan-Ganz catheterization is performed, which reveals a right atrial pressure of 24 mm
Hg, pulmonary capillary wedge pressure (PCWP) of 34 mm Hg, and a cardiac index of 1.4 L/min. Milrinone is
instituted with a decrease of PCWP to 22 mm Hg and an improvement in cardiac index to 1.9 L/m. Mentation
improves, but creatinine does not change. Within 24 hours of weaning the inotrope, the patient is noted to be
confused and her creatinine increased further.
Which of the following would be the next step for this patient?
A.Implantation of a CRT.
B.Continuous infusion of inotrope.
C.Initiation of oral inotrope.
D.Use of intermittent IV inotrope.
E.Initiation of dialysis.

The correct answer is B. The decision to use chronic infusions of inotropic therapy is clinical and must take
into account patient preferences and cost. The major reason to initiate therapy is palliation. Mortality is very
high in this patient cohort. Implantation of a CRT has not been rigorously studied in inotrope-dependent
patients, although case series have been presented. There are no oral inotropes currently approved for use.
Use of intermittent IV inotropic agents is a Class III recommendation; there are no double blinded clinical trials
to support the practice. Dialysis should only be considered under extenuating circumstances during terminal
care; dialysis will not change the natural history of heart failure in this setting.
References
1. Allen L, Stevenson LW, et al. AHA Position Paper: Advanced Heart Failure: Prognosis, Communication,
and Decision Making. Circulation [submission anticipated July 2011].
2. Hauptman PJ, Mikolajczak P, George A, et al. Chronic inotropic therapy in end-stage heart failure. Am
Heart J 2006;152:1096.e1-8.

Question
30
of 60
In which of the following clinical scenarios should an EMB be performed?
BOARD Page 28

In which of the following clinical scenarios should an EMB be performed?


A.A patient who presents to the emergency department with crushing chest pain, orthopnea, leg edema, and
an ECG showing a QRS duration of 150 msec.
B.A 20-year-old woman with acute heart failure, an EF of 25%, and third-degree AV block.
C.A 35-year-old man who complains of acute pleuritic chest pain and a normal EF on echocardiogram.
D.A 65-year-man with atypical chest pain and delayed gadolinium enhancement in the lateral walls.

The correct answer is B. Patient A has coronary disease until proven otherwise. Patient C has no evidence of
myocardial dysfunction. Patient D has evidence of coronary disease. EMB is a class I recommendation in
patients who develop new-onset heart failure and heart block. In this situation, infiltrative disorders including
sarcoid need to be considered, and the diagnosis is best established by biopsy.
References
1. Cooper LT, Baughman KL, Feldman AM, et al. The Role of Endomyocardial Biopsy in the Management of
Cardiovascular Disease: A Scientific Statement From the American Heart Association, the American
College of Cardiology, and the European Society of Cardiology Endorsed by the Heart Failure Society of
America and the Heart Failure Association of the European Society of Cardiology. J Am Coll Cardiol
2007;50:1914-31.

Question
31
of 60
A 50-year-old man with a history of paroxysmal atrial fibrillation and coronary artery disease is to be initiated
on dofetilide. His medications include aspirin 81 mg daily, lisinopril 10 mg daily, simvastatin 40 mg daily,
metoprolol 50 mg twice daily, and isosorbide dinitrate 20 mg three times daily. His heart rate is 55 bpm, blood
pressure is 123/75 mm Hg, and the remainder of the examination is unremarkable. His ECG shows sinus
bradycardia at 54 bpm with a nonspecific intraventricular conduction delay, and a QRS duration of 105 ms and
QT interval of 430 ms. Laboratory panel shows normal renal function, but with a slightly increased potassium
level of 5.6 mEq/L.
Which of the following effects of dofetilide may increase the risk for torsade de pointes in this patient?
A.Reverse use dependence that is exacerbated by bradycardia.
B.Use dependence that is exacerbated by bradycardia.
C.Decreased potassium efflux exacerbated by hyperkalemia.
D.Increased potassium efflux exacerbated by hyperkalemia.
E.Decreased drug metabolism in the presence of simvastatin.
BOARD Page 29

E.Decreased drug metabolism in the presence of simvastatin.

The correct answer is A. Dofetilide exhibits reverse use dependence, with an increased effect at slow heart
rates. Bradycardia in this patient may exacerbate dofetilides QT prolonging effects. Dofetilide does not have a
drug interaction with simvastatin.
References
1. Darbar D. Standard antiarrhythmic drugs. In: Zipes DP, Jalife J. Cardiac Electrophysiology: From Cell to
Bedside. 5th ed. Philadelphia: Saunders; 2009: 970-1.

Question
32
of 60
A 65-year-old man is admitted to the intensive care unit with recurrent episodes of VT that have been ongoing
for 12 hours. His past history includes CAD. Five days ago, he had an acute ST elevation inferior wall
myocardial infarction with a drug-eluting stent in the right coronary and was discharged home 2 days ago.
Four months ago, he had an anterior wall myocrdial infarction with a drug-eluting stent in the proximal left
anterior descending artery. His LVEF is 15% by echocardiogram, performed 3 days ago. His other past history
includes diabetes type 2, hyperlipidemia, chronic kidney disease stage 3, and emphysema. His current
medications are aspirin 81 mg/day, clopidogrel 75 mg/day, lisinopril 2.5 mg/day, carvedilol 3.125 mg twice
daily, rosuvastatin 20 mg/day, and insulin glargine 15 U/day.
His vital signs are blood pressure 110/60 mm Hg with a heart rate of 135 bpm, respirations of 14
breaths/minute with a pulse oximetry of 93% on 2 liters of oxygen by nasal cannula. On physical examination,
the jugular venous pulse is seen at about 9 cm H2O, and there are bibasilar crackles on examination. His ECG
confirms VT at a rate of 135 bpm, and telemetry shows VT to break periodically to sinus rhythm for about 1
minute before reinitiating back to VT. Six hours ago in the emergency room, he was initiated on amiodarone
with a 150 mg IV bolus and a drip of 1 mg/min, and a second 150 mg IV bolus was given about 1 hour ago. He
remains predominantly in VT. His lab work is notable for a potassium of 3.8 mEq/L and a creatinine of 2.2
mg/dl.
In considering whether to add lidocaine, which of the following clinical features, in addition to possible
interaction with amiodarone, places this patient at increased risk for lidocaine toxicity?
A.Systolic heart failure.
B.Renal disease.
C.Low serum potassium.
D.Recent myocardial infarction.
E.Emphysema.

The correct answer is A. Lidocaine is metabolized in the liver, and features that can impair hepatic blood flow,
BOARD Page 30

The correct answer is A. Lidocaine is metabolized in the liver, and features that can impair hepatic blood flow,
particularly heart failure, can increase the risk of toxicity. In the setting of heart failure, lidocaine should be
used cautiously, and the dose should be reduced by 50%. Lidocaine, a class IB agent, can be used in the setting
of a recent myocardial infarction, but class IC agents are contraindicated.
References
1. Woosley RL. Pharmacokinetics and pharmacodynamics of antiarrhythmic agents in patients with
congestive heart failure. Am Heart J 1987;114:1280-91.

Question
33
of 60
A 25-year-old woman presents with complaints of palpitations, exercise intolerance, fatigue, near syncope,
and syncope. She was completely well until 6 months ago, when she contracted a severe flu-like illness, after
which her symptoms began and have continued since.
Her 12-lead electrocardiogram and echocardiogram are normal, as are her thyroid profile and complete blood
count. Findings from her physical examination are normal, except that when going from sitting to standing her
blood pressure decreases from 110/70 mm Hg to 100/60 mm Hg and her pulse goes from 80 bpm while sitting
to 120 bpm while standing.
Which of the following conditions is the most likely diagnosis?
A.Panic attacks.
B.Postural tachycardia syndrome.
C.Neurocardiogenic syncope.
D.Occult hypothyroidism.
E.Multiple system atrophy.

The correct answer is B. The history and hemodynamics are consistent with postural tachycardia syndrome,
demonstrating an increase in heart rate when going from sitting to standing.1
References
1. Grubb BP, Kanjwal Y, Kosinski DJ. The postural tachycardia syndrome: a concise guide to diagnosis and
management. J Cardiovasc Electrophysiol 2006;17:108-12.

Question
34
BOARD Page 31

34
of 60
A 64-year-old man is referred to the clinic for evaluation of an abnormal ECG (Figure 1). His past medical
history is significant for hypertension, for which he is treated with lisinopril. He denies cardiac
symptomatology. He is active, works full-time as a car salesman, and plays golf regularly.

Which of the following is the next best diagnostic step for this patient?
A.Echocardiogram.
B.Carotid sinus massage.
C.Blood cultures.
D.Pacemaker.
E.Cardiac magnetic resonance imaging (MRI).

The correct answer is B. The ECG is consistent with sinus rhythm with 2:1 AV block. The differential is Mobitz
type I (Wenckebach) second-degree AV block versus Mobitz type II second-degree AV block. Noninvasive vagal
and sympathetic maneuvers can help to distinguish one from the other, since Mobitz I block is usually AV
nodal, and Mobitz II block is usually in the His-Purkinje system. In a patient with AV nodal block, carotid sinus
massage will slow the sinus rate and worsen AV conduction, which will worsen the AV block. The slowing of
sinus rate and depression of AV conduction will have a protective effect on the distal conduction system, so
conduction in the His-Purkinje may improve. Exercise (sympathetic stimulation) will improve AV nodal
conduction, and worsen His-Purkinje conduction. His ECG (Figure 1) is suggestive of Mobitz I second-degree
block, based on the long PR interval and narrow QRS interval.

BOARD Page 32

Option A is not correct: Although the detection of structural heart disease may help guide patient therapy, it is
not the next best diagnostic maneuver to assess the degree and level of AV block.
Option C is not correct: There is no clinical evidence of endocarditis.
Option D is not correct. If the site of block is within the AV node in an asymptomatic patient, a pacemaker is
not indicated.
Option E is not correct. Cardiac MRI is not indicated.

Question
35
of 60
A 67-year-old obese male patient with diabetes mellitus and obstructive sleep apnea presents with profound
dyspnea and a blood pressure of 69/40 mm Hg. The ECG shows evidence of a narrow complex tachycardia at
210 bpm, and a short RP interval.
Which of the following is the most appropriate initial therapy?
A.Adenosine 12 mg IV stat.
B.Diltiazem 20 mg IV stat.
C.Electrical cardioversion.
D.Metoprolol 5 mg IV stat.

The correct answer is C. Electrical cardioversion should always be utilized as first-line therapy in a
hemodynamically unstable patient.1
References
1. Hazinski MF, Nolan JP, Billi JE, et al. Part 1: Executive summary: 2010 International Consensus on
Cardiopulmonary Resuscitation and Emergency Cardiovascular Care Science With Treatment
Recommendations. Circulation 2010;122:S250-75.

Question
36
of 60
A 78-year-old male presents to your office for routine yearly evaluation. He feels well and has no current
symptoms or limitations. He has a past history of coronary artery disease and coronary artery bypass graft
BOARD Page 33

symptoms or limitations. He has a past history of coronary artery disease and coronary artery bypass graft
surgery 7 years ago, chronic obstructive pulmonary disease, diabetes mellitus type 2 (diet controlled), and
hypertension. You obtain a 12-lead ECG (Figure 1).

His examination reveals: well-nourished male in no distress. Blood pressure is 124/74 mm Hg. Pulse is mostly
regular (rate 70 bpm). No jugular venous pressure elevation and a soft carotid bruit. Well healed median
sternotomy scar. Normal heart sounds, no murmurs, and clear lung fields.
Current medications are: atenolol 50 mg daily, lisinopril 20 mg daily, and aspirin 325 mg daily.
At this time, you advise which of the following?
A.No change in medications or therapy.
B.Addition of clopidogrel to aspirin therapy.
C.Long-term warfarin therapy.
D.Cardioversion with anticoagulation coverage.
E.Catheter ablation.

The correct answer is C. Long-term warfarin therapy is appropriate, as he is at increased risk of stroke
(CHADS2 score = 3). The risk of stroke in patients with AFL is the same as that associated with AF.

BOARD Page 34

Question
37
of 60
A 52-year-old female is seen in a cardiology clinic with complaints of increasing episodes of palpitations.- She
has been diagnosed with paroxysmal AF and experiences episodes about once every month.- Initially they
lasted about a half hour, but now they persist for almost an entire day. During the episodes she feels
palpitations. As the event persists, she develops fatigue and has to stop her daily activities and rest.
The patients other medical history is significant for hypertension and gastroesophageal reflux disease. Her
current medications are aspirin 81 mg daily, hydrochlorothiazide 25 mg daily, metoprolol 25 mg twice a day,
and omeprazole 20 mg daily.
On exam today, her blood pressure is 127/74 mm Hg, heart rate is 51 bpm, and the rest of her exam is within
normal limits. Labs are unchanged from the previous visit and show sodium 139, potassium 4.1, magnesium
2.2, BUN 23, creatinine 1.5, and INR 1.1.
A treadmill stress echocardiogram earlier this year shows normal biventricular function, no evidence of LVH,
normal chamber sizes, and normal response with exercise with no evidence of ischemia. A 12-lead ECG is
shown (Figure 1).

Which of the following is the next best pharmacologic option?


A.Pill in the pocket approach with propafenone.
B.Amiodarone.
BOARD Page 35

B.Amiodarone.
C.Dofetilide.
D.Sotalol.
E.Increase daily metoprolol.

The correct answer is A. This is an ideal patient for a pill in the pocket approach, who can self-administer
her antiarrhythmic during an episode of AF.1 She seems to tolerate the arrhythmia for a short period before
becoming increasingly symptomatic. Note that the ECG shows sinus rhythm with a heart rate of 58 bpm and a
prolonged QTc interval (Figure 1).
Rate control will likely be insufficient in controlling her symptoms, particularly with her already low baseline
heart rate. Amiodarone would not be optimal in such a young patient. Sotalol should be avoided with renal
dysfunction, and dofetilide should not be initiated with QTc intervals >440 ms or used concurrently with
hydrochlorothiazide.2 Propafenone would be a good option for this patient.
References
1. Alboni P, Botto GL, Baldi N, et al. Outpatient treatment of recent-onset atrial fibrillation with the "pill-inthe-pocket" approach. N Engl J Med 2004;351:2384-91.
2. Fuster V, Rydn LE, Cannom DS, et al. 2011 ACCF/AHA/HRS focused updates incorporated into the
ACC/AHA/ESC 2006 Guidelines for the Management of Patients with Atrial Fibrillation: a report of the
American College of Cardiology Foundation/American Heart Association Task Force on Practice
Guidelines developed in partnership with the European Society of Cardiology and in collaboration with
the European Heart Rhythm Association and the Heart Rhythm Society. J Am Coll Cardiol
2011;57:e101-98.

Question
38
of 60
A 14-year-old boy, diagnosed with CPVT at age 8 and treated with nadolol 2.5 mg/kg/day, comes to a followup visit. During an exercise stress test, two runs of nonsustained VT (5 beats) occur after 10 minutes (maximal
tolerated with Bruce protocol).
Which of the following drugs could be considered on top of beta-blockers?
A.No further treatment is required.
B.Quinidine.
C.Verapamil.
D.Flecainide.
BOARD Page 36

D.Flecainide.
E.Amiodarone.

The correct answer is D. The evidence of nonsustained VT during exercise suggests poor protection from
adrenergically-induced arrhythmias in CPVT. The first approach is to increase beta-blocker dose. The standard
dose is 1-1.5 mg/kg/day of nadolol. In this case, the dosage is already at the usual tolerated level. Recent
evidence suggests that flecainide might achieve an additional antiarrhythmic effect.1 Since the arrhythmic
pattern in CPVT is often reproducible, the use of oral flecainide may be considered. The patient should be
reassessed after 2-3 weeks with an exercise stress test.
References
1. van der Werf C, Kannankeril PJ, Sacher F, et al. Flecainide therapy reduces exercise-induced ventricular
arrhythmias in patients with catecholaminergic polymorphic ventricular tachycardia. J Am Coll Cardiol
2011;57:2244-54.

Question
39
of 60
A 34-year-old male presents to the emergency room with palpitations and lightheadedness. The ECG shown in
Figure 1 is obtained. Sinus rhythm is restored with administration of intravenous lidocaine, and the ECG shown
in Figure 2 is obtained. Coronary angiography shows normal coronaries and anLV ejection fraction of 45% with
global hypokinesis.

BOARD Page 37

Cardiac MRI with assessment of gadolinium is most likely to reveal which of the following?
A.No areas of delayed enhancement.
B.Fat infiltration of the RV.
C.Delayed enhancement of the anterior wall of the LV.
D.Severe ventricular hypertrophy.

The correct answer is C. Figure 1 shows a wide QRS tachycardia at approximately 150 bpm. VT is indicated by
AV dissociation (well seen in V1),with lead aVR showing an initial broad Q wave. Following conversion
(Figure 2), the ECG shows sinus rhythm with first-degree AV block and RBBB with left-axis deviation. The
patient's relatively young age raises the question of idiopathic VT, in which case no abnormalities would be
expected on MRI. The abnormal sinus rhythm ECG, however, suggests underlying disease.
Most monomorphic VTs are due to scar that defines the origin of the VT. The VT has anRBBB-like configuration
in V1 consistent with an origin in the LV. The inferior axis suggests origin in the superior portion of the
ventricle. Thus, MRI is likely to show delayed enhancement of the anterior wall of the LV. ARVC, which often is
associated with fatty infiltration of the RV on MRI, can have a similar presentation with VT, but AV block is
rare, and VT is more likely to originate in the RV and have anLBBB configuration.
It is now also recognized that intramyocardial fat on MRI can be a normal variant. Monomorphic VT can occur
in hypertrophic cardiomyopathy, but is uncommon, and the sinus rhythm ECG does not suggest LV
hypertrophy. This patient's VT and scar were due to cardiac sarcoidosis.
References
1. Aliot EM, Stevenson WG, Almendral-Garrote JM, et al. EHRA/HRS Expert Consensus on Catheter Ablation
of Ventricular Arrhythmias: developed in a partnership with the European Heart Rhythm Association
BOARD Page 38

of Ventricular Arrhythmias: developed in a partnership with the European Heart Rhythm Association
(EHRA), a Registered Branch of the European Society of Cardiology (ESC), and the Heart Rhythm Society
(HRS); in collaboration with the American College of Cardiology (ACC) and the American Heart
Association (AHA). Heart Rhythm 2009;6:886-933.
2. Zipes DP, Camm AJ, Borggrefe M, et al. ACC/AHA/ESC 2006 guidelines for management of patients with
ventricular arrhythmias and the prevention of sudden cardiac death: a report of the American College of
Cardiology/American Heart Association Task Force and the European Society of Cardiology Committee
for Practice Guidelines (Writing Committee to Develop Guidelines for Management of Patients With
Ventricular Arrhythmias and the Prevention of Sudden Cardiac Death). J Am CollCardiol.
2006;48:247-346.
3. Marcus FI, McKenna WJ, Sherrill D, et al. Diagnosis of arrhythmogenic right ventricular
cardiomyopathy/dysplasia: proposed modification of the task force criteria. Circulation
2010;121:1533-41.
4. Muthappan P, Calkins H. Arrhythmogenic right ventricular dysplasia. ProgCardiovasc Dis 2008;51:31-43.

Question
40
of 60
A 71-year-old woman with an ischemic cardiomyopathy and atrial fibrillation underwent biventricular ICD
implantation 6 months prior. She has NYHA class III symptoms and continues to complain of very limited
functional capacity, with no change since resynchronization therapy was initiated.
She is currently taking carvedilol 25 mg BID, lisinopril 10 mg QD, furosemide 40 mg BID, simvastatin 20 mg QD,
aspirin 325 mg QD, and warfarin 5 mg QD. Her vital signs in the office today are blood pressure 90/60 mm Hg,
heart rate 111 bpm, respiration rate 20, oxygen saturation (SaO2) 94% room air, and she is afebrile. Her exam
is significant for bibasilar rales, irregularly irregular heartbeat, and 1+ bilateral lower extremity edema.
Interrogation of her biventricular ICD reveals good pacing and sensing thresholds. The battery status and lead
impedances are adequate. There have been two episodes of nonsustained ventricular tachycardia (VT), up to
15 beats, but no shocks have been required. She is 25% paced.
Which of the following should be the next step in her management?
A.Antiarrhythmic drug treatment for VT with amiodarone.
B.Increase carvedilol dose.
C.AV nodal ablation.
D.Referral for heart transplant evaluation.
E.Decrease carvedilol dose.

BOARD Page 39

The correct answer is C. Recent data support this approach for management of AF patients with CHF.
Asymptomatic nonsustained VT is not an indication for amiodarone therapy, and studies have shown that
antiarrhythmic drug agents increase mortality. Her blood pressure is tenuous, so increasing her beta-blocker is
risky. Because she is asymptomatic of her borderline blood pressure and not in decompenstated HF, there is
no reason to decrease her beta-blocker. She is not class IV, and therefore, would not be an appropriate
candidate for a heart transplant evaluation.

Question
41
of 60
The patient is a 58-year-old male with witnessed sudden collapse in Chicago OHare airport. A bystander
performed CPR and an automated external defibrillator (AED) was obtained and applied. EMS was
simultaneously alarmed. The AED detected a shockable rhythm and a single shock was performed. The AED
detected a nonshockable rhythm and additional chest compressions were begun. EMS arrived and detected a
pulse. Chest compressions were halted and a pulse of 110 with a blood pressure (BP) of 150/90 was
documented. However, the patient remained unresponsive. He is transported to the nearest hospital. The ECG
in the ED shows only sinus tachycardia.
Which of the following is the optimal post-resuscitation care?
A.Begin therapeutic hypothermia in the ED with rapid iced-saline infusion (1-2 liters), admit to ICU and
continue therapeutic hypothermia with a commercial mechanical surface cooling system to a goal of 33C for
24 hours. Rewarm slowly (0.3-0.5C/h) after 24 hours.
B.Provide comfort care and prepare the family for an inevitable poor outcome (i.e., either death or significant
and incapacitating anoxic brain injury).
C.Call the catheterization laboratory for emergency coronary angiography and potential PCI. Afterwards admit
the patient to the ICU for consideration of therapeutic hypothermia induction.
D.Begin therapeutic hypothermia in the ED while simultaneously calling the catheterization laboratory for
emergency coronary angiography. Attempt PCI for any acute coronary occlusion or unstable, high grade
presumed culprit lesion thought responsible for the cardiac arrest. Continue hypothermia in the
catheterization laboratory and later in the ICU, to a goal of 33C for 24 hours. Rewarm slowly (0.3-0.5 C/h)
after 24 hours.

The correct answer is D. This patient is ideal for aggressive post-resuscitation care. He had a witnessed VFCA,
received immediate bystander CPR, and received early defibrillation with an AED. His time to ROSC was short
and he was hemodynamically stable upon arrival at the ED. He remained comatose upon presentation and
hence was a good candidate for therapeutic hypothermia to preserve central nervous system function and
improve his chance for meaningful long-term survival. Such hypothermia should be begun immediately upon
his arrival and continued for 24 hours, with a goal of 33C.
His etiology of circulatory collapse is almost certainly cardiac, as manifested by its sudden, unexpected
occurrence. He deserves a careful search for an acute coronary ischemic event as the precipitating cause of
BOARD Page 40

occurrence. He deserves a careful search for an acute coronary ischemic event as the precipitating cause of
this cardiac arrest. This is best accomplished by emergency coronary angiography even though his postresuscitation ECG does not show ST elevation. One in four of such patients are identified as having an acutely
occluded or culprit coronary lesion that is responsible for their cardiac arrest, which can be treated with
emergent coronary angiography and PCI.

Question
42
of 60
A 75-year-old woman is hospitalized for shortness of breath with chest pain. A pulmonary embolism is
diagnosed and she is initiated on heparin. Several hours later, she becomes hypotensive and does not respond
to a fluid bolus. One hour later, she collapses on the way to the bathroom and a code arrest is called. The
monitor shows a slow, wide, complex rhythm at 35 bpm, without a pulse, and chest compressions are started.
External pacing through the defibrillator patches is attempted, but there is no ventricular capture. Chest
compressions have been ongoing for 4 minutes.
Which of the following is the next step that should be taken?
A.Administration of atropine 1 mg IV.
B.Placement of a transvenous temporary pacing wire.
C.Endotracheal intubation.
D.Administration of epinephrine 1 mg IV.
E.Thrombolytic therapy.

The correct answer is D. In PEA, the first administered drug is epinephrine. Vasopressin can be substituted for
the first or second dose of epinephrine. A transvenous pacing wire would require undue interruptions in chest
compressions, and should not be attempted prior to administration of a vasopressor agent, epinephrine.
Atropine can be considered after epinephrine is given, but is unlikely to be of benefit.
References
1. Neumar RW, Otto CW, Link MS, et al. Part 8: adult advanced cardiovascular life support: 2010 American
Heart Association Guidelines for Cardiopulmonary Resuscitation and Emergency Cardiovascular Care.
Circulation 2010;122:S729-67.

Question
43

BOARD Page 41

of 60
The aortic and peripheral vasculature in elderly patients differs from that of younger patients. These
differences can significantly affect the pressure in the periphery and can affect the data derived from direct
measurements of the gradient in AS.
Which of the following is the correct answer regarding the effect of aging on the hemodynamics observed in
elderly patients compared to young patients?
A.In general, the central aortic pressure is higher than the femoral artery pressure in younger patients, and it
is just the opposite in elderly patients.
B.The capillary bed microvessels are more likely to have pulsatile flow in younger patients than in elderly
patients.
C.There is a greater disparity between the central aortic pressure and the femoral artery pressure in a younger
patient than in an elderly patient.
D.When using the LV pressure and the femoral artery sheath pressure to measure the gradient across the
aortic valve, the gradient in the young patient will be greater than the gradient in the elderly patient for the
same severity of aortic stenosis.

The correct answer is C. Aging results in a progressive decline in collagen and elastin in the arterial walls, and
an increase in vascular rigidity. With age, the pulse pressure becomes greater.
Aging also results in arterial dilatation and, while the conduit function of the arterial system is generally not
affected, the cushioning function of the arterial tree is greatly affected. The pulse pressure is wider when the
arterial compliance is poor and there is loss of the cushioning effect in the central aorta.
The practical consequence is that the central aortic pressure is generally significantly lower than the femoral
artery pressure in young patients, but less so in elderly patients. When one measures the gradient using the
femoral artery sheath and the LV, the gradient will appear to be lower in the young patient than if the LV and
central aortic pressure was used. These differences are less marked in the elderly. The stiff aortic vessels also
result in the greater transmission of the pulsatile flow into the microvessels in the elderly patient compared to
the younger patient.

Question
44
of 60
Echocardiography in a 72-year-old man with a systolic murmur shows the following findings:

BOARD Page 42

Aortic maximum velocity 3.8 m/s.


Mean transaortic gradient 35 mm Hg.
Continuity equation AVA 1.1 cm2.
LVEF 40%.
These findings are most consistent with which of the following?
A.Aortic valve sclerosis.
B.Mild AS.
C.Moderate AS.
D.Severe AS.

The correct answer is C. These findings are consistent with moderate AS. The aortic velocity is 3-4 m/s, the
mean gradient is 20-40 mm Hg, and the valve area is 1.0-1.5 cm2 (Table 2). Although the EF is low, when lowoutput, low-gradient AS is present, the calculated valve areas is <1.0 cm2 with a relatively low gradient. In this
situation, the dilemma is distinguishing severe AS with a low transaortic stroke volume due to the high LV
afterload from only moderate AS with primary myocardial dysfunction. When severe AS is present, the aortic
velocity increases to >4 m/s during dobutamine stress echocardiography while the valve area remains <1.0
cm2. With moderate AS, valve area increases as stroke volume across the valve increases, resulting in only a
slight increase in aortic velocity and transaortic gradient.

References
1. Baumgartner H, Hung J, Bermejo J, et al. Echocardiographic assessment of valve stenosis: EAE/ASE
recommendations for clinical practice. Eur J Echocardiogr 2009;10:1-25.

BOARD Page 43

Question
45
of 60
Which of the following is the correct affirmation about tricuspid stenosis?
A.It is frequently an isolated rheumatic valve lesion.
B.The physical exam reveals pulmonary edema.
C.For symptomatic patients, medical therapy is the best option.
D.Tricuspid stenosis and regurgitation might be produced by pacemaker lead-related leaflet fibrosis.

The correct answer is D. Pacemaker lead-related leaflet fibrosis has been described as a cause of tricuspid
stenosis and regurgitation. Option A is incorrect because tricuspid stenosis is related to mitral stenosis most of
the time, and rarely ever an isolated lesion. Option B is incorrect because tricuspid stenosis presents with
pedal edema, hepatomegaly, and ascites, but not pulmonary congestion. Option C is incorrect because
medical therapy is not useful in most of the cases.

Question
46
of 60
A 72-year-old man with metastatic pancreatic cancer, hypertension, hyperlipidemia, and diabetes presents
with progressively worsening dyspnea on exertion. Echocardiography reveals severe AR with holodiastolic flow
reversal in the descending thoracic aorta as well as aortic root dilation of 5.5 cm. The calculated biplane
ejection fraction is 43%.
What is the next best step in the treatment of this patient?
A.Immediate surgical intervention.
B.Medical therapy with vasodilators.
C.Computed tomography of the chest.
D.Exercise stress testing.

BOARD Page 44

E.Angiography to evaluate for coronary artery disease.

The correct answer is B. Medical therapy in this patient is reasonable since he is not a surgical candidate. As
the patient has metastatic pancreatic cancer and a poor life expectancy, any surgical intervention is
contraindicated. Answers A, C, D, and E are thus incorrect.

Question
47
of 60
Which of the following echocardiographic measurements is consistent with severe TR?
A.Central TR jet with jet area >8 cm2 by color Doppler.
B.TR vena contracta width >7 mm.
C.TR PISA radius >0.6 cm.
D.Systolic dominance of hepatic vein flow.

The correct answer is B. An integrated approach is recommended for evaluation of TR severity, rather than a
singular measurement. This approach should include evaluation of the size of right-sided chambers and
various Doppler measurements. Doppler assessment should be performed in at least two orthogonal planes,
with evaluation of the jet area (for central TR jets), vena contracta width, PISA radius (at a Nyquist limit of 28
cm/sec), density and contour of the jet by continuous wave Doppler, and hepatic vein flow.
The American Society of Echocardiography has outlined measurements consistent with severe TR: dilated right
ventricle, atrium, and inferior vena cava in chronic TR; TR jet area >10 cm2; vena contracta width >7 mm; PISA
radius >0.9 cm; dense, triangular, continuous wave Doppler profile with early peaking; and systolic reversal of
hepatic vein flow.
References
1. Zoghbi WA, Enriquez-Sarano M, Foster E, et al. Recommendations for evaluation of the severity of
native valvular regurgitation with two-dimensional and Doppler echocardiography. J Am Soc
Echocardiogr 2003;16:777-802.

Question
48
of 60
An 18-year-old woman presents to the clinic with a history of a partial AV septal repair at age 12 months. She
BOARD Page 45

An 18-year-old woman presents to the clinic with a history of a partial AV septal repair at age 12 months. She
is asymptomatic from a cardiovascular standpoint. She is a competitive athlete and a senior in high school.
Which of the following are you most likely to find once you have examined her and reviewed her
echocardiogram?
A.Apical systolic murmur of mitral regurgitation.
B.Apical systolic and diastolic murmur.
C.Ejection systolic murmur at the base.
D.Loud pulmonic closure sound.

The correct answer is A. Patients with a partial AVSD almost always have some residual left AV valve or mitral
regurgitation. Thus, the most common murmur in such patients would be an apical systolic murmur. A
combined apical systolic and diastolic murmur might be expected in a patient with severe mitral regurgitation
and "relative mitral stenosis," or in a patient with mixed mitral stenosis and regurgitation, but not in a patient
doing well after repair of a partial AVSD. An ejection systolic murmur would not be expected in such a patient.
Nor would a loud pulmonic closure sound, which might reflect pulmonary arterial hypertension; it is not
expected in this patient. A widely split second heart sound could be due to an ASD that should not be present
if the surgery was successful.

Question
49
of 60
A 35-year-old man with TOF, which was surgically repaired in childhood, presents for evaluation of
palpitations. He is healthy and denies exercise intolerance. He is not on any medications. His physical
examination is significant for a blood pressure of 130/80 mm Hg, pulse is 75 bpm, and has normal oxygen
saturation. Cardiac examination has a well-healed median sternotomy scar. There is a normal S1 and
physiologically split S2 with a soft 2/6 systolic ejection murmur at the left sternal border and a 2/4 diastolic
murmur at the base of the heart. Lungs are clear. Pulses are equal and symmetric.
Which of the following findings is associated with an increased risk of adverse outcome in this patient?
A.>8,000 isolated premature ventricular contractions (PVCs) documented on 24-hour electrocardiography
monitoring.
B.Ventricular couplets during recovery from exercise stress testing.
C.Severe PR on echocardiography.
D.Right aortic arch noted on chest radiography.

BOARD Page 46

E.QRS duration of 210 ms on resting electrocardiogram.

The correct answer is E. A resting QRS duration of >180 ms has been established to be a risk factor for
syncope or sudden death in patients with repaired TOF.11 Isolated PVCs or ventricular couplets have not been
shown to correlate with adverse outcomes. Severe PR can lead to RV dilation and eventual dysfunction;
however, the finding of PR on echocardiography alone is not associated with definite adverse outcomes. The
finding of a right aortic arch is found in ~25% of patients with repaired TOF and does not imply increased risk
of adverse events.
References
1. Gatzoulis MA, Balaji S, Webber SA, et al. Risk factors for arrhythmia and sudden cardiac death late after
repair of tetralogy of Fallot: a multicentre study. Lancet 2000;356:975-81.

Question
50
of 60
A 29-year-old man undergoes an echocardiogram in evaluation of arrhythmias that have been occurring over
the prior year. Ambulatory ECG monitoring shows a wide complex irregular tachycardia associated with his
symptoms of palpitations. The echocardiogram is shown (Videos 1a, b).

http://assets.cardiosource.com/A8_16_6_Stout15a.mov
http://assets.cardiosource.com/A8_16_6_Stout15b.mov

Which of the following additional abnormality is commonly associated with Ebsteins anomaly?
A.Patent ductus arteriosus.
B.Bicuspid aortic valve.
C.Pulmonary stenosis.
D.Secundum ASD.
E.Anomalous pulmonary vein.

The correct answer is D. The most commonly associated abnormality is a secundum ASD/PFO, seen in over
three-fourths of patients. ASD/PFO may be clinically relevant, and can result in right-to-left shunting with
hypoxia or paradoxic emboli. Pulmonary stenosis is not as common, occurring in over one-third of patients,
although rarely is pulmonary stenosis severe. The other abnormalities have been reported in association with
BOARD Page 47

although rarely is pulmonary stenosis severe. The other abnormalities have been reported in association with
Ebstein's anomaly, although these are case reports and not common associations.

Question
51
of 60
A 19-year-old male with hypoplastic left heart syndrome presents to the adult congenital heart disease clinic
with a 2-week history of increasing pedal edema. He has gone several years without cardiology follow-up, but
called for an appointment when he was unable to fit into his shoes. He has undergone staged palliation for his
single ventricle physiology, with an extracardiac conduit type Fontan in childhood. In the course of outpatient
workup, laboratory testing reveals a serum albumin of 1.4 g/dl (normal range 3.2-5), and PLE is suspected.
Which of the following is the most accurate statement regarding PLE?
A.PLE is seen exclusively in patients with poor Fontan hemodynamics.
B.The diagnosis of PLE carries a significant 5-year associated mortality.
C.Steatorrhea is the presenting symptom most frequently associated with PLE.
D.Stool protein losses consist of albumin only.
E.Catheterization is rarely indicated in this setting.

The correct answer is B. PLE is an uncommon but significant source of morbidity and mortality; cumulative
mortality in one large multicenter series approached 50% at 5 years. While the elevated venous pressures in
the portal venous system with Fontan patients appear to have a role in the pathophysiology, some patients
with PLE have quite normal and acceptable Fontan hemodynamics. Presenting symptoms often include
peripheral edema, ascites, and pleural effusions due to hypoproteinemia, and increased stool frequency, but
rarely steatorrhea.
Protein losses are not limited to albumin, but include immunoglobulins and clotting system proteins, which in
severe cases can lead to immunodeficiency and predisposition to thrombosis. One can obtain a fecal alpha1
antitrypsin to confirm protein loss in the stool.-Catheterization is indicated with newly diagnosed PLE to assess
hemodynamics as well as to relieve any anatomic obstruction in the Fontan pathway (such as pulmonary
artery stenosis).
References
1. Warnes CA, Williams RG, Bashore TM, Child JS, Connolly HM, Dearani JA, Del NP, Fasules JW, Graham TP,
Jr., Hijazi ZM, Hunt SA, King ME, Landzberg MJ, Miner PD, Radford MJ, Walsh EP, Webb GD, Smith SC, Jr.,
Jacobs AK, Adams CD, Anderson JL, Antman EM, Buller CE, Creager MA, Ettinger SM, Halperin JL, Hunt
SA, Krumholz HM, Kushner FG, Lytle BW, Nishimura RA, Page RL, Riegel B, Tarkington LG, Yancy CW.
ACC/AHA 2008 guidelines for the management of adults with congenital heart disease: a report of the
American College of Cardiology/American Heart Association Task Force on Practice Guidelines (Writing
Committee to Develop Guidelines on the Management of Adults With Congenital Heart Disease).
Developed in Collaboration With the American Society of Echocardiography, Heart Rhythm Society,
BOARD Page 48

Developed in Collaboration With the American Society of Echocardiography, Heart Rhythm Society,
International Society for Adult Congenital Heart Disease, Society for Cardiovascular Angiography and
Interventions, and Society of Thoracic Surgeons. J Am Coll Cardiol. 2008;52:e1-121.

Question
52
of 60
A 48-year-old man presents to the emergency department with 3 hours of profound dyspnea and
lightheadedness and 2 days of right leg pain and swelling. He has a history of diabetes and coronary artery
disease status/post large anterior MI with a left ventricular ejection fraction of 30%. He is afebrile, but has a
blood pressure of 70/30 mm Hg and an oxygen saturation of 84% on room air. His ECG demonstrates sinus
tachycardia at 120 bpm and no other changes compared with a prior tracing. Chemistry panel, complete blood
count, and coagulation tests are normal. Cardiac troponin T is 2 ng/ml (normal <0.2 ng/ml). His chest X-ray
demonstrates cardiomegaly with clear lung fields. Contrast-enhanced chest CT demonstrates large bilateral
main pulmonary emboli. He is immediately started on intravenous unfractionated heparin as a bolus followed
by infusion.
Based on the presentation and initial assessment, which of the following is the most appropriate next step in
the management of this PE patient?
A.Refer the patient for catheter-based embolectomy.
B.Insert an IVC filter.
C.Administer recombinant tissue-plasminogen activator 100 mg over 2 hours via a peripheral intravenous line.
D.Refer the patient for open surgical pulmonary embolectomy.
E.Continue systemic anticoagulation with unfractionated heparin.

The correct answer is C. Answer C is correct because the patient has suffered massive PE and is at increased
risk of death from RV failure if reperfusion therapy is not instituted. He has no evidence of active bleeding,
thrombocytopenia, or other contraindication to fibrinolytic therapy.
Answers A and D are incorrect because the patient does not have any contraindications to fibrinolytic therapy.
Catheter-based and surgical pulmonary embolectomy are generally reserved for patients with
contraindications to fibrinolytic therapy or catheter-based techniques. Answer B is incorrect because IVC filter
placement will not reverse the RV pressure overload that is resulting in RV failure and hypotension. Answer E
is incorrect because systemic anticoagulation alone will not improve RV failure rapidly enough to avert further
hemodynamic collapse and possibly death.
References
1. Piazza G, Goldhaber SZ. Fibrinolysis for acute pulmonary embolism. Vasc Med 2010;15:419-28.
2. Jaff MR, McMurtry MS, Archer SL, et al. Management of massive and submassive pulmonary embolism,
iliofemoral deep vein thrombosis, and chronic thromboembolic pulmonary hypertension: a scientific
statement from the American Heart Association. Circulation 2011;123:1788-830.
BOARD Page 49

statement from the American Heart Association. Circulation 2011;123:1788-830.


3. Kearon C, Akl EA, Comerota AJ, et al. Antithrombotic therapy for VTE disease: Antithrombotic therapy
and prevention thrombosis, 9th ed: American College of Chest Physicians Evidence-Based Clinical
Practice Guidelines. Chest 2012;141:e419s-94s.

Question
53
of 60
A 68-year-old woman with a history of coronary disease status/post three-vessel coronary artery bypass
grafting presents to the clinic with symptoms of fatigue, dyspnea, edema, and early satiety. Examination is
remarkable for heart rate 58 bpm, and blood pressure is 134/85 mm Hg. There is jugular venous distention to
the jaw with prominent y descents. Heart is regular rate and rhythm, with a holosystolic murmur at the left
sternal border and an early diastolic sound. Abdomen is mildly distended with a fluid wave, and the liver is
palpated four fingerbreadths below the costal margin. There is 3+ bilateral pretibial edema noted.
Echocardiography shows normal LV size and function with ejection fraction 60%, moderate tricuspid
insufficiency, and estimated pulmonary artery systolic pressure of 55 mm Hg. Cardiac catheterization is
performed and reveals patent bypass grafts and the following hemodynamic tracings (Figure 1).

Which of the following do you recommend for this patient?


A.Endomyocardial biopsy.
B.Fat pad biopsy.
C.Tricuspid valve replacement.
BOARD Page 50

C.Tricuspid valve replacement.


D.Pericardiectomy.
E.Pericardiocentesis.

The correct answer is D. The correct answer is: Pericardiectomy.


This woman presents with marked right heart failure including systemic venous congestion, ascites, and
hepatomegaly. LV systolic function is normal, such that the differential diagnosis narrows to heart failure with
preserved ejection fraction, infiltrative/restrictive cardiomyopathy, constrictive pericarditis, pericardial
effusion, right-sided valvular disease, or isolated RV myopathy. The history of bypass surgery places her at
increased risk for post-pericardiotomy syndrome, which can lead to constriction.
The hemodynamic tracings show that during inspiration, there is an increase in RV systolic pressure with a
decrease in LV systolic pressure. This is termed enhanced ventricular interdependence, and is characteristic
of constrictive pericarditis (see transitions from beats 3-4 and particularly beats 10-11; Figure 2). In heart
failure or restrictive/infiltrative disease, LV and RV pressure would change similarly during respiration,
governed by the changes in intrathoracic pressure; thus, biopsy to assess for infiltrative disease would not be
warranted. Tricuspid valve surgery would not be the treatment of choice, as there is only moderate
insufficiency and the venous congestion is driven predominantly by pericardial constriction. Cardiac
tamponade results in blunting of the y descent rather than the rapid y descent noted on examination.

References
1. Hurrell DG, Nishimura RA, Higano ST, et al. Value of dynamic respiratory changes in left and right
ventricular pressures for the diagnosis of constrictive pericarditis. Circulation 1996;93:2007-13.
2. Talreja DR, Nishimura RA, Oh JK, Holmes DR. Constrictive pericarditis in the modern era: novel criteria
for diagnosis in the cardiac catheterization laboratory. J Am Coll Cardiol 2008;51:315-9.
3. Maisch B, Seferovic PM, Ristic AD, et al. Guidelines on the diagnosis and management of pericardial
diseases executive summary: the task force on the diagnosis and management of pericardial diseases of
the European Society of Cardiology. Eur Heart J 2004;25:587-610.
BOARD Page 51

the European Society of Cardiology. Eur Heart J 2004;25:587-610.

Question
54
of 60
A 30-year-old woman suffered chest trauma associated with a motor vehicular accident. She was the
restrained driver in a head-on collision at high velocity. She was alert on arrival to the emergency department,
but suffered progressive decrease in her level of consciousness, with a neurologic examination that
demonstrated a Glasgow coma scale of 7. She was sedated, intubated, and mechanically ventilated. A FAST
scan was conducted and was negative.
On physical examination, she has a blood pressure of 80/40 systolic in both upper extremities, and a heart rate
of 140 bpm, in apparent sinus tachycardia on telemetry. A neck examination demonstrates jugular venous
distention, with tracheal deviation to the left. Auscultation of the thorax demonstrates diminished breath
sounds on the right. An abdominal examination demonstrates diffuse ecchymosis, but no mass of note. The
extremities appear benign.
Which of the following is the next best step in this patients management?
A.Emergent pericardiocentesis and placement of a pericardial catheter, to address traumatic hemo-pericardial
tamponade.
B.Stat echocardiography to evaluate for structural issues, to include pericardial tamponade and septal rupture.
C.Left-sided decompression of a tension pneumothorax.
D.Right-sided decompression of a tension pneumothorax.

The correct answer is D. This patient has polytrauma, and will need a more detailed survey and evaluation to
define the extent of her injuries. The acute issue that must be addressed is her hemodynamic instability, and
the rapid reversal of identifiable contributing factors. Although pericardial tamponade is associated with
jugular venous distention, this would not explain the tracheal deviation or the poor aeration, and is less likely
in the setting of a negative FAST scan. Her examination demonstrates classic signs of a tension pneumothorax
with deviation of the trachea away from the side with the tension, and an ipsilateral hyperexpanded chest.
While the central venous pressure is usually elevated in a tension pneumothorax, note that it may be normal
or low if the patient is hypovolemic in the setting of concomitant hemorrhagic shock.1
References
1. Kulshrestha P, Munshi I, Wait R. Profile of chest trauma in a level I trauma center. J Trauma
2004;57:576-81.

Question

BOARD Page 52

55
of 60
A 65-year-old postmenopausal woman presents with ductal carcinoma in situ of the left breast. She has a prior
history of factor V Leiden and a deep vein thrombosis. Which of the following therapies should be avoided in
this patient?
A.Mastectomy.
B.Chest radiation.
C.Tamoxifen.
D.Aromatase inhibitor.
E.Coumadin.

The correct answer is C. Tamoxifen has mixed estrogen and antiestrogen properties, and has been shown to
be prothrombogenic in several clinical trials. In a meta-analysis of 32 breast cancer trials, the relative risk of
deep vein thrombosis was 1.87 and pulmonary embolus was 1.88, respectively, in patients who were treated
with tamoxifen therapy. Aromatase inhibitors are antiestrogenic and are associated with a significantly lower
risk of venous thromboembolism.
References
1. Braithwaite RS, Chlebowski RT, Lau J, George S, Hess R, Col NF. Meta-analysis of vascular and neoplastic
events associated with tamoxifen. J Gen Intern Med 2003;18:937-47.
2. Coombes RC, Kilburn LS, Snowdon CF, et al. Survival and safety of exemestane versus tamoxifen after
2-3 years' tamoxifen treatment (Intergroup Exemestane Study): a randomised controlled trial. Lancet
2007;369:559-70.
3. Forbes JF, Cuzick J, Buzdar A, Howell A, Tobias JS, Baum M. Effect of anastrozole and tamoxifen as
adjuvant treatment for early-stage breast cancer: 100-month analysis of the ATAC trial. Lancet Oncol
2008;9:45-53.

Question
56
of 60
The hemodynamic effects of CPAP include which of the following?
A.Increased afterload.
B.Increased preload.
C.Reduced preload.

BOARD Page 53

D.No hemodynamic effect.

The correct answer is C. CPAP increases intrathoracic pressure, which reduces blood return from the head,
abdomen, and lower extremities into the chest and heart. CPAP also reduces afterload, by decreasing the left
ventricular trans-mural pressure.
References
1. Yan AT, Bradley TD, Liu PP. The role of continuous positive airway pressure in the treatment of
congestive heart failure. Chest 2001;120:1675-85.

Question
57
of 60
A concerned parent brings her son to see you in the office because he is about to start playing college
basketball. She wants to know what causes SCD in young athletes, and whether her 19-year-old son is at risk
for such an event. Her son has not had any symptoms, nor cardiac testing, but his local team doctor has
cleared him to play all sports.
Which of the following are the top three causes (in correct order) of SCD in young athletes?
A.HCM, commotio cordis, and myocarditis.
B.Coronary artery disease, HCM, and commotio cordis.
C.Congenital heart disease, HCM, and myocarditis.
D.HCM, anomalous coronary artery, and Marfan syndrome.
E.HCM, commotio cordis, and anomalous coronary artery.

The correct answer is E. Epidemiologic data for the United States have been collected in a registry in
Minnesota for over 20 years. About one third of SCDs in athletes is caused by HCM, followed by commotio
cordis (direct blow to the chest during the vulnerable period of the cardiac cycle), and then anomalous
coronary artery. Myocarditis, Marfan syndrome, and congenital heart disease are further down the list of
causes. Coronary artery disease is the number one cause of SCD in the athlete over age 40 years.
References
1. Maron BJ, Doerer JJ, Haas TS, Tierney DM, Mueller FO. Sudden deaths in young competitive athletes:
analysis of 1866 deaths in the United States, 1980-2006. Circulation 2009;119:1085-92.

Question
58
BOARD Page 54

58
of 60
A 30-year-old man with known HCM and an implantable cardioverter-defibrillator (ICD) comes to your office
because he wishes to run a marathon. He has no symptoms, and has not had any shocks since the ICD was
implanted 5 years ago. Family history is negative for SCD, septal thickness by echo is 2.1 cm, and there is no LV
outflow obstruction.
Which of the following would be the proper recommendation according to the 36th Bethesda Conference
recommendations?
A.Since he is in a low-risk category, allow him to participate in the marathon.
B.Because he has an ICD, allow him to participate in the marathon.
C.Running a marathon is not recommended.
D.No sports are allowed, even low-dynamic, low-static sports.
E.All sports are allowed, provided genetic testing reveals a low-risk gene.

The correct answer is C. The 36th Bethesda Conference recommendations for participation in sports provide
recommendations as to whether play is allowable for an athlete with a cardiac condition. The underlying
cardiac pathology, and the physiology of the sport should be taken into consideration. As marathon running is
a high dynamic sport (Class IC), the athlete would be at high risk for a cardiac event during this activity. Thus, it
would not be allowed. In contrast, golf is considered low dynamic, and low static (Class IA), and would be
allowed.
Option A is not correct: An athlete with HCM would not be considered low risk, as the underlying substrate
is arrhythmogenic, regardless of pharmacologic therapy or corrective surgery. The presence of an ICD has not
yet been shown to protect against SCd under the conditions of athletic participation. Thus, option B is not
correct. There is reason to believe that ICDs may not be effective under the conditions of a rigorous sport,
where there are fluid and electrolyte shifts, and abnormally high levels of catecholamines. Option D is
incorrect because Class IA sports like golf are allowed. The Bethesda Conference recommendations do not
make a distinction between those HCM athletes who are genetically positive and those who are not. Thus,
option E is not correct.
References
1. Maron BJ, Ackerman MJ, Nishimura RA, Pyeritz RE, Towbin JA, Udelson JE. Task Force 4: HCM and other
cardiomyopathies, mitral valve prolapse, myocarditis, and Marfan syndrome. J Am Coll Cardiol
2005;45:1340-5.
2. Lawless CE. Implantable cardioverter defibrillators in athletes: rationale for use and issues surrounding
return to play. Curr Sports Med Rep 2008;7:86-92.
3. Lawless CE. Return-to-play decisions in athletes with cardiac conditions: guidelines and considerations.
In: Lawless CE, ed. Sports Cardiology Essentials: Evaluation, Management and Case Studies. 1st ed. New
York: Springer Science+Business Media; 2010:387-401.

Question
BOARD Page 55

Question
59
of 60
A 49-year-old woman with a history of chronic hypertension and diabetes mellitus is brought to the
emergency department by her husband, who reports that she has been acting confused. The patient, by his
report, was normal when he last saw her, but after returning from work, he found her to be lethargic and
less coherent than usual. Upon further questioning, her husband reports that the patient has been having a
difficult time with medication compliance and her diabetes, in particular, has been poorly controlled.
Her vital signs in the emergency department reveal a blood pressure of 240/120 mm Hg. Her exam is largely
unrevealing; other than her confusion, she has no focal neurologic deficits. A chest X-ray shows mild
pulmonary vascular redistribution and there is electrocardiographic evidence of left ventricular hypertrophy
with strain. A head computed tomography shows no acute intracranial process.
Which of the following is TRUE regarding the acute management of this patient?
A.The blood pressure should be acutely reduced to 120/80 mm Hg with intravenous antihypertensive therapy.
B.There is no target organ injury, so the blood pressure does not require acute lowering.
C.Thrombolytic therapy should be administered to treat a probable ischemic stroke.
D.The blood pressure should be reduced over minutes-to-hours to decrease the mean arterial pressure by
approximately 25%.
E.The patient should be admitted to a general medicine bed and treated with oral antihypertensive agents.

The correct answer is D. The patient has a hypertensive emergency. A hypertensive emergency represents an
acute elevation in blood pressure associated with end-organ injury. In this case, the end-organ manifestation
is her encephalopathy. Because there is end-organ injury, blood pressure should be lowered with parenteral
antihypertensive therapies. A patient should be monitored in a critical care setting, ideally with invasive blood
pressure monitoring. The goal for therapy is controversial, but most agree that blood pressure lowering should
occur over minutes-to-hours (except in aortic dissection, in which more rapid lowering is necessary), and
should target a modest reduction (e.g., decrease in mean arterial pressure of 20-25%). More rapid and
aggressive blood pressure lowering should be avoided, particularly in chronically hypertensive patients, as
they can experience critical hypoperfusion at normal blood pressures due to altered autoregulation.
References
1. Zampaglione B, Pascale C, Marchisio M, Cavallo-Perin P. Hypertensive urgencies and emergencies.
Prevalence and clinical presentation. Hypertension 1996;27:144-7.
2. Chobanian AV, Bakris GL, Black HR, et al. The Seventh Report of the Joint National Committee on
Prevention, Detection, Evaluation, and Treatment of High Blood Pressure: the JNC 7 report. JAMA
2003;289:2560-72.

Question

BOARD Page 56

60
of 60
A 58-year-old man with type 2 diabetes for 10 years visits his internist because he is worried about his
coronary disease risk. He is taking a statin for dyslipidemia (his last low-density lipoprotein was 90). His blood
pressure is 130/80 mm Hg. The patient denies any chest pain or shortness of breath. He walks daily for 30
minutes without any limitation. On physical exam, his blood pressure is 145/90 mm Hg and his heart rate is 78
bpm.
The internist should do which of the following?
A.Start an ACE inhibitor.
B.Order an exercise treadmill test (ETT).
C.Order an ETT with nuclear imaging.
D.Reassure him that he is at low risk.

The correct answer is A. Although coronary disease is highly prevalent in patients with diabetes mellitus,
screening for silent CAD in diabetics does not improve event-free survival in this patient population, as shown
in the DIAD study (see text). However, this patients blood pressure is not at target for a diabetic patient, and
an ACE inhibitor should be started.

The correct answer is B. Meta-analyses have shown a 1:1 ratio of blood pressure reduction for gram per day
reduction in salt intake. Therefore, a 2g/day reduction would lead to a 2mm Hg reduction in systolic blood
pressure. Blood pressure reductions would be greater for those with existing high blood pressure when
compared with those with elevated levels, but the mean level for a population would be in the 1:1 ratio. Mean
changes of 5-10mm Hg are more on the order for medications in a compliant population. However, at the
population level, a mean blood

Question
1
of 60
A reduction in daily salt intake by 2g/day would lead on average to which of the following reductions in
systolic blood pressure?

A.1 mm Hg.

B.2 mm Hg.

BOARD Page 57

C.5 mm Hg.

D.10 mm Hg.

pressure of 2mm Hg could result in up to 10 million deaths averted globally over a 10-year period.
References
1. Bibbins-Domingo K, Chertow GM, Coxson PG, et al. Projected effect of dietary salt reductions on future
cardiovascular disease. N Engl J Med 2010;362:590-9.
2. Asaria P, Chisholm D, Mathers C, Ezzati M, Beaglehole R. Chronic disease prevention: health effects and
financial costs of strategies to reduce salt intake and control tobacco use. Lancet 2007;370:2044-53.

Question
2
of 60
A novel biomarker is being evaluated for its ability to diagnose PE in patients presenting to the emergency
department with suggestive symptoms. Previous studies have suggested that a biomarker level over 500 U is
diagnostic for PE. One hundred patients are enrolled; their levels of this biomarker are measured, but not
revealed to the treating physicians in the emergency department, who are asked to determine whether the
patient has a PE based on standard testing alone.
The study results are as follows:

PE Diagnosed
No PE Diagnosed
Biomarker level >500 U 40
10
Biomarker level <500 U 20
30

Which of the following percentages is the specificity of this biomarker for diagnosis of PE?

A.40%.

B.60%.

BOARD Page 58

C.67%.

D.75%.

E.80%

The correct answer is D. Specificity is calculated as (true negatives)/(true negatives + false positives). For this
example, specificity = (30)/(30 + 10) = 30/40 = 0.75 or 75%.

Question
3
of 60
A 62-year-old woman with a history of hypertension and hyperlipidemia presents in the clinic with complaints
of exertional chest pressure associated with dyspnea that comes on predictably with climbing a flight of stairs
or hill. Her resting ECG is normal.
Which of the following is the next best diagnostic step for this patient?
A.Coronary calcium scoring.
B.Vasodilator nuclear scintigraphy.
C.Dobutamine stress echocardiography.
D.Treadmill echocardiogram.
E.Treadmill ECG stress testing.

The correct answer is E. The diagnostic accuracy of exercise-induced ST-segment depression for obstructive
CAD is lower in women. There are several reasons for this. Young and middle-aged women tend to have less
extensive CAD, which lowers the sensitivity to detect CAD using noninvasive tests such as exercise testing in
this group. In addition, it has been documented that catecholamine release during exercise is higher in
women, which could cause coronary vasoconstriction and increase ECG changes. Hormonal influences on
stress tests resulting in false-positive results have also been reported. Nonetheless, recommendations for the
initial evaluation of women with suspected CAD and a normal resting ECG with adequate exercise capacity
should be an exercise treadmill ECG test.
To optimize prognostic accuracy, the recommendations suggest that the ECG data should be integrated with
BOARD Page 59

To optimize prognostic accuracy, the recommendations suggest that the ECG data should be integrated with
the following: peak exercise capacity achieved, HR, and blood pressure changes. Imaging procedures (e.g.,
echo, SPECT) should be considered in symptomatic women with an intermediate-high probability of CAD,
when there are resting ECG abnormalities, or with reduced exercise capacity. Stress echocardiography and
SPECT stress tests perform similarly in men and women, and can be performed when indicated.
References
1. Mieres JH, Shaw LJ, Arai A, et al. Role of noninvasive testing in the clinical evaluation of women with
suspected coronary artery disease: Consensus statement from the Cardiac Imaging Committee, Council
of Clinical Cardiology, and the Cardiovascular Imaging and Intervention Committee, Council on
Cardiovascular Radiology and Intervention, American Heart Association. Circulation 2005;111:682-96.
2. Shaw LJ, Mieres JH, Hendel RH, et al., on behalf of the WOMEN Trial Investigators. Comparative
effectiveness of exercise electrocardiography with or without myocardial perfusion single photon
emission computed tomography in women with suspected coronary artery disease: results from the
What Is the Optimal Method for Ischemia Evaluation in Women (WOMEN) trial. Circulation
2011;124:1239-49.

Question
4
of 60
A 60-year-old businessman has history of obesity, with a body mass index (BMI) of 32. His history includes
prior cigarette smoking, hypertension, and hyperlipidemia. Six months ago, he decided to start climbing three
flights of stairs to his office each morning. When performing this activity, he notes a pressure sensation
localized to his substernal region that resolves within 2-3 minutes once he sits down at his desk. He describes
this symptom to his physician, who orders an ETT. The results are as follows: exercise duration 8 minutes of
the Bruce protocol; the patient requests to stop because of his chest pressure; exercise ECG is negative for
ischemia.
Which of the following statements is TRUE?
A.The post-test probability of CAD is >90%.
B.The post-test probability of CAD is <20%.
C.The post-test probability of severe (i.e., left main or three-vessel) CAD is <1%.
D.The annual risk of cardiac death is <1%.

The correct answer is A. A 60-year-old man with typical angina has a pretest probability of CAD that is >90%.
According to Bayes Theorem, a negative exercise ECG shifts this probability very little, and the posttest
probability of CAD remains >90%. Even for patients who can exercise into the third stage of the Bruce protocol
with a negative exercise ECG, the probability of left main or three-vessel CAD is approximately 10%. This
patients Duke treadmill score can be calculated as 0: 8 minutes Bruce protocol 0 (for no ST-segment
depression) 8 (4 x 2, for test limiting angina). A Duke treadmill score of zero places a patient into the
intermediate-risk category for cardiac death, where annual cardiac mortality ranges between 1-3%.

BOARD Page 60

Question
5
of 60
A 67-year-old male is seen in your office for post-MI follow-up. He was hospitalized 1 month prior with
worsening dyspnea of 2 days duration and diagnosed with an acute ST-elevation MI. He presented to an
outside hospital and received thrombolytic therapy with symptom relief and electrocardiographic resolution
of lateral ST elevation. Prior to discharge, he underwent a resting echocardiogram, which showed an EF of 44%
with akinesis and thinning of the lateral wall. He presents today with new exertional dyspnea when carrying
out his garbage. His current medications include aspirin, atorvastatin, atenolol, and enalapril.
Physical exam shows a regular heart rate of 62 bpm, blood pressure of 135/76 mm Hg, a normal heart exam,
including a jugular venous pressure of 6 cm water and clear lung fields. An ECG shows sinus rhythm with
pathologic Q waves in leads 1, aVL and V6 with associated T-wave inversions.
You order a pharmacologic stressrest Rb-82 PET with F-18 FDG scan for ischemia and viability assessment.
After the stress-rest PET is completed, you are called to review the study to make a decision whether to
proceed with viability scan. The images are shown (Figure 1).

After reviewing the images, which of the following is the next appropriate step?
A.Proceed with F-18 FDG viability scan.
B.Cancel F-18 FDG scan.
BOARD Page 61

B.Cancel F-18 FDG scan.


C.Request technologist to repeat stress scan.
D.Administer sublingual nitrate and re-inject Rb-82 for a third set of images.
E.Reprocess images with masking to eliminate gastric artifact.

The correct answer is B. The images show a large moderate severity predominantly
reversible defect along the basal-mid lateral wall consistent with ischemia in likely the
previously infarcted left circumflex territory. The remainder of the myocardium is well
perfused and nonischemic. Given that the viability question would arise only if there was a
fixed defect in the infarcted zone, in this case, the FDG portion of the study should be
cancelled to avoid unnecessary radiation, as the lateral wall is clearly ischemic and the
patient should be scheduled for coronary angiography. The scan quality is good with no
artifacts, and nitrates were not used in the PET imaging. The study also reflects the
limitations of using resting wall motion, Q waves, or wall thinning as signs for lack of viability.

Question
6
of 60
A middle-aged man with a Framingham risk score of 11% has a calcium score of 0.
Based on this finding, further evaluation using coronary CT angiography to detect noncalcified plaque is
considered which of the following?
A.Appropriate.
B.Uncertain appropriateness.
C.Inappropriate.
D.Unknown appropriateness.

The correct answer is C. Noncalcified plaque can be identified using coronary CT angiography as the only
finding in approximately 10% of intermediate risk, asymptomatic patients. The available data suggest that
noncalcified, nonobstructive plaque may portend increased cardiovascular risk, particularly when associated
with features of low CT attenuation or outward arterial remodeling. Despite this, a number of factors limit
acceptance of this approach, including concerns on increased radiation exposure with CT angiography, need
for iodinated contract, the potential for false-positive and false-negative findings, and the lack of evidence on
whether management and outcomes are altered. The 2010 ACCF Appropriate Use Criteria identify coronary CT
angiography as inappropriate in asymptomatic intermediate-risk patients.
References
BOARD Page 62

References
1. Taylor AJ, Cerqueira M, Hodgson JM, et al. ACCF/SCCT/ACR/AHA/ASE/ASNC/NASCI/SCAI/SCMR 2010
appropriate use criteria for cardiac computed tomography. A report of the American College of
Cardiology Foundation Appropriate Use Criteria Task Force, the Society of Cardiovascular Computed
Tomography, the American College of Radiology, the American Heart Association, the American Society
of Echocardiography, the American Society of Nuclear Cardiology, the North American Society for
Cardiovascular Imaging, the Society for Cardiovascular Angiography and Interventions, and the Society
for Cardiovascular Magnetic Resonance. J Am Coll Cardiol 2010;56:1864-94.

Question
7
of 60
A 67-year-old man with type 2 diabetes presents for a scheduled diagnostic cardiac catheterization. He has
been having new exertional angina for a month and is associated with a positive stress echo, which suggested
multivessel disease. His ejection fraction was 55% with anterior and inferior wall ischemia. Over the last
several days, he has had worsening angina with little activity. He is admitted and biomarkers are normal. He
undergoes diagnostic catheterization. He is receiving a beta-blocker, statin, aspirin, and a long-acting nitrate.
His diabetes is controlled with insulin and metformin, and the metformin is discontinued the day of the
procedure. His creatinine is 1.4 mg/dl (creatinine clearance 60 cc/min). He receives 1 liter of IV hydration
before the procedure. Coronary angiography reveals focal mid LAD and mid RCA subtotal stenosis. The
following day after his creatinine is shown to be unchanged, he is loaded with clopidogrel, and he goes on to
have uncomplicated stenting of both lesions.
After PCI, which of the following is the most appropriate management of his medications?
A.Resume all preprocedure medications immediately.
B.Resume all medications except metformin.
C.Resume all medications, but hold metformin and insulin for 48 hours pending laboratory results.
D.Resume all medications and restart metformin in 48 hours after showing that the creatinine is stable.

The correct answer is D. Lactic acidosis associated with metformin is a rare, but sometimes fatal
complication. Because metformin is renally excreted, renal dysfunction increases the risk for lactic acidosis
due to metformin accumulation. Therefore, metformin is contraindicated in the presence of baseline renal
dysfunction (serum creatinine 1.5 mg/dl in men, 1.4 mg/dl in women, or creatinine clearance 50 ml/min).
Contrast-induced nephropathy is a known risk after any dye exposure, and the magnitude of risk is directly
related to the presence of pre-existing renal dysfunction. Diabetes increases that risk.
To be certain contrast-induced nephropathy has not occurred in any patient, metformin should not be
resumed for 48 hours after the last contrast exposure and then only when renal function is documented to
have returned to baseline. Metformin should not be resumed immediately (option A), metformin need not be
discontinued indefinitely (option B), and insulin should not be stopped, as this could lead to loss of diabetic
control (option C). The usual recommendation is to hold metformin until the creatinine is reassessed 48 hours
after the procedure (option D).
BOARD Page 63

after the procedure (option D).


References
1. Tommaso CL. Contrast-induced nephrotoxicity in patients undergoing cardiac catheterization. Cathet
Cardiovasc Diagn 1994;31:316-21.
2. Heupler FA Jr. Guidelines for performing angiography in patients taking metformin. Members of the
Laboratory Performance Standards Committee of the Society for Cardiac Angiography and Interventions.
Cathet Cardiovasc Diagn 1998;43:121-3

Question
8
of 60
Ionizing radiation can result in cellular DNA injury. A stochastic injury from medical radiation exposure results
in which of the following consequences?
A.Skin injury.
B.Hair loss.
C.Carcinogenesis.
D.Nausea and vomiting.
E.Cellular death.

The correct answer is C. Stochastic injuries can occur after a single exposure to the cell. It is therefore not
threshold dependent. The DNA injury that results does not kill the cell, but rather the cell lives on mutated.
This can eventually lead to cancers later in life. A deterministic injury results when the cell dies from the DNA
damage. If enough cells of an organ die, then the organ becomes dysfunctional. This is seen most often in skin
damage, but can result in hair loss or, if extensive (such as acute radiation injury), can result in sloughing of
the gastrointestinal tract. Deterministic injury only occurs after a threshold level of radiation exposure has
occurred.

Question
9
of 60
A 68-year-old female with diabetes and tobacco history is referred for evaluation of pain in both legs. Upon
questioning, the patient states that she gets pain while standing for long time periods and walking >100-150
BOARD Page 64

questioning, the patient states that she gets pain while standing for long time periods and walking >100-150
feet. The pain is characterized as a sharp and occasionally cramping pain in her calves, which is relieved with
rest. Over the last month, the pain has worsened and occurs sometimes at night.
On examination, the patient has a blood pressure of 148/68 mm Hg and heart rate of 68 bpm. She does not
have carotid bruits, and she has some point tenderness in her low-back sacro-iliac region. Her straight leg test
reveals some limitation in movement. She has loss of sensation in her toes, and pulse examination reveals
normal radial, brachial, and femoral pulses. Her posterior tibial and dorsalis pedis pulses are palpable but
diminished.
Given the patients symptoms, which of the following is the most appropriate test to perform next?
A.Magnetic resonance imaging of the lumbar-sacral spine.
B.Rest and exercise ABI.
C.Magnetic resonance angiography of the abdomen and lower extremities.
D.Nerve conduction studies of the lower extremities.
E.Vascular ultrasound of the lower extremities.

The correct answer is B. The patient presented represents a typical patient seen in clinical practice where the
determination of the etiology of exertional leg pain is complex. The differential diagnosis for this patient
includes PAD leading to intermittent claudication versus spinal disease and sciatica (often referred to as
pseudoclaudication). Magnetic resonance imaging of the lumbar spine may be indicated once PAD has been
excluded, but at this point it is an expensive first test that is not indicated. Rest and exercise ABI is the correct
option. It allows for evaluation of PAD and excludes exertional leg pain due to PAD. The ACC/AHA guidelines
recommend this test for the evaluation of suspected pseudoclaudication.
Magnetic resonance angiography and vascular ultrasound of the lower extremities would provide anatomic
information that might be useful, but PAD has to be established first. Nerve conduction studies would be
indicated if peripheral neuropathy was suspected.
References
1. Hirsch AT, Haskal ZJ, Hertzer NR, et al. ACC/AHA 2005 guidelines for the management of patients with
peripheral arterial disease (lower extremity, renal, mesenteric, and abdominal aortic): executive
summary a collaborative report from the American Association for Vascular Surgery/Society for Vascular
Surgery, Society for Cardiovascular Angiography and Interventions, Society for Vascular Medicine and
Biology, Society of Interventional Radiology, and the ACC/AHA Task Force on Practice Guidelines
(Writing Committee to Develop Guidelines for the Management of Patients With Peripheral Arterial
Disease) endorsed by the American Association of Cardiovascular and Pulmonary Rehabilitation;
National Heart, Lung, and Blood Institute; Society for Vascular Nursing; TransAtlantic Inter-Society
Consensus; and Vascular Disease Foundation. J Am Coll Cardiol 2006;47:1239-312.

Question
10

BOARD Page 65

of 60
According to the NCEP ATP III guidelines, which of the following is a CHD risk equivalent?
A.Asymptomatic carotid artery disease with stenosis exceeding 50%.
B.Popliteal artery aneurysm.
C.Parental history of MI before the age of 40 years.
D.Sibling history of MI before the age of 40 years.
E.Hs-CRP protein >3 mg/L.

The correct answer is A. CHD risk equivalents defined by the NCEP ATP III include lower extremity peripheral
artery disease, symptomatic carotid artery disease, asymptomatic carotid artery disease with stenosis
exceeding 50%, abdominal aortic aneurysm, diabetes mellitus, and a 10-year hard CHD FRS exceeding 20%.

Question
11
of 60
A 46-year-old man with a strong family history of coronary artery disease, but no other significant past
medical history, presents for a general evaluation. His weight is 210 lbs, BMI 31 kg/m2, waist circumference 40
inches, and BP 135/84 mm Hg. Laboratory studies reveal a total cholesterol of 189 mg/dl, LDL-C of 110 mg/dl,
HDL-C of 38 mg/dl, triglycerides of 205 mg/dl, and a fasting glucose of 107 mg/dl. He is a nonsmoker and only
exercises sporadically.

In terms of initial recommendations for weight loss in this patient, which of the following is correct?
A.He should attempt to lose 35 lbs in the next 6 months.
B.He should reduce his calorie intake by 500-1000 kcal/day.
C.Increasing his exercise is more likely to result in weight loss than caloric reduction.
D.He should perform at least 30 minutes a day of moderate intensity physical activity that should occur all at
once rather than in segments.

The correct answer is B. Caloric reduction of 500-1000 kcal/day will result in 1-2 lbs of weight loss per week,
BOARD Page 66

The correct answer is B. Caloric reduction of 500-1000 kcal/day will result in 1-2 lbs of weight loss per week,
which is optimal. The initial goal for weight loss is 10% of body weight, which is approximately 20 lbs in this
patient. According to the NHLBI Practical Guide, more weight loss occurs due to caloric reduction than to
increased physical activity, although both are recommended to facilitate weight loss. Physical activity can be
accumulated in segments, and does not have to be done at one time. In an overweight patient who is
sedentary, low or very low intensity physical activity for short sessions (i.e., just a few minutes) may be
recommended initially.

Question
12
of 60
A 72-year-old man with a history of MI, triple-vessel coronary artery bypass surgery (9 years ago), and a recent
percutaneous coronary intervention/stenting to the proximal right coronary artery for exertional angina is
referred to your practice. His LVEF is 35%. Current medications include: aspirin, clopidogrel, statin, and a betablocker. A follow-up peak or symptom-limited exercise stress test revealed borderline significant (~1 mm) STsegment depression at peak exercise, isolated premature ventricular contractions, no anginal symptoms, and
a physiologic blood pressure response. His exercise capacity is 7 METs.
Which of the following activities would you advise against?
A.Bowling.
B.Billiards.
C.Walking at 3 mph up to 2.5% grade.
D.Snow shoveling.
E.Sexual activity.

The correct answer is D. Snow shoveling is associated with excessive cardiac demands and an aerobic
requirement of approximately 6 METs. Each year, numerous nonfatal CV events are reported during or soon
after snow shoveling in middle-aged and older adults with known or occult CVD.

Question
13
of 60
Mr. K is a white 76-year-old male who presents to the ED. He complains for epigastrial pressure, shortness of
BOARD Page 67

Mr. K is a white 76-year-old male who presents to the ED. He complains for epigastrial pressure, shortness of
breath, nausea, and fatigue, all of which suddenly started about 2 hours earlier. He has been in good health in
the past, has no major comorbidities, and his history is remarkable only for diabetes treated with oral
hypoglycemic medication.
He is clinically stable with a blood pressure of 112/78 mm Hg, heart rate 75 bpm, S02 96%, and temperature
36.4C. His ECG showed ST depression and large, inverted T waves in V1-V3. An initial set of cardiac
biomarkers turns out to be negative. After a tablet of sublingual nitroglycerin, he says that his symptoms are
slightly improved.
Which of the following is the most appropriate next step for management?
A.Keep him in the ED, monitor his ECG, and perform a second set of cardiac markers in 6 hours.
B.This patient likely has UA, but no high-risk features. Admit him to the floor and manage him conservatively.
C.A recording of posterior ECG leads (V7-V9) is needed to better define the diagnosis.
D.This patient likely has UA, no clinical instability, but several risk factors. Application of a risk score will guide
further management.
E.The patient has atypical symptoms, nondiagnostic ST depression, and negative cardiac markers. Order a
stress protocol to clarify the diagnosis.

The correct answer is C. The patient has a true posterior MI presented as possible UA.
Option A is incorrect. This patient presents with possible ischemic symptoms and ST-segment depression on
ECG. Therefore, he has high likelihood and high risk. While a second set of cardiac markers should be
obtained, this should not delay the admission and proper treatment. Moreover, the presence of maximal STdepression in V1-V3 should prompt recording ECG posterior leads to rule out a true posterior MI. Presence of
ST-segment elevation in V7-V9 will confirm the diagnosis of true posterior STEMI and the need to perform
urgent reperfusion.
Option B is incorrect. MI rule-out requires negative serial markers (at least two negative sets of cardiac
markers 6 hours apart). Also, ST depression is a high-risk feature, and in this case, the location should induce
the suspicion of posterior STEMI. Finally, this patient has ongoing symptoms.
Option D is incorrect. MI rule-out requires negative serial markers (at least two negative sets of cardiac
markers 6 hours apart). Posterior ECG lead recording should be done.
Option E is incorrect. Elderly and diabetic patients often have atypical symptoms as angina equivalents. The
presence of a deviation of the ST segment is the most specific sign of ischemia. Therefore, the probability that
the presenting symptoms are indeed related to cardiac ischemia is substantially increased by the ECG findings.
The patient is also still symptomatic. A stress test would be unsafe in the setting of ongoing ischemia.

Question
14

BOARD Page 68

of 60
Which of the following phrases bests describes tenecteplase?
A.Antigenic, administered as two boluses 10 minutes apart, not fibrin specific.
B.Nonantigenic, administered as two boluses 10 minutes apart, fibrin specific.
C.Nonantigenic, administered as single bolus, not fibrin specific.
D.Nonantigenic, administered as a single bolus, fibrin specific.
E.Antigenic, administered as a single bolus, fibrin specific.

The correct answer is D. Tenecteplase, or TNK, is a fibrin-specific, nonantigenic fibrinolytic. Although its
administration is weight based, its main advantage is that it can be administered as a single bolus. Reteplase,
or Retavase, has limited fibrin specificity. It is given as two boluses 10 minutes apart but does not have to be
adjusted for weight. TPA has moderate fibrin specificity but is administered as a drip. Streptokinase has no
fibrin specificity and is highly antigenic.
References
1. Antman EM, Anbe DT, Armstrong PW, et al. ACC/AHA guidelines for the management of patients with
ST-elevation myocardial infarction--executive summary: a report of the American College of
Cardiology/American Heart Association Task Force on Practice Guidelines (Writing Committee to revise
the 1999 guidelines for the management of patients with acute myocardial infarction). J Am Coll Cardiol
2004;44:671-719.
2. Bonow RO, Mann DL. Braunwald's Heart Disease: A Textbook of Cardiovascular Medicine. Philadelphia:
W.B. Saunders Company; 2011.

Question
15
of 60
A 79-year-old man calls emergency medical services, complaining of crushing chest pain for the past 15
minutes. Upon arrival to the emergency room, symptoms have been present for 35 minutes. Medical history is
notable for hypertension and a 40-pack/year tobacco history. ECG upon arrival demonstrates anterior ST
elevations. Blood pressure is 95/55 mm Hg, and heart rate is 82 bpm. Routine laboratories, drawn by his
primary care physician 3 days prior, demonstrate normal baseline creatinine. The receiving facility does not
have a cardiac catheterization laboratory. Transfer to a facility with interventional capabilities will require at
least 120 minutes.
Which of the following is an appropriate management strategy for this patient?
A.325 mg aspirin/60 mg prasugrel/TPA, followed by immediate transfer to the PCI-capable facility.

BOARD Page 69

B.325 mg aspirin/300 mg clopidogrel/transfer for primary PCI.


C.325 mg aspirin/heparin/reteplase, followed by immediate transfer to the PCI-capable facility.
D.325 mg aspirin/abciximab/half-dose tenecteplase and transfer for PCI.
E.325 mg aspirin/tenecteplase/enoxaparin without transfer for PCI.

The correct answer is C. Given the time that would be required for transfer to occur, the patient should
undergo fibrinolysis at the initial facility, rather than awaiting definitive management after transfer.
Expeditious transfer should nonetheless follow lysis, given the results of the CARESS and TRANSFER-AMI trials;
hence, while the medical therapies in option E are reasonable, the patient should then be transferred.
Prasugrel (option A) has not been studied in the setting of fibrinolytics, and thus, should not be employed in
this setting. The loading dose of clopidogrel (option B) is reasonable, but transfer for PCI prior to attempted
reperfusion via fibrinolysis is inadvisable in this setting. Facilitated PCI with reduced-dose lytic administered
prior to planned PCI was studied in the ASSENT-4 trial, and was associated with increased in-hospital
mortality; thus, option D is incorrect.
References
1. Di Mario C, Dudek D, Piscione F, et al. Immediate angioplasty versus standard therapy with rescue
angioplasty after thrombolysis in the Combined Abciximab REteplase Stent Study in Acute Myocardial
Infarction (CARESS-in-AMI): an open, prospective, randomised, multicentre trial. Lancet
2008;371:559-68.
2. Cantor WJ, Fitchett D, Borgundvaag B, et al. Routine early angioplasty after fibrinolysis for acute
myocardial infarction. N Engl J Med 2009;360:2705-18.
3. Assessment of the Safety and Efficacy of a New Treatment Strategy with Percutaneous Coronary
Intervention (ASSENT-4 PCI) investigators. Primary versus tenecteplase-facilitated percutaneous
coronary intervention in patients with ST-segment elevation acute myocardial infarction (ASSENT-4 PCI):
randomised trial. Lancet 2006;367:569-78.

Question
16
of 60
A 46-year-old man with a history of long-standing diabetes presents to the emergency department with 16-20
hours of generalized malaise followed by shortness of breath. He has a strong family history of premature CAD
and he has an extensive smoking history dating back to when he was a teenager. In the emergency
department, his ECG showed sinus tachycardia at a rate of 110 bpm with QS complexes across the anterior
precordium. His SBP was 110/70 mm Hg in the right upper limb, and a heart rate of 18 bpm. He weighed 100
kg, and his heart sounds were faint with no significant murmurs. He was transferred emergently for cardiac
catheterization, which revealed single-vessel disease with a completely occluded proximal LAD with TIMI 0
flow. A DES was placed with TIMI 2 flow at the end of the procedure. An echocardiogram reveals an apical
aneurysm with a large LAD wall motion abnormality and a calculated LVEF of 36%. The patient has an
uneventful early postinfarction course. His baseline creatinine is 1.2 mg/dl and his serum potassium is 4
mEq/L.
BOARD Page 70

mEq/L.
Which of the following in addition to dual antiplatelet therapy and a statin constitutes ideal outpatient
therapy?
A.Metoprolol 50 mg twice daily and enalapril 20 mg/day.
B.Metoprolol 100 mg twice daily and losartan 100 mg/day.
C.Carvedilol 25 mg twice daily and eplerenone 25 mg/day.
D.Carvedilol 25 mg twice daily, lisinopril 40 mg/day, and eplerenone 25 mg/day.
E.Metoprolol SR 200 mg/day, ramipril 10 mg/day, and valsartan 160 mg/day.

The correct answer is D. The patient described is recovering from a large anterior wall MI and has significantly
reduced discharge LV ejection. The subject is at risk for adverse ventricular remodeling, LV dilatation, and
clinical HF. In this setting, a medical regimen consisting of carvedilol, lisinopril, and eplerenone will provide
him optimal therapeutic benefit.

Question
17
of 60
A 30-year-old man with diabetes mellitus and peripheral arterial disease is referred for evaluation. He has no
history of angina pectoris. His vital signs and physical examination are normal. Serum laboratories show a total
cholesterol of 205 mg/dl, LDL-C of 115 mg/dl, and HDL-C of 45 mg/dl. A 12-lead electrocardiogram (ECG) is
normal.
Which of the following medications should be prescribed to reduce his CV mortality?
A.Beta-blocker.
B.Long-acting calcium channel blocker.
C.Niacin.
D.ACE inhibitor.
E.Aspirin.

The correct answer is D. Administration of ramipril to patients at "high-risk" for CAD (i.e., those with known
coronary, peripheral, or cerebral vascular disease, or those with diabetes and at least one other coronary risk
factor) decreased mortality approximately 20% in the HOPE study. Decreased mortality with beta-blockers
BOARD Page 71

factor) decreased mortality approximately 20% in the HOPE study. Decreased mortality with beta-blockers
occurs in patients with prior MI. Neither calcium channel blockers nor niacin have been shown to decrease
mortality. Aspirin may reduce vascular events in middle-aged men (40 years old), but it has not been proven
effective in younger individuals.

Question
18
of 60
You are asked to evaluate a 76-year-old woman with severe osteoarthritis of the right knee who is scheduled
for total knee replacement surgery. Her medical history is notable for diabetes, hypertension, and CABG 8
years ago. She describes occasional mild substernal chest discomfort with exertion that is relieved with rest or
sublingual nitroglycerin. Her symptoms are unchanged from an office visit 12 months earlier. Medications
include aspirin, atenolol, lisinopril, atorvastatin, and isosorbide mononitrate. On physical examination, her
blood pressure is 134/78 mm Hg, and her CV exam is unremarkable.
Which of the following is the best next step in her treatment?
A.Recommend that atenolol be continued during the perioperative period.
B.Refer for coronary angiography; perform PCI if indicated and defer surgery for at least 12 months if a drugeluting stent is used.
C.Refer for coronary angiography; if indicated, perform PCI with a bare-metal stent and delay surgery for at
least 6 weeks.
D.Refer for coronary angiography and perform PCI with balloon angioplasty, if possible.

The correct answer is A. This patient has chronic mild angina and clinically stable CAD. There is no evidence
that revascularization prior to noncardiac surgery improves outcomes, and the indications for
revascularization for this patient are no different than for any patient with SIHD. In cases when preoperative
revascularization is required, drug-eluting stents should be used only when there is a reasonable degree of
confidence that dual antiplatelet therapy can be continued without interruption for 12 months. Following
bare-metal stenting, surgery should be delayed for at least 6 weeks; if surgery cannot be delayed and dual
antiplatelet therapy cannot be continued, then PCI with balloon angioplasty should be considered.1
References
1. Levine GN, Bates ER, Blankenship JC, et al. 2011 ACCF/AHA/SCAI guideline for percutaneous coronary
intervention. A report of the American College of Cardiology Foundation/American Heart Association
Task Force on Practice Guidelines and the Society for Cardiovascular Angiography and Interventions. J
Am Coll Cardiol 2011;58:e44-122.

Question

BOARD Page 72

19
of 60
A 65-year-old woman with hypertension is admitted to the hospital with chest pain for 1 hour and an ST
elevation MI. She immediately undergoes successful percutaneous coronary intervention of her infarct-related
artery. A DE-CMR viability study was obtained (Figure 1). Her LVEF is 35%.

You conclude that her myocardial dysfunction is a result of which of the following?
A.Cardiac sarcoid.
B.Myocardial stunning.
C.Myocardial hibernation.
D.Myocarditis.
E.A large LAD MI.

The correct answer is B. Options A, D, and E are incorrect because the DE-CMR shows no evidence of any of
these conditions. Option C is incorrect because coronary blood flow has been restored in this patient

Question
20
of 60
BOARD Page 73

of 60
A 67-year-old woman visiting the United States from the Netherlands presents to the emergency department
following a brief episode in which she lost vision in her left eye. Physical examination is unremarkable. A head
CTA does not disclose evidence of acute intracranial pathology. She reports that she has had several months of
episodic headache and pain in the jaw when she chews.
Which of the following diagnostic tests is most likely to yield the definitive diagnosis?

A.Serology for erythrocyte sedimentation rate.

B.CTA of the head and neck.

C.Fat pad biopsy.

D.Temporal artery biopsy.

The correct answer is D. This patient's history and demographic background are most
likely consistent with temporal (giant cell) arteritis. Temporal artery biopsy and
histopathologic evaluation may provide the definitive diagnosis. The sedimentation rate
typically is elevated in temporal arteritis, as in many other inflammatory conditions; choice A
therefore does not provide definitive diagnosis. A CTA of the head and neck may not
demonstrate definitive arteriopathy. A fat pad biopsy would not show abnormality in temporal
arteritis.

Question
21
of 60
A 57-year-old woman with a history of cigarette smoking and hypertension presents to the emergency room 8
hours after the onset of left-sided weakness. A head CT scan confirms the clinical diagnosis of cerebral
infarction, and there is no evidence of ICH. Which of the following is a part of optimal early management of
ischemic stroke?
A.Intravenous unfractionated heparin without a bolus, targeting a goal partial thromboplastin time of 50-70
seconds, for 48 hours.

BOARD Page 74

B.Allowing moderate hypertension without intensive blood pressure management with a target blood
pressure of <130/80 mm Hg.
C.The addition of either clopidogrel or extended-release dipyridamole to aspirin therapy.
D.Therapeutic hypothermia should be considered in patients presenting with acute ischemic stroke of a high
degree of severity.
E.The administration of an ACE inhibitor such as perindopril 4 mg a day.

The correct answer is B. Option A is incorrect. Heparin, either unfractionated or low molecular weight, is not
recommended for the early treatment of ischemic stroke. The use of heparin should be considered in ischemic
stroke of cardioembolic origin, not to benefit the stroke that has already occurred, but rather to prevent a
second stroke event.1
Option B is correct. Available data suggest that intensive lowering of BP in the acute stroke setting is
associated with worse cerebral and cardiovascular outcomes. It is recommended that the BP be lowered by
approximately 15% if systolic BP is >220 mm Hg or diastolic BP >120 mm Hg, and that the BP be maintained at
<180/105 mm Hg for 24 hours after rt-PA is given.1
Option C is incorrect. The only antiplatelet therapy that has been tested in the early treatment of ischemic
stroke is aspirin. Nonaspirin antiplatelet therapy has been found to improve outcomes in the secondary
prevention of ischemic stroke, but not in the acute stroke setting.1
Option D is incorrect. Though of great therapeutic potential, a clinical trial of therapeutic hypothermia in the
setting of acute ischemic stroke has not been reported. It has been found, however, to improve neurological
outcomes in the setting of some types of out-of-hospital cardiac arrest.
Option E is incorrect. It has not been shown that ACE inhibitors are of benefit in the early treatment of
ischemic stroke.
References
1. Adams HP Jr, del Zoppo G, Alberts MJ, et al. Guidelines for the early management of adults with
ischemic stroke: a guideline from the American Heart Association/American Stroke Association Stroke
Council, Clinical Cardiology Council, Cardiovascular Radiology and Intervention Council, and the
Atherosclerotic Peripheral Vascular Disease and Quality of Care Outcomes in Research Interdisciplinary
Working Groups: The American Academy of Neurology affirms the value of this guideline as an
educational tool for neurologists. Circulation 2007;115:e478-534.

Question
22
of 60
According to the Framingham Heart Study, at 40 years of age, the lifetime risk for the development of HF for
both men and women is which of the following?
BOARD Page 75

both men and women is which of the following?


A.1 in 2.
B.1 in 5.
C.1 in 10.
D.1 in 15.
E.1 in 20.

The correct answer is B. In the Framingham Heart Study, at 40 years of age, the lifetime risk for the
development of HF for both men and women is 1 in 5.1 Without an antecedent MI, at 40 years of age, the
lifetime risk for the development of HF is 1 in 9 for men and 1 in 6 for women.1
References
1. Tang WH, Francis GS. Natural history of heart failure. Chapter 1. In: Kukin ML, Fuster V, eds. Oxidative
Stress and Cardiac Failure. Armonk, NY: Futura Publishing Company; 2002: 3-47.

Question
23
of 60
Ms. Simpson is a 54-year-old woman with familial dilated cardiomyopathy. Her last measured ejection fraction
was 20%. She has stable New York Heart Association class III symptoms. She is on a regimen of carvedilol 25
mg bid, lisinopril 20 mg daily, and spironolactone 25 mg daily. She uses furosemide when necessary. She had a
cardiac resynchronization therapy device placed 5 years ago.
Which of the following perturbations will result in the largest fall in stroke volume?
A.A 40 mm Hg rise in systolic blood pressure.
B.A 5 mm Hg fall in left atrial pressure.
C.A 20% rise in circulating catecholamines.
D.Holding furosemide for 2 weeks.
E.Addition of digoxin.

The correct answer is A. Because of the flattened Frank-Starling relationship, hearts with chronic systolic
dysfunction are relatively insensitive to changes in preload, but are very sensitive to changes in afterload
BOARD Page 76

dysfunction are relatively insensitive to changes in preload, but are very sensitive to changes in afterload
because of the increased steepness of the relationship between stroke volume and systemic vascular
resistance. A 40 mm Hg rise in systolic blood pressure is indicative of a large change in afterload, and will lead
to a significant fall in stroke volume in a severely dysfunctional ventricle such as Ms. Simpson's. A rise in
circulating catecholamines will increase the contractile state of the heart and increase stroke volume.
However, the failing heart is relatively resistant to changes in catecholamine levels because of chronic
downregulation and desensitization of beta-receptors.
Chronic use of beta-blocking drugs may have restored some of this sensitivity in the patient. Because the
patient uses furosemide only when necessary, holding it for 2 weeks is unlikely to result in a significant
increase in preload. Because the pressure-volume relationship is flattened in chronic HF, increases in preload
are relatively ineffective at increasing stroke volume. Addition of digoxin may result in a mild increase in
inotropic state, but this effect is very minor and may provide a small increase in stroke volume; the
predominant effect of digoxin is likely neurohormonal and improves the balance between sympathetic and
parasympathetic tone.

Question
24
of 60
A 35-year-old woman has been followed in your clinic for 5 years for heart failure in the setting of a
nonischemic cardiomyopathy. She has been treated with beta-blockers, angiotensin-converting enzyme
inhibitors, furosemide 60 mg bid, and spironolactone 25 mg a day. Her LVEF remained at 27%. An implantable
cardioverter defibrillator was implanted 3 years ago, and following two shocks for ventricular tachycardia, she
was placed on amiodarone 1 year ago. She has had no further ventricular dysrhythmias since then.
She is now admitted to the hospital for a 1-month history of increased shortness of breath, which did not
improve following a trial of escalated oral diuretic doses. She does not report a cough or sputum production.
On examination, she appears mildly short of breath at rest, with a respiratory rate of 24 breaths/minute. Her
supine and standing blood pressures and heart rates are 120/80 mm Hg and 100 bpm, respectively. Blood
pressure during a Valsalva maneuver fell by 10 mm Hg during the strain phase. Body mass index (BMI) is 25
kg/m2. She is afebrile. Her lungs have diffuse coarse rales bilaterally. Her JVP is flat and there is no
hepatojugular reflux. Cardiac auscultation reveals a normal S1, S2, no S3, and no murmurs. Her abdomen is
soft, with no hepatomegaly. Extremities are warm and there are no edema.
At this point, which of the following is the likely cause of her dyspnea?
A.Amiodarone lung toxicity.
B.Atypical pneumonia.
C.Acute on chronic systolic heart failure.
D.Pulmonary embolus.
E.Volume depletion leading to a low cardiac output.
BOARD Page 77

E.Volume depletion leading to a low cardiac output.

The correct answer is A. Pneumonia is unlikely, given the absence of fever or sputum production. Acute on
chronic systolic heart failure is unlikely, given the findings of her examination including flat jugular veins, no
hepatojugular reflux, and a fall in blood pressure during the strain phase of a Valsalva maneuver. Further, a
low BNP in a patient with normal BMI argues against CHF as the cause of acute dyspnea. Pulmonary embolus
is unlikely given the diffuse bilateral rales on examination.
Her examination suggests she is not volume overloaded (see above). Similarly,the absence of orthostatic
changes in blood pressure or heart rate suggests she is not volume depleted. Further, volume depletion would
not cause diffuse bilateral ronchi. The likely cause of her dyspnea is amiodarone pulmonary toxicity.
References
1. Drazner MH, Hellkamp AS, Leier CV, et al. Value of clinician assessment of hemodynamics in advanced
heart failure: the ESCAPE trial. Circ Heart Fail 2008;1:170-7.
2. Felker GM, Cuculich PS, Gheorghiade M. The Valsalva maneuver: a bedside "biomarker" for heart failure.
Am J Med 2006;119:117-22.
3. Silva CP, Bacal F, Pires PV, et al. The importance of amiodarone pulmonary toxicity in the differential
diagnosis of a patient with dyspnea awaiting a heart transplant. Arq Bras Cardiol 2006;87:e4-7.

Question
25
of 60
A 40-year-old man is referred for evaluation of exertional chest pain and shortness of breath. His past medical
history is notable for hypercholesterolemia, for which he takes rosuvastatin 20 mg qhs. He is a lifetime
nonsmoker and does not drink alcohol. The family history is notable for his maternal grandfather and his
brother having developed HF in their 50s.
His exam is notable for a blood pressure of 110/70 mm Hg, heart rate of 80 bpm, jugular venous distention to
16 cm H20, clear lungs to auscultation, an S4 gallop, and mild peripheral edema. ECG reveals sinus rhythm,
with generally low QRS voltage and no evidence of prior MI. His echocardiogram shows mild concentric LVH
and EF is 60% with no valvular abnormalities. Initial laboratories are notable for sodium of 132 mEq/L, serum
creatinine of 1.8 mg/dl, and N-terminal portion of probrain natriuretic peptide (NT-proBNP) of 1800 pg/ml.
Which of the following is the most reasonable unifying diagnosis for this patient with HF and preserved EF?
A.Loeffler endocarditis.
B.Sarcoidosis.
C.Fabry disease.
D.Amyloidosis.
E.Glycogen storage disease.
BOARD Page 78

E.Glycogen storage disease.

The correct answer is C. This presentation with HF in the setting of preserved EF associated with chest pain,
normal coronary arteries, LVH, and renal dysfunction is most typical of Fabry disease. The family history is
consistent with transmission of an X-linked disorder, since if the brother and maternal grandfather were also
affected, his mother may have been an unaffected carrier of the mutation.
The echocardiographic findings and renal insufficiency are atypical for patients with Loeffler endocarditis,
which is not inherited and is characteristically associated with endocardial thickening and mural thrombi as
well as peripheral eosinophilia. Cardiac sarcoidosis is also an acquired rather than inherited disorder, and is
commonly associated with AV block or ventricular arrhythmias in addition to HF.
While familial amyloidosis related to elaboration of mutant transthyretin may be also associated with cardiac
and renal involvement, autosomal dominant inheritance is typical. Patients with glycogen storage diseases
typically present with HF at a young age, and even with appropriate therapy, rarely survive to adulthood.
References
1. Kushwaha SS, Fallon JT, Fuster V. Restrictive cardiomyopathy. N Engl J Med 1997;336:267-76.

Question
26
of 60
A 76-year-old white male is admitted with worsening dyspnea in the last 3 weeks. He is otherwise in good
health except for chronic back pain. He has no history of heart disease, but admits to having hypertension. He
denies any history of chest pain. He has jugular venous distention, moderate peripheral edema, and is
otherwise normal on physical examination.
Lab values show a B-type natriuretic peptide (BNP) level of 2354 pg/ml, creatinine:1.3 mg/dl, cardiac troponin
I is undetectable, glycated hemoglobin (HbA1c) is 6.9%. He is successfully diuresed with IV furosemide and his
dyspnea improves. An echocardiogram reveals a dilated LV of 6.3 cm, LVEF 25-30%, with global hypokinesis
and some regional variation. His electrocardiogram (ECG) is shown (Figure 1).

BOARD Page 79

Which of the following choices is the most appropriate next step for this patient?
A.Implantation of a cardioverter-defibrillator.
B.Initiation of amiodarone.
C.Coronary artery calcium scoring.
D.Coronary angiography.
E.Exercise electrocardiography.

The correct answer is D. The electrocardiogram shows pathologic Q waves in inferior leads. He has at least
three risk factors for CAD (age, hypertension, and diabetes-HbA1c>6.5%). Coronary angiography is a class IIa
indication in this setting. It is too early to implant a cardioverter-defibrillator despite the ventricular ectopy.
Amiodarone is not indicated for ventricular ectopy alone. Even a zero coronary artery calcium will not rule out
CAD in this patient. Exercise electrocardiography is not a recommended test to diagnose CAD in the setting of
systolic HF.
References
1. Hunt SA, Abraham WT, Chin MH, et al. 2009 focused update incorporated into the ACC/AHA 2005
Guidelines for the Diagnosis and Management of Heart Failure in Adults: a report of the American
College of Cardiology Foundation/American Heart Association Task Force on Practice Guidelines:
developed in collaboration with the International Society for Heart and Lung Transplantation. J Am Coll
Cardiol 2009;53:e1-e90.

BOARD Page 80

Question
27
of 60
An 83-year-old retired engineer with atrial fibrillation and nonischemic dilated cardiomyopathy is admitted to
the hospital with ADHF. His past medical history includes type 2 diabetes mellitus, chronic kidney disease, and
gout. His admission blood pressure is 90/76 mm Hg and his heart rate is 94 bpm and irregular. A urinary
catheter is placed for accurate assessment of fluid balance, and bumetanide is administered as a 4 mg IV bolus
every 12 hours. Despite high-dose IV diuretics, his urine output remains <70 ml/hr, his weight decreases <0.5
kg, and his serum creatinine increases from 1.4 mg/dl to 2.1 mg/dl over the first 48 hours. In addition, short
runs of nonsustained ventricular tachycardia are noted on telemetry.
Which of the following would be a reasonable next step in the management of this patient?
A.Addition of dopamine 2 mcg/kg min by continuous infusion.
B.Addition of milrinone 0.25 mcg/kg/min by continuous infusion.
C.Addition of nitroprusside 20 mcg/min by continuous infusion.
D.Placement of a pulmonary artery catheter for assessment of central hemodynamics.
E.Initiation of bedside venovenous ultrafiltration at 200 ml/hour.

The correct answer is D. Cardiorenal syndrome, or coexistent cardiac and renal


dysfunction, is common in patients with HF.1 Furthermore, during a HF hospitalization,
15-30% of patients will develop worsening renal function. This patient has several
independent risk factors for the development of worsening renal function, including
older age, diabetes, and baseline renal dysfunction. Diuretic resistance may be
contributing as well, and the next step in management is critical but empiric. Adding a
low-dose inotrope (option A) or inodilator (option B) has not been shown to improve
short-term outcomes, and may exacerbate this patient's arrhythmias. Empiric
administration of nitroprusside (option C) without arterial monitoring may be difficult
given the patient's relatively low blood pressure.
The safety and efficacy of ultrafiltration (option E) as a rescue strategy in this setting
has not been tested, and clinical reports raise concerns about further worsening of
renal function. While there is no role for routine pulmonary artery catheterization in
the management of ADHF,2 the ACCF/AHA 2009 Focused Update of the 2005
Guidelines for the Diagnosis and Management of Heart Failure in Adults suggest that
hemodynamic measurements may help in the choice and titration of parenteral
BOARD Page 81

hemodynamic measurements may help in the choice and titration of parenteral


agents, especially in patients who are refractory to initial therapy, including high-dose
or combination diuretics (Table 1).

References
1. Shah RV, Givertz MM. Managing acute renal failure in patients with acute
decompensated heart failure: the cardiorenal syndrome. Curr Heart Fail Rep
2009;6:176-81.
2. Hunt SA, Abraham WT, Chin MH, et al. 2009 focused update incorporated into the
ACC/AHA 2005 Guidelines for the Diagnosis and Management of Heart Failure in
Adults: a report of the American College of Cardiology Foundation/American
Heart Association Task Force on Practice Guidelines: developed in collaboration
with the International Society for Heart and Lung Transplantation. J Am Coll
Cardiol 2009;53:e1-e90.

Question
28
of 60
A 60-year-old woman presents to the emergency room with NYHA class IV dyspnea. She has been admitted
with HF three times in the past year. She had a large anterior myocardial infarction 2 years ago and despite
BOARD Page 82

with HF three times in the past year. She had a large anterior myocardial infarction 2 years ago and despite
coronary stenting of her left anterior descending artery, she was left with an EF of 10%, end-diastolic
dimension of 8 cm, and moderately severe mitral regurgitation. Her medications include carvedilol, lisinopril,
spironolactone, furosemide, aspirin, and simvastatin.
On exam, she is mildly tachypneic with a heart rate of 90 bpm and a blood pressure of 100/70 mm Hg. Her
venous pressure is 16 cm H20. The lungs are clear. The heart has audible and palpable gallop; murmur of
mitral regurgitation is present. The liver edge is palpable. The extremities are warm; there is pedal edema. An
ECG demonstrates sinus rhythm with an anterior myocardial infarction pattern and a narrow QRS.
A cardiopulmonary exercise test demonstrates an oxygen consumption of 9.0 cc/kg/min with a respiratory
quotient of 1.15, a ventilatory efficiency of 25, and a peak blood pressure of 130 mm Hg.
She ultimately undergoes transplantation 6 months later. She experiences one episode of cellular rejection in
the first year. Her LVEF remains normal.
Which of the following is the most likely cause of death in the first year after transplant?
A.Cellular rejection.
B.Antibody-mediated rejection.
C.Cardiac allograft vasculopathy.
D.Post-transplant lymphoproliferative disorder.
E.Infection.

The correct answer is E. Infection (particularly bacterial) remains the most common
cause of death after the first year due to aggressive immunosuppression to prevent
rejection (which is most common in the first year, but an uncommon cause of death).
Antibody-mediated rejection is less common than cellular rejection, but probably plays
a role in primary graft failure and cardiac graft vasculopathy. PTLD can be a very
aggressive lymphoma and is seen with Epstein-Barr virus serologic-positive donors and
when OKT3 is used for induction immunosuppression, but is uncommon.

Question
29
of 60
A 53-year-old man with ischemic cardiomyopathy and a left ventricular (LV) ejection fraction of 26% is
hospitalized with acute decompensated HF, his third such admission in the last 6 months.

BOARD Page 83

Medications: aspirin 81 mg qd, warfarin, furosemide 40 mg qd, carvedilol 25 mg bid, lisinopril 20 mg qd,
spironolactone 25 mg qd, atorvastatin 40 mg qd.
Physical examination: Heart rate: 110 bpm, irregular. Blood pressure: 180/90 mm Hg. Neck: Jugular venous
pressure 15 cm H20. Lungs: Crackles over two thirds of both lung fields. Heart: S3 gallop, III/VI holosystolic
murmur at the apex. Abdomen: Mild hepatomegaly. Extremities: 3+ bilateral lower extremity edema.
Laboratory: troponin I 0.54 ng/ml, B-type natriuretic peptide 1523 pg/ml, creatinine 1.8 mg/dl, international
normalized ratio 4.1.
Electrocardiogram (ECG): atrial fibrillation with rapid ventricular response, left bundle branch block (no change
from prior ECG).
Which of the following is the most likely cause of this patients current episode of decompensated HF?
A.Acute coronary syndrome.
B.Atrial fibrillation with inadequate rate control.
C.Hypertensive urgency.
D.Nonadherence to medications and/or diet.
E.Progressive LV dilatation with worsening mitral regurgitation.

The correct answer is D

Question
30
of 60
A 35-year-old man presents to the emergency department at 5:00 a.m. after awakening 2 hours ago with a
sensation of a rapid heart rate. An electrocardiogram (ECG) has been performed showing atrial fibrillation at a
rate of 140 bpm. He has no past medical history and does not consume alcohol. There is no family history of
atrial fibrillation except in relatives over the age of 75. He is an avid runner, and typically runs 5-10 miles a day.
On examination, his heart rate is 130-140 bpm with a blood pressure of 110/60 mm Hg, and the remainder of
the examination is unremarkable.
Which of the following electrophysiologic mechanisms best explains this patients initiation of atrial
fibrillation?

BOARD Page 84

A.Sympathetic activation of the IKATP channel.


B.Parasympathetic activation of the IKACH channel.
C.Parasympathetic activation of the If channel.
D.Phosphorylation of the ICa channel at slow heart rates.
E.Enhanced ITO activity during sleep.

The correct answer is B. Parasympathetic (vagal) stimulation activates muscarinic


receptors, which are coupled to the IKACH channel by a G protein. Activation of the IKACH
channel leads to slowing of the sinus rate, a decrease in atrioventricular (AV) nodal
conduction, and importantly, a decrease in the duration of the action potential in the
atrium. This last effect is thought to explain the increased incidence of atrial fibrillation
in the setting of a heightened vagal tone, such as during sleep.
References
1. Oudit GY, Backx PH. Voltage-regulated potassium channels. In: Zipes DP, Jalife J.
Cardiac Electrophysiology: From Cell to Bedside. 5th ed. Philadelphia: Saunders;
2009:29-42.
Question
31
of 60
A 53-year-old man is referred for management of AF. When he was hospitalized last month he was initiated on
propafenone. He has episodes lasting about 1-2 hours, 3-4 times per week, causing weakness and shortness of
breath. His past history includes a dilated cardiomyopathy with an LVEF of 30%, hypertension, diabetes type 2,
chronic kidney disease (stage III), peripheral arterial disease, chronic obstructive pulmonary disease, and
dyslipidemia.
With regard to antiarrhythmic therapy for AF, it should be recommended that propafenone be discontinued
because of the patients prior history of which of the following:
A.Chronic kidney disease.
B.Dilated cardiomyopathy.
C.Diabetes type 2.
D.Peripheral arterial disease.

BOARD Page 85

E.Chronic obstructive pulmonary disease.

The correct answer is B. Propafenone is contraindicated in the setting of chronic


systolic HF due to its negative inotropic effects and proarrhythmic risk as well as in
ischemic heart disease (post-MI). The other listed conditions are not contraindications
to this drug.
References
1. Fuster V, Ryden LE, Cannom DS, et al. 2011 ACCF/AHA/HRS Focused Updates
Incorporated into the ACC/AHA/ESC 2006 Guidelines for the Management of
Patients with Atrial Fibrillation: A report of the American College of Cardiology
Foundation/American Heart Association Task Force on Practice Guidelines
developed in partnership with the European Society of Cardiology and in
collaboration with the European Heart Rhythm Association and the Heart Rhythm
Society. J Am Coll Cardiol 2011;57:e101-98.

Question
32
of 60
A 57-year-old man calls your office complaining of malaise and a fever of 102.5F, 2 weeks after a catheterbased pulmonary vein isolation for the treatment of paroxysmal atrial fibrillation. He admits to some pain on
swallowing for the past 3-4 days.
Which of the following is the most appropriate next step in his management?
A.One-week treatment with Keflex 500 mg four times daily as an outpatient.
B.One-month treatment of nystatin three times daily as an outpatient.
C.Emergency room visit and urgent upper endoscopy.
D.Emergency room visit and urgent CT scan of chest.
E.Emergency room visit and treatment with intravenous tissue plasminogen activator.

The correct answer is D. Atrial esophageal fistula is a serious and often fatal
complication of catheter-based pulmonary vein isolation for the treatment of atrial
fibrillation. It often presents with high fever 1-3 weeks after the procedure, and is
often associated with painful swallowing. Septic emboli or air embolism may be
manifested as a stroke. Treatment is emergent surgical (not antibiotics or antifungals)
BOARD Page 86

manifested as a stroke. Treatment is emergent surgical (not antibiotics or antifungals)


after chest imaging with CT or magnetic resonance imaging. Instrumenting the
esophagus with endoscopy can introduce air into the left atrium and can be fatal.
References
1. Stllberger C, Pulgram T, Finsterer J. Neurological consequences of
atrioesophageal fistula after radiofrequency ablation in atrial fibrillation. Arch
Neurol 2009;66:884-7.

Question
33
of 60
A 70-year-old man presents with chronic weakness, exercise intolerance, fatigue, and near syncope. His
symptoms have developed gradually over the last several years. He reports that his symptoms occur while he
is standing; that they are relieved by lying supine; and that they are exacerbated by heat and after eating.
Electrocardiogram, echocardiogram and stress test results are normal, as are results from thyroid and adrenal
studies and a complete blood count. His physical examination is unremarkable except that his blood pressure
while supine is 170/95 mm Hg, while sitting is 130/80 mm Hg, and while standing is 70/40 mm Hg, at which
time he becomes lightheaded and near syncopal. His heart rate is 80 bpm without orthostatic change.
What is the most appropriate initial therapy to prevent this patients lightheadedness and near syncope?
A.Metoprolol.
B.Permanent pacemaker placement.
C.Clonidine.
D.Pyridostigmine.
E.Erythropoietin.

The correct answer is D. Pyridostigmine prevents orthostatic hypotension without


worsening supine hypertension.1,2
References
1. Singer W, Sandroni P, Opfer-Gehrking TL, et al. Pyridostigmine treatment trial in
neurogenic orthostatic hypotension. Arch Neurol 2006;63:513-8.
2. Raj SR, Black BK, Biaggioni I, Harris PA, Robertson D. Acetylcholinesterase
inhibition improves tachycardia in postural tachycardia syndrome. Circulation
2005;111:2734-40.
BOARD Page 87

2005;111:2734-40.

Question
34
of 60
A 55-year-old man has had three events of syncope over the past 10 weeks. The last event occurred while at
home and was witnessed by his wife, who reported that his eyes rolled back and he slumped in his chair for
about 30 seconds. He recalls perhaps feeling a funny heartbeat beforehand, but does not remember much.
He was brought to the emergency department, discharged a few hours later, and advised to increase his fluid
intake. He otherwise feels well, and he exercises daily without any problems. His past medical history includes
coronary artery disease, with an inferior wall myocardial infarction 2 years ago, for which a stent was placed.
A cardiac catheterization was performed 3 months ago for complaints of chest pain and showed an EF of 45%,
with hypokinesis of the inferior wall and a patent stent in the right coronary artery. Other past history includes
hypertension and dyslipidemia. His medications are metoprolol, atorvastatin, aspirin, and clopidogrel. His
physical examination is unremarkable, and his ECG shows sinus rhythm at 70 bpm, with evidence of an old
inferior myocardial infarction.
Which of the following should be the next step in management of this patient?
A.Electrophysiologic study.
B.Cardiac CT.
C.Nuclear stress imaging.
D.Carotid duplex ultrasound.
E.Tilt table study.

The correct answer is A. This patient has a history of coronary artery disease with
mild left ventricular dysfunction and syncope that may be due to VT, likely related to
myocardial scar. An electrophysiologic study is recommended in this situation, and if
VT is inducible, an ICD would be indicated. His recent stress imaging study does not
need to be repeated, and there are no symptoms to suggest ongoing myocardial
ischemia.
References
1. Strickberger SA, Benson DW, Biaggioni I, et al. AHA/ACCF scientific statement on
the evaluation of syncope: from the American Heart Association Councils on
Clinical Cardiology, Cardiovascular Nursing, Cardiovascular Disease in the Young,
and Stroke, and the Quality of Care and Outcomes Research Interdisciplinary
BOARD Page 88

and Stroke, and the Quality of Care and Outcomes Research Interdisciplinary
Working Group; and the American College Of Cardiology Foundation in
Collaboration with the Heart Rhythm Society. J Am Coll Cardiol 2006;47:473-84.

Question
35
of 60
An 18-year-old athlete has a history of infrequent episodes of a narrow complex tachycardia that have
previously been terminated with adenosine. He is intolerant of beta-blockers, has a delta wave with a large
dominant R wave (right bundle pattern) in lead V1, and is recommended radiofrequency ablation. In this
context, the patient can be counseled with information on which of the following?
A.5-10% chance of damage to the AV node with a 97% chance of success.
B.<2% change of requiring a permanent pacemaker, and a >95% success rate.
C.<1% chance of AV nodal damage, and an 80% chance of success.
D.0% chance of ablation on the left side, with a >95% chance of success.

The correct answer is B. The patient has Wolff-Parkinson-White syndrome, as


evidenced by the delta wave and probable orthodromic AVRT. This can be treated with
a low incidence of complications and high rate of success.1,2
References
1. Blomstrom-Lundqvist C, Scheinman MM, Aliot EM, et al. ACC/AHA/ESC guidelines
for the management of patients with supraventricular arrhythmias--executive
summary: a report of the American College of Cardiology/American Heart
Association Task Force on Practice Guidelines and the European Society of
Cardiology Committee for Practice Guidelines (Writing Committee to Develop
Guidelines for the Management of Patients With Supraventricular Arrhythmias). J
Am Coll Cardiol 2003;42:1493-531.
2. Jackman WM, Wang XZ, Friday KJ, et al. Catheter ablation of accessory
atrioventricular pathways (Wolff-Parkinson-White syndrome) by radiofrequency
current. N Engl J Med 1991;324:1605-11.

Question

BOARD Page 89

36
of 60
A 56-year-old male comes to see you for evaluation of palpitations, which have been bothersome now for at
least 2 months. Previously, he was diagnosed with severe mitral valve regurgitation. At surgery, his mitral
valve was not thought to be repairable and, therefore, he underwent mitral valve replacement with a
mechanical prosthesis and did well until recently. Other symptoms include: decreased effort tolerance,
shortness of breath with exertion, and occasional lightheadedness.
Examination reveals a well-nourished male in no acute distress. Blood pressure is 110/68 mm Hg, heart rate is
140 bpm, and jugular venous pressure is visible. Heart sounds are normal with soft systolic murmur heard best
at the apex without radiation. No diastolic murmur. Lungs: bi-basal inspiratory crackles. Abdomen: soft,
nontender. No organomegaly.
You obtain an ECG (Figure 1).

The critical zone of this rhythm most likely involves which of the following?
A.The left atrium.
B.The CTI.
C.A right atrial scar.
D.Pulmonary veins.

BOARD Page 90

The correct answer is A. The circuit shown is most likely the left atrium, a result of the
mitral valve surgery with left atrial scarring that provides a substrate for re-entry
within the left atrium. The morphology of the flutter is not typical of an isthmusdependent mechanism, and pulmonary vein firing manifests as AF.

Question
37
of 60
A young man comes to your office complaining of frequent palpitations. He is healthy and athletic, and has no
past medical history. He states that palpitations seem to be worse with exercise, but that they also bother him
when he is trying to sleep at night. He presently takes no medications. The ECG demonstrates normal sinus
rhythm with frequent PVCs and short runs of VT with a left bundle, inferior-axis morphology.
Which of the following tests should be performed at this time?
A.A transthoracic echocardiogram.
B.Cardiac catheterization.
C.Signal-averaged ECG.
D.No tests are necessary; the patient should be reassured that there is nothing wrong.
E.Genetic testing to diagnose long QT syndrome.

The correct answer is A. An echocardiogram would be helpful in excluding structural


heart disease, either causative or as a result of the arrhythmias. Cardiac
catheterization has no role in explaining the etiology of suspected idiopathic
ventricular arrhythmias. Signal-averaged ECG can be useful in distinguishing idiopathic
VT from ARVC/D, but would not be the most helpful evaluation.
Long QT syndrome does not cause uniform ventricular arrhythmias, and is initially
diagnosed by abnormalities on the surface ECG rather than random genetic testing.
Although reassurance can often be helpful, exclusion of structural heart disease is the
first objective; due to the patients symptoms, some form of treatment will likely be
necessary as well.

BOARD Page 91

Question
38
of 60
A 30-year-old man presents with 2 mm ST elevation with a saddleback morphology ECG in lead V2, which was
detected during a routine sports pre-participation screening. He experienced one syncopal episode while
standing up in a hot environment at age 18. Flecainide challenge induced a Brugada type I ECG.
Which of the following is the most correct approach for this patient?
A.To perform programmed electrical stimulation to assess VT/VF inducibility.
B.To implant an ICD for primary prevention of SCD.
C.To start oral quinidine chronic treatment.
D.To position an implantable loop monitor.
E.To perform follow-up visit with 12-lead Holter monitoring.

The correct answer is E. The detection of the presence/absence of spontaneous STsegment elevation is a critical step in Brugada syndrome risk stratification. The
absence of spontaneous ST-segment elevation is a low-risk marker, according to all
major registries.1-4 The use of quinidine for primary prophylaxis is still under
investigation. Finally, given the likely vasovagal nature of the patient fainting, loop
monitor is not the best choice. Regular follow-up every 3-6 months may be of help to
assess the presence of spontaneous ST elevation. The patient should also be
encouraged to record an ECG in case of fever.
References
1. Brugada J, Brugada R, Antzelevitch C, et al. Long-term follow-up of individuals
with the electrocardiographic pattern of right bundle-branch block and STsegment elevation in precordial leads V1 to V3. Circulation 2002;105:73-78.
2. Brugada J, Brugada R, Brugada P. Determinants of Sudden Cardiac Death in
Individuals With the Electrocardiographic Pattern of Brugada Syndrome and No
Previous Cardiac Arrest. Circulation 2003;108:3092-3096.
3. Priori SG, Napolitano C, Memmi M, et al. Clinical and molecular characterization
of patients with catecholaminergic polymorphic ventricular tachycardia.
Circulation 2002;106:69-74.
4. Probst V, Veltmann C, Eckardt L, et al. Long-term prognosis of patients diagnosed
with Brugada syndrome: Results from the FINGER Brugada Syndrome Registry.
Circulation 2010;121:635-643.
BOARD Page 92

Circulation 2010;121:635-643.

Question
39
of 60
A 60-year-old man presents with a large anterior MI. An acute intervention with stenting of the proximal left
anterior descending artery is performed within 2 hours of presentation. Three days after presentation, an
echo reveals a large anterolateral scar and EF of 20%. His NYHA classification is II.
Which of the following should be done to prevent sudden cardiac death?
A.An ICD should be implanted now because the EF is unlikely to improve.
B.Repeat the echo in 1 week and, if still not improved, implant an ICD.
C.Place patient on optimal medical therapy and reassess EF 40 days after MI. If EF is still 35%, then implant
ICD.
D.Perform EP study and, if positive, implant an ICD.

The correct answer is C. In addition to optimal medical therapy, one may consider the
use of the wearable ICD vest in these situations. At least two studies have shown that
implantation of ICDs early post-MI did not result in reduced mortality and was
associated with more complications.
References
1. Hohnloser SH, Kuck KH, Dorian P, et al. Prophylactic use of an implantable
cardioverter-defibrillator after acute myocardial infarction. N Engl J Med
2004;351:2481-8.

Question
40
of 60
BOARD Page 93

of 60
A 67-year-old man with a nonischemic cardiomyopathy, NYHA class III CHF, diabetes mellitus, and
hypertension recently had a biventricular ICD implanted. He returns for his first follow-up visit and states that
he has had no improvement in his CHF symptoms. He has been taking his CHF medical regimen regularly.
Physical examination reveals a blood pressure 105/60 mm Hg, heart rate 72 bpm, and respiratory rate 18. He
has elevated jugular venous pressure, and the rest of his exam is unremarkable.
His wound is healing well with no erythema, induration, or exudate. Interrogation of his device reveals loss of
LV capture, with normal impedance. The RV and right atrial lead parameters are stable from immediate
postoperative numbers.
Which of the following is the appropriate next step?
A.Increase LV output.
B.Program to pace RV only.
C.Chest X-ray to evaluate LV lead position.
D.Program shorter AV delay to ensure LV capture.
E.Reoperation to replace LV lead.

The correct answer is C. The most common cause of loss of LV capture immediately
post-implantation of CRT is LV lead dislodgement. This is the likely reason for
"nonresponse." A chest X-ray will reveal whether the lead has moved, and then
reoperation can be considered. One could increase the output and capture may occur;
however, there may have been movement of the lead, and that should be
investigated. Programming to pace the RV only would not give the patient the benefit
of CRT, and a shorter AV delay would not aid in LV pacing in this situation of loss of
capture.

Question
41
of 60
The patient is a 58-year-old male with witnessed sudden collapse in Chicago OHare airport. A bystander
performed CPR and an automated external defibrillator (AED) was obtained and applied. EMS was
simultaneously alarmed. The AED detected a shockable rhythm and a single shock was performed. The AED
detected a nonshockable rhythm and additional chest compressions were begun. EMS arrived and detected a
pulse. Chest compressions were halted and a pulse of 110 with a blood pressure (BP) of 150/90 was
documented. However, the patient remained unresponsive. He is transported to the nearest hospital. The ECG
BOARD Page 94

documented. However, the patient remained unresponsive. He is transported to the nearest hospital. The ECG
in the ED shows only sinus tachycardia.
Which of the following is the optimal post-resuscitation care?
A.Begin therapeutic hypothermia in the ED with rapid iced-saline infusion (1-2 liters), admit to ICU and
continue therapeutic hypothermia with a commercial mechanical surface cooling system to a goal of 33C for
24 hours. Rewarm slowly (0.3-0.5C/h) after 24 hours.
B.Provide comfort care and prepare the family for an inevitable poor outcome (i.e., either death or significant
and incapacitating anoxic brain injury).
C.Call the catheterization laboratory for emergency coronary angiography and potential PCI. Afterwards admit
the patient to the ICU for consideration of therapeutic hypothermia induction.
D.Begin therapeutic hypothermia in the ED while simultaneously calling the catheterization laboratory for
emergency coronary angiography. Attempt PCI for any acute coronary occlusion or unstable, high grade
presumed culprit lesion thought responsible for the cardiac arrest. Continue hypothermia in the
catheterization laboratory and later in the ICU, to a goal of 33C for 24 hours. Rewarm slowly (0.3-0.5 C/h)
after 24 hours.

The correct answer is D. This patient is ideal for aggressive post-resuscitation care. He
had a witnessed VFCA, received immediate bystander CPR, and received early
defibrillation with an AED. His time to ROSC was short and he was hemodynamically
stable upon arrival at the ED. He remained comatose upon presentation and hence
was a good candidate for therapeutic hypothermia to preserve central nervous system
function and improve his chance for meaningful long-term survival. Such hypothermia
should be begun immediately upon his arrival and continued for 24 hours, with a goal
of 33C.
His etiology of circulatory collapse is almost certainly cardiac, as manifested by its
sudden, unexpected occurrence. He deserves a careful search for an acute coronary
ischemic event as the precipitating cause of this cardiac arrest. This is best
accomplished by emergency coronary angiography even though his post-resuscitation
ECG does not show ST elevation. One in four of such patients are identified as having
an acutely occluded or culprit coronary lesion that is responsible for their cardiac
arrest, which can be treated with emergent coronary angiography and PCI.

Question
42
BOARD Page 95

42
of 60
A 52-year-old man underwent recent elective percutaneous coronary intervention through a femoral artery
approach. The nurse came out of his room to let you know that his systolic blood pressure is 80 mm Hg and
the patient appears drowsy.
On examination, the patient has cool extremities and is confused, with a weak peripheral pulse. Pulling back
the blanket on his bed, you see a large amount of blood and active bleeding from his vascular access site.
Immediately, you should do which of the following?
A.Give protamine to reverse the effects of the heparin he was given in the catheterization laboratory.
B.Hold pressure at the site of bleeding, give a bolus of crystalloid fluid, and order a blood transfusion.
C.Initiate dopamine and order an echocardiogram.
D.Order a complete blood cell count and have blood ready for transfusion if the hemoglobin is low.
E.Place the patient in the Trendelenburg position and recheck the blood pressure.

The correct answer is B. The patient is in hypovolemic shock as a consequence of


significant hemorrhage from his femoral artery. The immediate goals of therapy are to
control the bleeding and to rapidly replace the intravascular volume deficit. The
greater the bleeding and the longer it is left untreated, the greater the risk for
mortality. Until immediately available, crystalloid should be administered while
waiting for blood products. Dopamine, and other vasopressors, will increase the mean
arterial pressure, but are less effective when the intravascular volume is not restored.
Resuscitation efforts should not be delayed for ancillary studies, such as an
echocardiogram or laboratory values.
References
1. Falk JL, OBrien JF, Kerr R. Fluid resuscitation in traumatic hemorrhagic shock. Crit
Care Clin 1992;8:323-40.
2. Moranville MP, Mieure KD, Santayana EM. Evaluation and management of shock
states: hypovolemic, distributive, and cardiogenic shock. J Pharm Pract
2011;24:44-60.

Question
43
of 60
BOARD Page 96

of 60
The epidemiology of IE is changing, although it still remains a very serious disease worldwide, with a
continuing high mortality rate despite advances in both antibiotics and surgical intervention. Which of the
following statements is a correct response regarding the current status of IE?
A.The age of the population with IE is declining due to the increasing worldwide use of illicit intravenous
drugs.
B.Streptococcus infection remains the most common organism that results in IE.
C.The increased use of implanted devices (i.e., pacemakers and defibrillators) has resulted in a marked
increase in the number of IE cases reported.
D.In contemporary series, the most common valvular involvement with IE is the aortic valve.

The correct answer is C. The age of the population with IE is advancing due to the
increase in degenerative valve disease and the increase in the use of implanted
devices. The most common infecting organism has shifted from Streptococcal species
to Staphylococci (especially Staph aureus). The mitral valve continues to be the most
common valve involved with IE.

Question
44
of 60
Which of the following is the most likely long-term outcome in a patient with a congenital bicuspid aortic
valve?
A.Aortic root replacement for aortic dissection.
B.Surgery for aortic coarctation.
C.AVR for asymptomatic severe aortic regurgitation.
D.AVR for symptomatic severe calcific AS.
E.No need for aortic or valve surgery.

The correct answer is D. The most likely long-term outcome with a congenital
bicuspid aortic valve is superimposed calcific AS with severe obstruction leading to
BOARD Page 97

bicuspid aortic valve is superimposed calcific AS with severe obstruction leading to


symptom onset, typically at age 50-60 years. The underlying valve anatomy is a
bicuspid valve in more than one half of AVRs performed in the United States,
accounting for 60% of valve replacements under age 70 and 40% of those in older
adults. The presence of a bicuspid aortic valve is associated with a higher risk of aortic
aneurysm and dissection, but this outcome is relatively rare. A subgroup of bicuspid
valve patients present with severe aortic regurgitation as young adults, and some of
these patients will develop asymptomatic significant LV dilation prompting surgical
intervention. However, the indication for intervention is symptom onset in 80% of
adults undergoing valve replacement for aortic regurgitation. In patients with an aortic
coarctation, about 50% also have a bicuspid aortic valve. Conversely, only 5-10% of
patients with a bicuspid valve also have an aortic coarctation.
References
1. Michelena HI, Desjardins VA, Avierinos JF, et al. Natural history of asymptomatic
patients with normally functioning or minimally dysfunctional bicuspid aortic
valve in the community. Circulation 2008;117:2776-84.
2. Tzemos N, Therrien J, Yip J, et al. Outcomes in adults with bicuspid aortic valves.
JAMA 2008;300:1317-25.

Question
45
of 60
Which of the following statements is correct about when severe tricuspid stenosis is diagnosed?
A.There is a pulmonary capillary wedge >18 mm Hg and right atrial pressure >10 mm Hg.
B.A mean diastolic tricuspid gradient of 7 mm Hg is present or the valve area is 1cm2.
C.A Wilkins score of >8 is observed in the echocardiogram.
D.The right atrial pressure is increased, but the RV systolic pressure is normal.

The correct answer is B. Tricuspid stenosis is considered severe when there is a mean diastolic gradient of 7
mm Hg or the valve area is 1 cm2 in the echocardiogram. Option A is incorrect because the hemodynamic
values listed are not relevant for tricuspid stenosis. Option C, the Wilkins score, is not validated for tricuspid
stenosis and is not used for assessing the severity of the valve disease. Option D is incorrect because despite
the fact that tricuspid stenosis is associated with increased right atrial pressure, a diastolic gradient must be
demonstrated.

BOARD Page 98

Question
46
of 60
A 42-year-old woman is evaluated for palpitations. She has noted an irregular, rapid heart rhythm for the past
3-4 days. She has not experienced chest discomfort, light-headedness, syncope, orthopnea, paroxysmal
nocturnal dyspnea, or edema. She has a history of chronic severe MR. A transthoracic echocardiogram 6
months ago showed left ventricular ejection fraction of 68% with left ventricular end-diastolic diameter of 48
mm and end-systolic diameter of 33 mm. The left atrium was dilated. There was bileaflet mitral valve prolapse
with severe regurgitation (effective regurgitant orifice area of 0.4 cm2). The estimated right ventricular
systolic pressure is 34 mm Hg.
On physical examination, temperature is 37.1C, blood pressure is 125/70 mm Hg, pulse is 108 bpm and
irregular, and respiration rate is 18 breaths/minute. Lungs are clear to auscultation bilaterally. Jugular venous
pressure and carotid upstrokes are normal. There is a grade 3/6 holosystolic murmur at the apex radiating to
the left axillary region.
An electrocardiogram shows atrial fibrillation with a ventricular rate of 95 bpm with prominent voltage and
normal intervals.
In addition to starting warfarin, which of the following is the most appropriate management at this time?
A.Sotalol.
B.Direct-current cardioversion.
C.Exercise echocardiogram.
D.Mitral valve repair surgery.

The correct answer is D. This patient should undergo mitral valve repair surgery. In
patients with chronic severe MR and normal left ventricular systolic function, surgical
repair is indicated in the setting of new-onset atrial fibrillation to reduce possible longterm adverse events.
A rhythm-control strategy using an antiarrhythmic drug, such as sotalol, is not likely to
maintain sinus rhythm due to the presence of severe MR and left atrial dilation.
Similarly, direct-current cardioversion (after ruling out a left atrial thrombus) is not
likely to be associated with long-term maintenance of sinus rhythm given this patient's
severe MR and left atrial dilation.
Exercise echocardiography may be useful in patients with chronic severe MR in order
BOARD Page 99

Exercise echocardiography may be useful in patients with chronic severe MR in order


to evaluate the pulmonary artery pressure during exercise by Doppler measurements
when the estimated pulmonary artery pressure at rest is elevated (but <50 mm Hg),
but this diagnostic test does not offer additional, useful information for the patient
who already meets the criteria for valve repair surgery.
References
1. Bonow RO, Carabello BA, Chatterjee K, et al. 2008 focused update incorporated
into the ACC/AHA 2006 guidelines for the management of patients with valvular
heart disease: a report of the American College of Cardiology/American Heart
Association Task Force on Practice Guidelines (Writing Committee to revise the
1998 guidelines for the management of patients with valvular heart disease).
Endorsed by the Society of Cardiovascular Anesthesiologists, Society for
Cardiovascular Angiography and Interventions, and Society of Thoracic Surgeons. J
Am Coll Cardiol 2008;52:e1-e142.

Question
47
of 60
A 55-year-old woman presents to an outpatient clinic for symptoms of progressive dyspnea with exertion. She
has a history of rheumatic heart disease in childhood, and a heart murmur has been noted for several years.
For the past 6 months, she has experienced progressive dyspnea with walking and now is unable to walk up
one flight of stairs without dyspnea. She has also experienced mild orthopnea and bilateral lower extremity
edema. She has not had chest discomfort or palpitations.
Her past medical history is notable only for two uncomplicated pregnancies more than 20 years ago. She is not
taking any medications.
On physical examination, temperature is 37.1C, blood pressure is 134/72 mm Hg, heart rate is 82 bpm, and
respirations are 15 breaths/minute. Lung exam demonstrates dullness to percussion and reduced breath
sounds at the right base. Jugular venous pressure is elevated (5 cm above clavicle in upright, seated position)
with large cv wave noted. The apical impulse is displaced laterally. There is a regular rhythm with reduced S1
intensity. A soft opening snap is heard. There is a 3/6 holosystolic murmur heart at the apical region, which
radiates over the precordium, and a 2/6 decrescendo, low frequency murmur also at the apex. The liver span
is increased and liver is pulsatile. Bilateral lower extremity edema to the midtibia is present.
Electrocardiogram shows sinus rhythm with left ventricular hypertrophy by voltage. Transthoracic
echocardiogram shows normal left ventricular dimensions with ejection fraction of 60%. The mitral valve
leaflets are thickened with decreased mobility and mild calcification. There is moderate to severe mitral
regurgitation and moderate mitral stenosis (mean mitral gradient is 9 mm Hg, mitral valve area is 1.6 cm2).
BOARD Page 100

regurgitation and moderate mitral stenosis (mean mitral gradient is 9 mm Hg, mitral valve area is 1.6 cm2).
Echo score is 8. Severe tricuspid valve regurgitation (vena contracta 0.8 cm) is present due to poor coaptation
of tricuspid leaflets, with dilated inferior vena cava and reversal of hepatic vein flow and estimated right
ventricular systolic pressure of 60 mm Hg.
Which of the following is the most appropriate management of this patient?
A.Transesophageal echocardiography, followed by balloon mitral valvotomy.
B.Metoprolol and furosemide, and repeat echocardiogram in 6 months.
C.Mitral and tricuspid valve repair.
D.Mechanical mitral valve replacement.
E.Mechanical mitral valve replacement and tricuspid valve repair.

The correct answer is E. The patient has rheumatic mitral valve regurgitation and
stenosis, and concomitant TR. She is symptomatic and has evidence of significant
pulmonary hypertension due to mitral valve disease. Her mitral valve anatomy is not
favorable for balloon mitral valvotomy due to mitral valve regurgitation, so surgical
mitral valve replacement is indicated. Because of the limited durability of biologic
prosthetic valves in the mitral position, mechanical valve replacement is favored for
younger patients if the patient is a candidate for life-long anticoagulation. Tricuspid
valve repair is recommended at the time of mitral valve surgery in patients with
concomitant severe TR, and has been shown to result in improved long-term outcome
compared with MV surgery alone.
Option A is incorrect because balloon mitral valvotomy is contraindicated when the
degree of MV regurgitation is moderate or greater.
Option B is incorrect. Although medical therapy of mitral stenosis with metoprolol and
diuretics may provide symptomatic improvement, the patient has evidence of severe
pulmonary hypertension, indicating the need for MV intervention.
Option C is incorrect. Because of the rheumatic etiology of the patient's MV disease
and abnormal leaflet anatomy with thickening, calcification, and stenosis, mitral valve
repair is unlikely to provide a durable result without recurrent regurgitation or
stenosis.
Option D is incorrect. Although the degree of TR may be reduced by improvement of
pulmonary hypertension after mitral valve surgery, TR generally remains and may
progress (particularly in cases of rheumatic heart disease) after successful mitral valve
replacement.
BOARD Page 101

replacement.
References
1. Bonow RO, Carabello BA, Chatterjee K, et al. 2008 focused update incorporated
into the ACC/AHA 2006 guidelines for the management of patients with valvular
heart disease: a report of the American College of Cardiology/American Heart
Association Task Force on Practice Guidelines (Writing Committee to revise the
1998 guidelines for the management of patients with valvular heart disease).
Endorsed by the Society of Cardiovascular Anesthesiologists, Society for
Cardiovascular Angiography and Interventions, and Society of Thoracic Surgeons. J
Am Coll Cardiol 2008;52:e1-142.

Question
48
of 60
A 25-year-old man is seen for outpatient evaluation of CHD. He was known to have a large VSD as a child, and
this was never surgically corrected because pulmonary hypertension was evident at the time of discovery. He
works full time at a bank. He has slowed down somewhat in recent years, but is still able to do most of what
he wants as long as he takes his time. He is not on any medications. Vital signs: pulse 72 bpm and regular; BP
120/80 mm Hg; respirations 16 breaths per minute; pulse oximetry 88%. He has clubbing of both fingers and
toes.
Which of the following is the most likely kind of murmur in this patient?
A.Pansystolic murmur maximal at the left lower sternal border.
B.Pansystolic murmur maximal at the second left interspace.
C.High-pitched diastolic murmur.
D.Low-pitched diastolic murmur.
E.Mid- and late systolic murmur maximal at the left lower sternal border.

The correct answer is C. The most likely murmur would be a high-pitched diastolic murmur reflecting
pulmonary arterial hypertension and high-pressure pulmonary regurgitation (Graham Steell murmur). Even
though a pansystolic murmur maximal at the left lower sternal border is characteristic of a small to moderate
VSD, a large VSD does not cause a pansystolic murmur. A pansystolic murmur is rarely if ever heard at the
second left interspace, the sole exception probably being severe mitral valve prolapse with posterior leaflet
prolapse. A low-pitched diastolic murmur would not fit with what we know about this patient. A mid- and late
systolic murmur would also not be expected.

BOARD Page 102

Question
49
of 60
A 20-year-old woman presents during the second trimester of pregnancy with blood pressure of 210/100 mm
Hg. She reports that in childhood, she was found to have CoA; however, no interventions were performed and
she has had mildly elevated systemic blood pressure prior to conception.
Which of the following statements is true of the patient?
A.Her offspring will have a 50% chance of having congenital heart disease.
B.Her blood pressure will return to normal in the third trimester of pregnancy.
C.An LVOT peak systolic gradient of >30 mm Hg by echocardiography is associated with adverse maternal
outcomes.
D.CMR with gadolinium administrated should be performed immediately.
E.Angiotensin-converting enzyme (ACE) inhibition should be started immediately to lessen the afterload on
the left ventricle.

The correct answer is C. Unrepaired LVOT obstruction lesions are problematic during
pregnancy. In a multicenter study of over 600 pregnancies in women with heart
disease, a LVOT peak systolic gradient of >30 mm Hg by echocardiography was
associated with adverse maternal outcomes. Imaging should be done to confirm the
diagnosis; however, the contrast agent gadolinium should be avoided in pregnancy
due to uncertain effects on the fetus. ACE inhibitors are contraindicated in pregnancy
due to the placental transfer and potential teratogenic effects to the unborn child. In
the setting of maternal congenital heart disease (CHD), there is an increased chance of
offspring affected with CHD. However, the risk does not approach 50%, except in the
setting of certain genetic associations (e.g., DiGeorge syndrome).
References
1. Siu SC, Sermer M, Colman JM, et al. Prospective multicenter study of pregnancy
outcomes in women with heart disease. Circulation 2001;104:515-21.

Question

BOARD Page 103

50
of 60
An 18-year-old man presents for a sports participation physical prior to starting college as a middle-distance
runner. He is a scholarship athlete at a Division I school, and has never noticed any exercise limitations. He
reports palpitations over the last several years that occur every 1-2 months, last from 10 seconds to 5
minutes, and spontaneously remit. The palpitations are associated with dizziness, but he has not had syncope.
On examination, a faint systolic murmur is heard at the left lower sternal border. A resting ECG and
echocardiogram are obtained. The echocardiogram is shown (Figure 1).

Which of the following diagnostic ECG findings is expected?


A.Sinus bradycardia, otherwise normal.
B.Normal sinus rhythm with first-degree AV block.
C.Complete heart block.
D.Right bundle branch block.
E.Pre-excitation.

The correct answer is E. In a patient with Ebstein's anomaly and palpitations consistent with a tachycardia,
the most likely cause is an AV re-entrant tachycardia related to a bypass tract which often manifests as preexcitation on a resting ECG. Although atrial arrhythmias such as atrial flutter or atrial fibrillation occur, those
rhythms tend to be seen in patients with more severe disease and heart failure. Severe disease would not be
expected in a patient with sufficient cardiopulmonary fitness to be a scholarship track athlete, and the
BOARD Page 104

expected in a patient with sufficient cardiopulmonary fitness to be a scholarship track athlete, and the
echocardiogram shows relatively mild disease. Though the other findings are commonly found in patients with
Ebstein's anomaly, none would be expected to be related to symptomatic palpitations.

Question
51
of 60
Some coronary anomalies may not create symptoms, but their course must be appreciated at time of surgical
intervention for repair of the heart condition. Which of the following coronary anomalies may directly
interfere with a surgical corrective procedure?
A.LAD artery origin from the RCA in tetralogy of Fallot.
B.RCA origin from the anterior cusp in corrected transposition of the great vessels (congenitally corrected
transposition).
C.Ectopic origin of the conus artery from a second right cusp ostium in hypertrophic cardiomyopathy.
D.Coronary ectasia in patients undergoing CABG.
E.Anomalous LCX artery from the RCA in patients undergoing atrial septal defect repair.

The correct answer is A. The anomalous origin of the LAD artery from the RCA in
tetralogy of Fallot crosses anteriorly to the RV outflow tract in 5-7% of all patients with
tetralogy. Surgical patch repair of the RV outflow obstruction in tetralogy of Fallot can,
therefore, result in injury to this coronary. When encountered, a conduit over the
coronary from the RV to the PA is usually created to prevent coronary injury. The RCA
usually arises from the anterior cusp in congenitally corrected transposition. The most
common anomaly is the ectopic origin of the conus artery from a second ostium in the
right cusp, and it has no significance. Coronary ectasia is common in patients
undergoing CABG and prevents little technical issue. Finally, atrial septal defect repair
occurs through an incision in the RA lateral wall, and the location of an anomalous LCX
artery does not interfere with the procedure.

Question
BOARD Page 105

52
of 60

A 50-year-old woman presents to the Cardiovascular Medicine Clinic after a recent diagnosis of PE and
bilateral calf DVT. She denies any trauma, medical illness, surgery, immobilization, or other precipitating
factors preceding her PE and DVTs. She has a history of hyperlipidemia treated with a statin. She is currently
taking warfarin and her last international normalized ratio (INR) was 2.3.
She is normotensive in the office with a resting oxygen saturation of 99% on room air. She is obese and has
varicose veins with mild pitting edema at the ankles. Her ECG demonstrates sinus rhythm with no
abnormalities. Cardiac biomarkers during her admission were normal. Testing for thrombophilias was
conducted during her admission and was unremarkable.
Based on the patients history, which of the following is the most appropriate anticoagulation regimen?
A.Oral anticoagulation with warfarin for 6 months with a target INR of 2-3 and then indefinitely with a target
INR of 1.5-2.
B.Oral anticoagulation with warfarin for 6 months with a target INR of 2-3.
C.Oral anticoagulation with for 3 months with a target INR of 2-3.
D.LMWH monotherapy via subcutaneous self-injection indefinitely.
E.LMWH via subcutaneous self-injection for 3 months.

The correct answer is A. Answer A is correct because the patients presentation is


consistent with unprovoked or idiopathic VTE. The patient has an increased risk of
recurrent events and would benefit from indefinite duration anticoagulation if the
bleeding risk is low. After the initial 6 months of warfarin anticoagulation with an INR
goal of 2-3, she may be treated indefinitely with either a low-intensity regimen (INR
1.5-2) or continue at her current intensity (INR 2-3). Both regimens have been
validated.
Answers B and C are incorrect because the patient has suffered unprovoked VTE and
neither provides protection against recurrent events after the initial treatment period.
Answer D is incorrect because LMWH monotherapy is reserved for patients with active
cancer and VTE or patients in whom warfarin therapy is not possible (for example, due
to drug allergy). Answer E is incorrect because the patient has suffered unprovoked
VTE and the proposed short-term regimen does not provide protection against
recurrent events after the initial treatment period.
BOARD Page 106

recurrent events after the initial treatment period.


References
1. Goldhaber SZ, Piazza G. Optimal duration of anticoagulation after venous
thromboembolism. Circulation 2011;123:664-67.
2. Ridker PM, Goldhaber SZ, Danielson E, et al. Long-term, low-intensity warfarin
therapy for the prevention of recurrent venous thromboembolism. N Engl J Med
2003;348:1425-34.
3. Kearon C, Kahn SR, Agnelli G, Goldhaber S, Raskob GE, Comerota AJ, on behalf of
the American College of Chest Physicians. Antithrombotic therapy for venous
thromboembolic disease: American College of Chest Physicians Evidence-Based
Clinical Practice Guidelines (8th Edition). Chest 2008;133:454S--545S.

Question
53
of 60
A 60-year-old woman presents to her primary care doctor with an approximate 2- to 3-month history of lowgrade fevers, myalgias, and weight loss. Labs are notable for an elevated erythrocyte sedimentation rate (ESR;
72 mm/hr) and elevated total white blood cell count (12.6 K/L). A transthoracic echocardiogram and then a
transesophageal echocardiogram are performed (Video 1).

http://www.cardiosource.com/img/A8_20_1_Aguilar09.mov
The next best appropriate therapeutic step is which of the following?
A.Referral to cardiac surgery.
B.Blood cultures followed by empiric broad-spectrum antibiotics.
C.Referral to the hospital for initiation of parental anticoagulant therapy.
D.Referral for a cardiac MRI.

The correct answer is A. The video demonstrates a transesophageal echocardiogram with a large left atrial
myxoma (Video 1). The appearance is typical for an atrial myxoma, with a location in the left atrium and
attachment at the interatrial septum near the fossa ovalis. Further imaging is not typically necessary, given the
characteristic appearance. The constitutional signs and symptoms and laboratory abnormalities are believed
to be related to interleukin-6 release by the tumor cells. The treatment of symptomatic myxomas is surgical
resection of the tumor.

BOARD Page 107

Question
54
of 60
An HIV-infected patient comes to your office because he says his friend told him he was going to have a heart
attack, but he says he feels fine. He is a 42-year-old black male with a long-standing HIV infection. He has not
been in care for 2 years, but he remembers that he was told last time he has AIDS. He smokes one-half pack
per day and drinks occasionally. You do some routine tests that show a blood pressure of 127/88 mm Hg,
LDL is 177 mg/dl, HDL is 27 mg/dl, and TG are 334 mg/dl.
Which of the following is the most efficient step to decrease his CV risk based on the Framingham risk score?
A.Refer him to your infectious diseases colleague so he can be restarted on antiretrovirals.
B.Start him on pravastatin.
C.Start him on extended-release niacin.
D.Start him on 2 g fish oil daily.
E.Counsel him on smoking cessation.

The correct answer is E. HIV-infected individuals have high rates of smoking, and
smoking cessation is perhaps the most modifiable risk factor to minimize CVD risk in
HIV-infected individuals. While this patient does need to be started on antiretrovirals,
as well as needs to improve his lipid profile, smoking cessation would have the most
immediate effect on his overall health and CVD risk.
References
Friis-Moller N, Sabin CA, Weber R, et al. Combination antiretroviral therapy and the
risk of myocardial infarction. N Engl J Med 2003;349:1993-2003.

Question
55
of 60
A 53-year-old woman with metastatic colon cancer is undergoing active treatment with FOLFOX, containing
leucovorin, 5-FU, and oxaliplatin. She presents with sudden-onset chest pain while walking to her
chemotherapy appointment. An ECG shows ST-segment elevation in the anterolateral leads. She is taken
emergently to the catheterization laboratory and has a 40% proximal left anterior descending artery (LAD)
BOARD Page 108

emergently to the catheterization laboratory and has a 40% proximal left anterior descending artery (LAD)
lesion, but no other epicardial disease. Her cTnI peaks at 4.3 ng/ml. Her echocardiogram reveals an LVEF of
40% with anterolateral hypokinesis.
Which of the following is the best therapy for this patient?
A.Aspirin, statin, nitrates, and ACE inhibitor.
B.Aspirin, statin, calcium channel blocker, and ACE inhibitor.
C.Aspirin, statin, beta-blocker, and ACE inhibitor.
D.Aspirin, beta-blocker, and statin.
E.Angioplasty and stenting of LAD, followed by medical therapy.

The correct answer is A. 5-FU cardiotoxicity occurs with an incidence of 1-18%. The
most frequent side effects are chest pain, myocardial ischemia, hypotension, ECG
changes (i.e., ST-segment changes, T-wave abnormalities), and arrhythmias.
Cardiotoxicity typically occurs early, during the first to third dose of therapy, and is
more common after high-dose, continuous therapy. The pathophysiology behind these
effects is unclear; however, the leading hypothesis is that 5-FU leads to coronary
vasospasm and ischemia, as seen in this case.
5-FU therapy should be discontinued in case of cardiotoxicity. In light of vasospasm,
nitrates and calcium channel blockers have been recommended in 5-FU-induced
cardiotoxicity. However, in this case, nitrates would be the better choice given the
patients diminished EF. Angioplasty and stenting would not be recommended since
the underlying lesion alone is not flow-limiting.
References
1. Yeh ET, Bickford CL. Cardiovascular complications of cancer therapy: incidence,
pathogenesis, diagnosis, and management. J Am Coll Cardiol 2009;53:2231-47.
Question
56
of 60
Which of the following is the recommended current treatment of CSR in heart failure?
A.Optimize medical therapy.
B.Supplemental CO2 therapy.

BOARD Page 109

C.Sedative-hypnotic administration.
D.Supplemental oxygen therapy.

The correct answer is A. CSR often occurs in the setting of left ventricular heart
failure. The severity and presence of disease is related to an elevated left atrial
pressure. Treatment that reduces left atrial pressure improves CSR. Although
supplemental CO2 can be used to increase the pCO2 above the apneic threshold and
improve CSR, the long-term benefit of this approach is not clear. Supplemental oxygen
therapy tends to have little effect on repetitive arousals and central apneas seen in
CSR, although the degree of hypoxemia might be improved. Finally, the use of
sedative-hypnotics has not been studied adequately in CSR.
References
1. Solin P, Bergin P, Richardson M, Kaye DM, Walters EH, Naughton MT. Influence of
pulmonary capillary wedge pressure on central apnea in heart failure. Circulation
1999;99:1574-9.
2. Fleetham J, Ayas N, Bradley D, et al.; Canadian Thoracic Society guidelines:
diagnosis and treatment of sleep disordered breathing in adults. Can Respir J
2006;13:387-92.
3. Sharma B, Owens R, Malhotra A. Sleep in congestive heart failure. Med Clin North
Am 2010;94:447-64.

Question
57
of 60
The mother of a 13-year-old high school football player sits next to you at a local game and says to you: Ive
heard a lot about high school student athletes getting cardiac tests prior to participating in sports. At what age
should my child have an ECG?
Which of the following should you tell her?
A.Routine ECGs are not recommended before participating in high school athletics.
B.ECGs are recommended at age 10.
C.ECGs are recommended at age 16.
D.Echocardiograms, but not ECGs are recommended for screening.
BOARD Page 110

D.Echocardiograms, but not ECGs are recommended for screening.

The correct answer is A. In a 2007 document, the AHA stated many concerns about
implementation of ECG screening in the United States. Thorough cardiac examinations
(which include the 12-point AHA elements) were recommended. Some of the concerns
included: lack of trained workforce to read the athlete ECG, already low incidence of
sudden cardiac arrest in this population, lack of a randomized trial, and the issue of
large number of false-positive ECGs due to athletic adaptations. This can be as high as
40%. Data published in JAMA in 2006 were not randomized data, but retrospective
data.
References
1. Maron BJ, Thompson PD, Ackerman MJ, et al. Recommendations and
considerations related to preparticipation screening for cardiovascular
abnormalities in competitive athletes: 2007 update: a scientific statement from
the American Heart Association Council on Nutrition, Physical Activity, and
Metabolism: endorsed by the American College of Cardiology Foundation.
Circulation 2007;115:1643-455.
Question
58
of 60
A 35-year-old man, whose father died of an aortic dissection at the age of 50 years, presents to your office for
evaluation. Given the possibility of a familial aortic syndrome, you order an echocardiogram that reveals a
tricuspid aortic valve with normal appearing leaflets and trace central aortic insufficiency, a 5.6 cm aortic root,
a 3.4 cm ascending thoracic aorta, and normal left ventricle size and function. Which of the following is the
preferred management strategy?
A.Annual surveillance echocardiograms to monitor the aortic diameter, waiting until the aortic root diameter
reaches 6.0 cm before recommending aortic repair.
B.Aortic root replacement using a composite aortic graft with a mechanical prosthetic valve.
C.Aortic root replacement using a composite aortic graft with a bioprosthetic valve.
D.Aortic root replacement using a composite aortic graft with a stentless bioprosthesis.
E.A valve-sparing aortic root repair (David procedure).

The correct answer is E. In the setting of an aortic root aneurysm, when the patient's native valve has healthy
leaflets, it is currently preferable to perform a valve-sparing root repair (the David procedure) to avoid the
inherent risks associated with prosthetic valves. For nonsyndromic aortic root aneurysms, the threshold for
aortic repair is a diameter of 5.5 cm; however, in patients with Marfan syndrome, BAV, or a family history of
BOARD Page 111

aortic repair is a diameter of 5.5 cm; however, in patients with Marfan syndrome, BAV, or a family history of
TAA or dissection, the threshold for surgery is a diameter of 5.0 cm. Historically, a composite aortic graft with
a mechanical prosthetic valve (the Bentall procedure) had been favored to repair aortic root aneurysms;
however, now a valve-sparing root repair is preferred because the patient's long-term prognosis is more
favorable with his native aortic valve than a mechanical prosthetic valve.

Question
59
of 60
A number of risk conditions are unique to women, yet these conditions are not routinely included in the usual
visit to a cardiovascular specialist.
Which of the following should be added?
A.Polycystic ovarian syndrome.
B.Pre-eclampsia or eclampsia.
C.Low or very high birth weight children.
D.Hypothalamic hypoestrogenemia.
E.All of the above.

The correct answer is C. Although the exact mechanisms are unknown, a number of
studies have documented that all of the above conditions are associated with
increased risk of CAD-related adverse outcomes later in life. They deserve
consideration in the intake assessment for cardiovascular disease risk for all women.
The ability to quantify microvascular blood flow in resting and stress states allow for
determination of CFR, the ratio of maximal hyperemic to resting flow rates. Mild
variation in a normal cutoff exist between different quantification methods, such that
CFR <2.0 and 2.5 is generally considered abnormal. CFR typically is diminished when
measured in a vessel with epicardial stenosis, but in absence of this, decreased CFR is
representative of coronary microvascular dysfunction. Patients with underlying
microvascular dysfunction need to be managed aggressively to decrease long-term
risks of developing significant mortality and morbidity.

Question
BOARD Page 112

Question
60
of 60
Anticoagulant therapy is indicated in all of the following scenarios EXCEPT:
A.NSTEMI due to plaque erosion with superimposed thrombosis.
B.Chest pain experienced in a patient with known coronary disease in the setting of severe active
gastrointestinal bleeding.
C.Unstable angina due to acute plaque rupture in the setting of active exercise.
D.In situ thrombosis in a prior coronary stent presenting as chest pain in a patient who has stopped taking
antiplatelet therapy.

The correct answer is B. All of the clinical scenarios except option B constitute forms
of UA/NSTEMI that are associated with coronary thrombosis as the root cause of the
unstable acute coronary syndrome. In these scenarios, anticoagulant therapy is
indicated as soon as possible following clinical presentation per the ACCF/AHA
guidelines. Option B is a form of secondary ACS, or demand ischemia in a patient
with fixed coronary artery disease. In this case, the patients chest pain is secondary to
another cause, active severe gastrointestinal bleeding, a scenario in which
anticoagulant therapy would be contraindicated.
References
1. Wright RS, Anderson JL, Adams CD, et al. 2011 ACCF/AHA focused update
incorporated into the ACC/AHA 2007 Guidelines for the Management of Patients
with Unstable Angina/Non-ST-Elevation Myocardial Infarction: a report of the
American College of Cardiology Foundation/American Heart Association Task
Force on Practice Guidelines developed in collaboration with the American
Academy of Family Physicians, Society for Cardiovascular Angiography and
Interventions, and the Society of Thoracic Surgeons. J Am Coll Cardiol
2011;57:e215-367.

BOARD Page 113

BOARD Page 114

BOARD Page 115

Potrebbero piacerti anche